Combo with "Infectious" and 13 others

Réussis tes devoirs et examens dès maintenant avec Quizwiz!

A nurse is teaching a preoperative client about postoperative breathing exercises. What information should the nurse include? (Select all that apply.) 1. Take short, frequent breaths 2. Exhale with the mouth open wide 3. Perform the exercises twice a day 4. Place a hand on the abdomen while feeling it rise 5. Hold the breath for several seconds at the height of inspiration

4. Place a hand on the abdomen while feeling it rise 5. Hold the breath for several seconds at the height of inspiration

A client arrives for a vaccination at an influenza prevention clinic. A nursing assessment identifies a current febrile illness with a cough. The nurse should: 1. Give the vaccine 2. Administer aspirin with the vaccine 3. Hold the vaccine and notify the health care provider 4. Reschedule administration of the vaccine for the next month

4. Reschedule administration of the vaccine for the next month

A client has a chest tube inserted to treat a right hemopneumothorax. In which position should the nurse place the client to facilitate chest drainage? 1. Supine 2. Left Sims 3. Immobilized 4. Right side-lying

4. Right side-lying

What should a nurse do when caring for a client with continuous bladder irrigation? 1. Measure the output hourly. 2. Monitor the specific gravity of the urine. 3. Irrigate the catheter with saline three times daily. 4. Subtract the amount of irrigant instilled from the output.

4. Subtract the amount of irrigant instilled from the output.

A client with emphysema has a history of smoking two packs of cigarettes a day. What is the best approach for the nurse to help the client stop smoking? 1. Teach pursed-lip breathing 2. Encourage the client to reduce emotional stress 3. Obtain a referral to a smoking cessation program in the community 4. Suggest that the client limit smoking to one pack of cigarettes a day

4. Suggest that the client limit smoking to one pack of cigarettes a day

On the third postoperative day after a subtotal thyroidectomy for a tumor, a client complains of a "funny, jittery feeling." On the basis of this statement, the nurse's best action is to: 1. Explain that this reaction is expected and not a concern 2. Take the vital signs and place the client in a high-Fowler position 3. Request stat serum calcium and phosphorus levels and chart the results 4. Test for Chvostek's and Trousseau's signs and notify the health care provider of the complaints

4. Test for Chvostek's and Trousseau's signs and notify the health care provider of the complaints

A lithotripsy to break up renal calculi is unsuccessful, and a nephrolithotomy is performed. Which postoperative clinical indicator should the nurse report to the health care provider? 1. Passage of pink-tinged urine 2. Pink drainage on the dressing 3. Intake of 1750 mL in 24 hours 4. Urine output of 20 to 30 mL/hr

4. Urine output of 20 to 30 mL/hr **Output should be at least 30 mL/hr or more; a decreased output may indicate obstruction or impaired kidney function.

A nurse is caring for a client who has been taking several antibiotic medications for a prolonged time. Because long-term use of antibiotics interferes with the absorption of fat, the nurse anticipates a prescription for: 1. High fat diet 2. Supplemental cod liver oil 3. Total parenteral nutrition (TPN) 4. Water-soluble forms of vitamins A and E

4. Water-soluble forms of vitamins A and E

Which is the best advice the nurse can give regarding foot care to a client diagnosed with diabetes? 1. Remove corns on the feet 2. Wear shoes that are larger than the feet 3. Examine the feet weekly for potential sores 4. Wear synthetic fiber socks when exercising

4. Wear synthetic fiber socks when exercising

Discharge teaching for a client with hypercholesterolemia includes nutritional instructions for a diet low in saturated fat. Which items included by the client on a list of foods to avoid supports the nurse's conclusion that teaching has been effective? 1. High fiber foods 2. Canned vegetables 3. Citrus fruits and juices 4. Whole milk and hard cheeses

4. Whole milk and hard cheeses

-A client's problem with ineffective control of type 1 diabetes is pinpointed as a sudden decrease in blood glucose level followed by rebound hyperglycemia. What should the nurse do when this event occurs? 1. Give the client 8 oz of orange juice. 2. Seek a prescription to increase the insulin dose at bedtime. 3. Encourage the client to eat smaller, more frequent meals. 4.Collaborate with the health care provider to alter the insulin prescription.

4.Collaborate with the health care provider to alter the insulin prescription.

A nurse is preparing to give a client a tepid bath and uses a bath thermometer to test the water temperature. What is the acceptable temperature range for a tepid bath? 1 92° to 94° F 2 95° to 97° F 3 98° to 100° F 4 101° to 103° F

98° to 100° F

A client who had a transurethral resection of the prostate is to be discharged from the outpatient surgical department. Which client statement indicates to the nurse that discharge teaching about self-care is understood?

"I will notify my health care provider if persistent bleeding occurs."

A client is scheduled to have a coronary artery bypass graft (CABG). The client's spouse asks what the benefit of the surgery is. How should the nurse respond?

"This surgery significantly decreases symptoms in most clients."

A diet that contains restricted amounts of protein, sodium, and potassium has been prescribed for a client with end-stage renal disease who is receiving dialysis. The nurse is providing dietary instructions and evaluates that the teaching is effective when the client says:

1 "I should avoid using salt substitutes."

A client had thoracic surgery. The nurse should monitor for what clinical manifestations that may indicate acute pulmonary edema? Select all that apply. 1 . Crackles 2 . Cyanosis 3 . Dyspnea 4 . Bradypnea 5 . Frothy sputum

1 . Crackles 2 . Cyanosis 3 . Dyspnea 5 . Frothy sputum

Immediately after cataract surgery a client complains of feeling nauseated. The nurse should:

1 Administer the prescribed antiemetic

A client with end-stage renal disease is hospitalized. For what signs and symptoms of complications should the nurse monitor the client? (Select all that apply.)

1 Anemia 2 Dyspnea 4 Anasarca

A client has a diagnosis of trigeminal neuralgia. When assessing the client's trigeminal nerve function, the nurse should evaluate:

1 Corneal sensation

A client had thoracic surgery. The nurse should monitor for what clinical manifestations that may indicate acute pulmonary edema? Select all that apply.

1 Crackles 2 Cyanosis 3 Dyspnea 5 Frothy sputum

A client with acute renal failure moves into the diuretic phase after one week of therapy. For which signs during this phase should the nurse assess the client? (Select all that apply.)

1 Dehydration 2 Hypovolemia

A client is diagnosed with thrombophlebitis. The client states, "I am worried about getting a clot in my lungs that will kill me." The nurse's initial response should be to:

1 Discuss the client's concerns

A client diagnosed with bone cancer of the leg will receive radiation therapy as part of the treatment plan. The client has voiced concern about the side effects of the radiation treatments. The nurse will prepare the patient for which major side effects of radiation therapy? (Select all that apply.)

1 Fatigue 5 Altered taste sensations

The nurse is providing dietary teaching to a 40-year-old client who is receiving hemodialysis. The nurse should encourage the client to include what in the client's dietary plan?

1 Rice

A female client has a history of recurrent urinary tract infections. What should the nurse include in the teaching plan when teaching the client about health practices that may help decrease future urinary tract infections?

1 Wear cotton underpants

The nurse is providing instructions about foot care for a client with diabetes mellitus. What should the nurse include in the instructions?(Select all that apply.) 1. Wear shoes when out of bed. 2. Soak the feet in warm water daily. 3. Dry between the toes after bathing. 4. Remove corns as soon as they appear. 5. Use a heating pad when the feet feel cold

1 Wear shoes when out of bed. 3 Dry between the toes after bathing.

A client who has had a transurethral prostatectomy (TURP) experiences dribbling after the indwelling catheter is removed. To address this problem, an appropriate nursing response is: 1. "Increase your fluid intake and urinate at regular intervals." 2. "I know you're worried, but it will go away in a few days." 3. "Limit your fluid intake and urinate when you first feel the urge." 4. "The catheter will have to be reinserted until your bladder regains its tone."

1. "Increase your fluid intake and urinate at regular intervals."

A client with end-stage renal disease is hospitalized. For what signs and symptoms of complications should the nurse monitor the client? (Select all that apply.) 1. Anemia 2. Dyspnea 3. Jaundice 4. Anasarca 5. Hyperexcitability

1. Anemia 2. Dyspnea 4. Anasarca **Anemia is due to decreased production of erythropoietin by the kidneys, which causes decreased erythropoiesis by bone marrow. Dyspnea is a result of fluid overload, which is associated with chronic kidney failure. Diffuse, profound interstitial edema caused by altered capillary permeability and decreased cellular perfusion (anasarca) is associated with end-stage renal disease.

Despite receiving 2900 mL intake for two days, the client's urine output has progressively diminished. The nurse identifies that the urinary output is less than 40 mL/hr over the past three hours. What action should the nurse take? 1. Assess breath sounds and obtain vital signs 2. Decrease the intravenous (IV) flow rate and increase oral fluids 3. Insert an indwelling catheter to facilitate emptying of the bladder 4. Check for dependent edema by assessing the lower extremities

1. Assess breath sounds and obtain vital signs

A client has an endotracheal tube and is receiving mechanical ventilation. Periodic suctioning is necessary and the nurse follows a specific protocol when performing this procedure. Select in order of priority the nursing actions that should be taken when suctioning. 1. Assess client's vital signs and lung sounds 2. Insert the catheter without applying suction 3. Rotate the catheter while suction is applied 4. Administer oxygen via a ventilation bag

1. Assess client's vital signs and lung sounds, 4. Administer oxygen via a ventilation bag , 2. Insert the catheter without applying suction, 3. Rotate the catheter while suction is applied

A nurse is caring for a client with end-stage renal disease who has a mature arteriovenous (AV) fistula. What nursing care should be included in the client's plan of care? (Select all that apply.) 1. Auscultate for a bruit. 2. Palpate the site to identify a thrill. 3. Irrigate with saline to maintain patency. 4. Avoid drawing blood from the affected extremity. 5 . Keep the fistula clamped until ready to perform dialysis.

1. Auscultate for a bruit. 2. Palpate the site to identify a thrill. 4. Avoid drawing blood from the affected extremity.

A primary health care provider prescribes three stool specimens for occult blood for a client who complains of blood-streaked stools and a 10-pound weight loss in one month. To ensure valid test results, the nurse should instruct the client to: 1. Avoid eating red meat before testing 2. Test the specimen while it is still warm 3. Discard the day's first stool and use the next three stools 4. Take three specimens from different sections of the fecal sample

1. Avoid eating red meat before testing

A client is admitted via the emergency department with the tentative diagnosis of diverticulitis. Which test commonly is prescribed to assess for this problem? 1. Computed tomography (CT) scan 2. Gastroscopy 3. Colonoscopy 4. Barium enema

1. Computed tomography (CT) scan **A CT scan with contrast is the test of choice for diverticulitis because it effectively reflects the involved colon.

A nurse is assessing a client with hypothyroidism. Which clinical manifestations should the nurse expect the client to exhibit? (Select all that apply.) 1. Cool skin 2. Photophobia 3. Constipation 4. Periorbital edema 5. Decreased appetite

1. Cool skin 3. Constipation 4. Periorbital edema 5. Decreased appetite

A nurse is teaching a client about drug therapy for gonorrhea. Which fact about drug therapy should the nurse emphasize? 1. Cures the infection 2. Prevents complications 3. Controls its transmission 4. Reverses pathological changes

1. Cures the infection

Which clinical findings should the nurse expect when assessing a client with hyperthyroidism? (Select all that apply.) 1. Diarrhea 2 Listlessness 3. Weight loss 4. Bradycardia 5. Decreased appetite

1. Diarrhea 3. Weight loss

The nurse provides a dietary list to a client who is taking oral anticoagulants with foods that should be avoided because they are high in vitamin K. What foods should be included on the list? (Select all that apply.) 1. Eggs 2. Liver 3. Cheese 4. Squash 5. Chicken

1. Eggs 2. Liver 3. Cheese

The nurse is assessing a client with hyperthyroidism. Which clinical indicators are consistent with this diagnosis? (Select all that apply.) 1. Emotional lability 2. Dyspnea on exertion 3. Abdominal distension 4. Decreased bowel sounds 5. Hyperactive deep tendon reflexes

1. Emotional lability 2. Dyspnea on exertion 5. Hyperactive deep tendon reflexes

Which action should be included in the plan of care for a client who has had pelvic surgery? 1. Encouraging the client to ambulate in the hallway. 2. Elevating the client's legs by raising the bed's knee support. 3. Assisting the client to dangle the legs over the side of the bed. 4. Maintaining the client on bed rest until the bandages are removed.

1. Encouraging the client to ambulate in the hallway.

The most essential nursing intervention for a client with a nephrostomy tube is to: 1. Ensure free drainage of urine 2. Milk the tube every two hours 3. Instill 2 mL of normal saline every eight hours 4. Keep an accurate record of intake and output

1. Ensure free drainage of urine

The primary responsibility of a nurse when caring for a client with a chest tube attached to a three-chamber underwater-seal drainage system is to: 1. Ensure maintenance of the closed system 2. Maintain mechanical suction to the system 3. Encourage the client to deep breathe and cough 4. Keep the client in the dorsal recumbent position

1. Ensure maintenance of the closed system

A client has a kidney transplant. The nurse should monitor for which signs associated with rejection of the transplant? (Select all that apply.) 1. Fever 2. Oliguria 3. Jaundice 4. Moon face 5. Weight gain

1. Fever 2. Oliguria 5. Weight gain

A nurse is assessing a female client with Cushing syndrome. Which clinical findings can the nurse expect to identify? (Select all that apply.) 1. Hirsutism 2. Menorrhagia 3. Buffalo hump 4. Dependent edema 5. Migraine headaches

1. Hirsutism 3. Buffalo hump

A client is admitted to a medical unit with the diagnosis of acute kidney failure. The nurse reviews the client's laboratory data, performs a physical assessment, and obtains the client's vital signs. What should the nurse conclude the client is most likely experiencing? 1. Hyperkalemia 2. Hyponatremia 3. Hypouricemia 4. Hypercalcemia

1. Hyperkalemia

A nurse is concerned that a client with a diagnosis of cirrhosis of the liver may experience the complication of hepatic coma. For which clinical indicator should the nurse assess this client? 1. Icterus 2. Urticaria 3. Uremic frost 4. Hemangioma

1. Icterus **Bile deposits will impart a yellowish tinge (jaundice or icterus) to the skin, often first observed in the sclerae.

After the removal of a cast from a fractured arm, an 82-year-old client is to receive physical therapy. In an older adult, mild exercise is expected to cause respirations to: 1. Increase to 24 breaths per minute 2. Become progressively more difficult 3. Decrease in rate as their depth increases 4. Become irregular but remain within normal rates

1. Increase to 24 breaths per minute

A nurse is caring for a client after a thyroidectomy. For which signs of thyroid storm should the client be monitored? (Select all that apply.) 1. Increased heart rate 2. Increased temperature 3. Decreased respirations 4. Increased pulse deficit 5. Decreased blood pressure

1. Increased heart rate 2. Increased temperature

Which is an independent nursing action that should be included in the plan of care for a client after an episode of ketoacidosis? 1. Monitoring for signs of hypoglycemia as a result of treatment 2. Withholding glucose in any form until the situation is corrected 3. Giving fruit juices, broth, and milk as soon as the client is able to take fluids orally 4. Regulating insulin dosage according to the amount of ketones found in the client's urin

1. Monitoring for signs of hypoglycemia as a result of treatment

A client is diagnosed with Crohn's disease, and parenteral vitamins are prescribed. The client asks why the vitamins have to be given intravenously (IV) rather than by mouth. What rationales for this route should the nurse include in a response to the question? (Select all that apply.) 1. More rapid action results. 2. They are ineffective orally. 3. They decrease colon irritability. 4. Intestinal absorption may be inadequate. 5. Allergic responses are less likely to occur.

1. More rapid action results. 2. They are ineffective orally. 4. Intestinal absorption may be inadequate.

A nurse is caring for a client who is admitted to the hospital with a diagnosis of unstable angina. Sublingual nitroglycerin has been prescribed. What client response indicates that nitroglycerin is effective? 1. Pain subsides as a result of arteriole and venous dilation 2. Pulse rate increases because the cardiac output has been stimulated 3. Sublingual area tingles because sensory nerves are being triggered 4. Capacity for activity improves as a response to increased collateral circulation

1. Pain subsides as a result of arteriole and venous dilation **Nitroglycerin causes vasodilation, increasing the flow of blood and oxygen to the myocardium and reducing anginal pain.

A client has a hypoglycemic reaction to insulin. Which client responses should the nurse document as clinical manifestations of hypoglycemia? (Select all that apply.) 1. Pallor 2. Tremors 3 . Glycosuria 4 . Acetonuria 5. Diaphoresis

1. Pallor 2. Tremors 5. Diaphoresis

A nurse is caring for a client who is receiving total parenteral nutrition (TPN). The nurse should monitor the client for which complications? (Select all that apply.) 1. Phlebitis 2. Infection 3. Hepatitis 4. Anorexia 5. Dysrhythmias

1. Phlebitis 2. Infection

A client is admitted to the hospital with a history of cancer of the liver and jaundice. In relation to the jaundice, the nurse expects the client to report the presence of: 1. Pruritus 2. Diarrhea 3. Blurred vision 4. Bleeding gums

1. Pruritus

The nurse is providing dietary teaching to a 40-year-old client who is receiving hemodialysis. The nurse should encourage the client to include what in the client's dietary plan? 1. Rice 2. Potatoes 3. Canned salmon 4. Barbecued beef

1. Rice **Foods high in carbohydrates and low in protein, sodium, and potassium are encouraged for these clients.

A woman comes to the emergency department reporting signs and symptoms determined by the health care provider to be caused by a myocardial infarction. The nurse obtains a health history. Which reported symptoms does the nurse determine are specifically related to a myocardial infarction in women? (Select all that apply.) 1. Severe fatigue 2. Sense of unease 3. Choking sensation 4. Chest pain relieved by rest 5. Pain radiating down the left arm

1. Severe fatigue 2. Sense of unease

A client with cholecystitis is placed on a low fat, high protein diet. What nutrient should the nurse teach the client is included with this diet? 1. Skim milk 2. Boiled beef 3. Poached eggs 4. Steamed broccoli

1. Skim milk

The nurse is counseling a client with type 1 diabetes about the client's favorite foods that are lowest in carbohydrates (CHO). The nurse determines that this concept is understood when the client chooses eight ounces of: 1. Skim milk 2. Apple juice 3. Nonfat yogurt 4. Fresh orange juice

1. Skim milk *Skim milk contains about 12 grams of CHO per cup. There are about 30 grams CHO in 1 cup of apple juice. There are about 16 grams CHO in 1 cup of nonfat yogurt. There are about 25 grams CHO in 1 cup of orange juice

A nurse is assessing an older adult client. Which clinical findings are expected responses to the aging process? (Select all that apply.) 1. Slowed neurological responses 2. Lowered intelligence quotient 3. Long-term memory impairment 4. Forgetfulness about recent events 5. Reduced ability to maintain an erection

1. Slowed neurological responses 4. Forgetfulness about recent events 5. Reduced ability to maintain an erection

The nurse is caring for a client two days after the client was admitted with burn injury. When performing the respiratory assessment, the nurse observes for sputum that is: 1. Sooty 2. Frothy 3. Yellow 4. Tenacious

1. Sooty

A client admitted to the emergency department has ketones in the blood and urine. Which situation associated with this physiological finding should be the nurse's focus when collecting additional data about this client? 1. Starvation 2. Alcoholism 3. Bone healing 4. Positive nitrogen balance

1. Starvation

A nurse is monitoring for clinical manifestations of infection in a client with a diagnosis of Addison disease. Which body mechanism related to infectious processes does the nurse conclude is impaired as a result of this disease? 1. Stress response 2. Electrolyte balance 3. Metabolic processes 4. Respiratory function

1. Stress response

A person is brought to the emergency department after prolonged exposure to cold weather. What clinical manifestations of hypothermia does the nurse expect? (Select all that apply.) 1. Stupor 2. Erythema 3. Increased anxiety 4. Rapid respirations 5. Paresthesia in affected body parts

1. Stupor 5. Paresthesia in affected body parts

A nurse is preparing a teaching plan for a client with syphilis. The nurse includes that syphilis is not considered contagious in the: 1. Tertiary stage 2. Primary stage 3. Secondary stage 4. Incubation stage

1. Tertiary stage

After a nephrectomy a client arrives in the post-anesthesia care unit in the supine position. Which action should be employed by the nurse to assess the client for signs of hemorrhage? 1. Turn the client to observe the dressings. 2. Press the client's nail beds to assess capillary refill. 3. Observe the client for hemoptysis when suctioning. 4. Monitor the client's blood pressure for a rapid increase.

1. Turn the client to observe the dressings.

A client is admitted with a diagnosis of a ruptured spleen. The client's blood pressure is 100/60. The nurse should assess the client for an early sign of decreased arterial pressure which is: 1. Weak radial pulses 2. Warm, flushed skin 3. Lethargy with confusion 4. Increased pulse pressure

1. Weak radial pulses **Hypovolemia occurs with decreased cardiac output and the resulting decreased arterial pressure is reflected in weak, thready peripheral pulses.

A nurse is providing dietary instruction to a client with cardiovascular disease. Which dietary selection by the client indicates the need for further instruction? 1. Whole milk with oatmeal 2. Garden salad with olive oil 3. Tuna fish with a small apple 4. Soluble fiber cereal with skim milk

1. Whole milk with oatmeal **Although oatmeal is a soluble fiber, whole milk is high in saturated fat and should be avoided.

A client with heart failure is digitalized and placed on a maintenance dose of digoxin (Lanoxin) 0.25 mg by mouth daily. What responses does the nurse expect the client to exhibit when a therapeutic effect of digoxin is achieved? 1.Diuresis and decreased pulse rate 2.Increased blood pressure and weight loss 3. Regular pulse rhythm and stable fluid balance 4.Corrected heart murmur and decreased pulse pressure

1.Diuresis and decreased pulse rate

The nurse is caring for a client who recently returned from another country who exhibits signs and symptoms suspicious of severe acute respiratory syndrome (SARS). Which clinical manifestations support this diagnosis? (Select all that apply.) 1.Dry cough 2. Chest pain 3. Hemoptysis 4. Shortness of breath 5 . Fever greater than 100.4° F

1.Dry cough 4. Shortness of breath 5 . Fever greater than 100.4° F

A nurse in the postanesthesia care unit identifies a progressive decrease in blood pressure in a client who had major abdominal surgery. What clinical finding supports the conclusion that the client is experiencing internal bleeding? 1.Oliguria 2. Bradypnea 3. Pulse deficit 4. High potassium levels

1.Oliguria

The nurse provides teaching about self-care management to a client who recently was diagnosed with emphysema. The nurse concludes that further teaching is needed when the client states:

2 "I will maintain complete bed rest.

A client has undergone surgery with general anesthesia. Within how many hours after surgery should the nurse notify the health care provider if the client does not void?

2 8 hours

A client is diagnosed with emphysema. For what long-term problem should the nurse monitor this client?

2 Carbon dioxide retention

A client has a decreased serum sodium level. The nurse should assess the client for which signs of hyponatremia? (Select all that apply.)

2 Confusion 5 Muscle weakness

A nurse is caring for a client with chronic kidney failure. Which clinical findings should the nurse expect when assessing this client? (Select all that apply.)

2 Lethargy 4 Muscle twitching

A client has rotator cuff surgery. What should be included when the nurse performs a neurovascular assessment of the affected extremity immediately after surgery? (Select all that apply.)

2 Skin color 4 Movement of the hand 5 Sensations in the extremity

A client who has been on hemodialysis for several weeks asks the nurse what substances are being removed by the dialysis. The nurse informs the client that one of the substances passing through the membrane is:

2 Sodium

After resection of a lower lobe of the lung, a client has excessive respiratory secretions. Which independent nursing action should the nurse implement?

2 Turning and positioning

A nurse is caring for a client who had a pneumonectomy. What is the priority nursing assessment?

2 Ventilatory exchange

A nurse is assessing clients who are to be given the smallpox vaccination. Which client should the nurse remove from the immunization line for medical counseling? 1. 20-year-old healthy woman 2. 45-year-old woman with breast cancer 3. 50-year-old man with diabetes mellitus 4. 75-year-old man who has Parkinson disease

2. 45-year-old woman with breast cancer

A client with varicose veins is scheduled for sclerotherapy. What clinical finding does the nurse expect to identify when assessing the lower extremities of this client? 1. Pallor 2. Ankle edema 3. Yellowed toenails 4. Diminished pedal pulses

2. Ankle edema **Ankle edema results from venous pooling with increased hydrostatic pressure; fluid moves from intravascular to interstitial spaces.

After thoracic surgery a client has a chest tube connected to a water-seal drainage system that is attached to suction. When excessive bubbling is observed in the water-seal chamber, the nurse should: 1. Strip the chest tube catheter 2. Check the system for air leaks 3. Decrease the amount of suction pressure 4. Recognize that the system is functioning correctly

2. Check the system for air leaks

A nurse concludes that the anemia that accompanies chronic kidney disease should be treated because it contributes to: 1. Uremic frost 2. Chronic fatigue 3. Tubular necrosis 4. Dependent edema

2. Chronic fatigue

A nurse is caring for a client with a diagnosis of cholecystitis. For which clinical manifestation of obstructive jaundice should the nurse assess the client? 1. Gray-colored skin 2. Clay-colored stools 3. Pale-colored conjunctiva 4. Light amber-colored urine

2. Clay-colored stools

A client is admitted to the hospital with a diagnosis of acute salmonellosis. What does the nurse expect the health care provider to prescribe? 1. Cathartics 2. Electrolytes 3. Antidiarrheals 4. Antispasmodics

2. Electrolytes

A nurse is caring for a client who is being treated with continuous ambulatory peritoneal dialysis (CAPD) for chronic glomerulonephritis. What dietary need should the nurse discuss with the client? 1. Low-calorie foods 2. High-quality protein 3. Increased fluid intake 4. Foods rich in potassium

2. High-quality protein

A nurse is caring for a client with a diagnosis of Cushing syndrome. What is the most common cause of Cushing syndrome that the nurse should consider before assessing this client for physiological responses? 1. Pituitary hypoplasia 2. Hyperplasia of the adrenal cortex 3. Deprivation of adrenocortical hormones 4. Insufficient adrenocorticotropic hormone (ACTH) production

2. Hyperplasia of the adrenal cortex

A nurse is assessing a client who is scheduled for a liver biopsy. What assessment finding needs to be reported immediately because it warrants a postponement of the liver biopsy? 1. Mental confusion 2. International normalized ratio (INR) of 4.0 3. Presence of an infectious disease 4. Foods high in vitamin K eaten before the biopsy

2. International normalized ratio (INR) of 4.0

A client with a history of hypertension develops dyspnea on exertion. What does the nurse conclude is the most likely cause of the client's dyspnea? 1. Cor pulmonale 2. Left heart failure 3. Bronchial spasms 4. Right ventricular failure

2. Left heart failure

When assessing the breath sounds of a client with chronic obstructive pulmonary disease (COPD), the nurse hears coarse rhonchi. They are described best as: 1.Snorting sounds during the inspiratory phase 2. Moist rumbling sounds that clear after coughing 3. Musical sounds more pronounced during expiration 4. Crackling inspiratory sounds unchanged with coughing

2. Moist rumbling sounds that clear after coughing

A client follows a vegetarian diet and must compensate for the lack of vitamin B12 found in food of animal origin. Which food should the nurse encourage the client to consume each day? 1. One orange 2. One glass of soy milk 3. Two handfuls of nuts 4. Two servings of green vegetables

2. One glass of soy milk

A nurse is caring for a client with a history of chronic obstructive pulmonary disease (COPD) who develops a pneumothorax and has a chest tube inserted. What is the primary purpose of the chest tube? 1. Lessens the client's chest discomfort 2. Restores negative pressure in the pleural space 3. Drains accumulated fluid from the pleural cavity 4. Prevents subcutaneous emphysema in the chest wall

2. Restores negative pressure in the pleural space

When two nurses are getting an older adult out of bed, the client reports feeling lightheaded. The nurse identifies that the client's pulse is stable and the client's color has not changed. What should the nurses assist the client to do? 1. Slide slowly to the floor to prevent a fall and injury. 2. Sit on the edge of the bed while they hold the client upright. 3. Bend forward because this will increase blood flow to the brain. 4. Lie down quickly so the legs can be raised above the heart level.

2. Sit on the edge of the bed while they hold the client upright.

A client with a history of a pulmonary embolus is to receive 3 mg of warfarin (Coumadin) daily. The client has blood drawn twice weekly to ascertain that the international normalized ratio (INR) stays within a therapeutic range. The nurse provides dietary teaching. Which food selected by the client indicates that further teaching is necessary? 1. Poached eggs 2. Spinach salad 3. Sweet potatoes 4. Cheese sandwich

2. Spinach salad **Dark green, leafy vegetables are high in vitamin K. Influencing the level of vitamin K alters the activity of warfarin because vitamin K acts as a catalyst in the liver for the production of blood-clotting factors and prothrombin.

During a blood transfusion a client develops chills and a headache. What is the priority nursing action? 1. Cover the client. 2. Stop the transfusion at once. 3. Decrease the rate of the blood infusion. 4. Notify the health care provider immediately

2. Stop the transfusion at once.

When assessing a client during peritoneal dialysis, a nurse observes that drainage of the dialysate from the peritoneal cavity has ceased before the required volume has returned. What should the nurse instruct the client to do? 1. Drink a glass of water 2. Turn from side to side 3. Deep breathe and cough 4. Rotate the catheter periodically

2. Turn from side to side **Turning from side to side will change the position of the catheter, thereby freeing the drainage holes of the tubing, which may be obstructed.

A health care provider prescribes 2 L of intravenous (IV) fluid to be administered every 12 hours to a client who sustained a burn injury. The drop factor of the tubing is 10 gtts/mL. The nurse should set the flow rate at how many drops per minute? Record your answer using a whole number. __________ gtts/min Multiply the amount to be infused (2000 mL) by the drop factor (10), and divide the result by the amount of time in minutes (12 hours × 60 min).

28

A client has chronic obstructive pulmonary disease (COPD). To decrease the risk of CO2 intoxication (CO2 narcosis), the nurse should:

3 Administer oxygen at a low concentration to maintain respiratory drive

A client with uremic syndrome has the potential to develop many complications. Which complication should the nurse anticipate?

3 Flapping hand tremors

A client with emphysema is admitted to the hospital with pneumonia. On the third hospital day, the client complains of a sharp pain on the right side of the chest. The nurse suspects a pneumothorax. What breath sound is most likely to be present when the nurse assesses the client's right side? 1. Crackling 2. Wheezing 3. Decreased sounds 4. Adventitious sounds

3. Decreased sounds

A client with hepatic cirrhosis develops hepatic encephalopathy. Neomycin sulfate (Mycifradin) is prescribed. The nurse concludes that the purpose of neomycin is to: 1. Decrease intestinal edema 2. Reduce abdominal distention 3. Diminish the blood ammonia level 4. Limit development of systemic infections

3. Diminish the blood ammonia level

A nurse obtains a health history from a client with the diagnosis of renal calculi. The nurse concludes that the factor that most likely contributed to the calculi development is the client's: 1. High cholesterol diet 2. Excessive exercise program 3. Excess ingestion of antacids 4. Frequent consumption of alcohol

3. Excess ingestion of antacids

A nurse is collecting information about a client who has type 1 diabetes and is being admitted because of diabetic ketoacidotic coma. Which factors can predispose a client to this condition? (Select all that apply.) 1. Taking too much insulin 2. Getting too much exercise 3. Excessive emotional stress 4. Running a fever with the flu 5. Eating fewer calories than prescribed

3. Excessive emotional stress 4. Running a fever with the flu

A nurse is caring for a client receiving hemodialysis for chronic kidney disease. The nurse should monitor the client for what complication? 1. Peritonitis 2. Renal calculi 3. Hepatitis B 4. Bladder infection

3. Hepatitis B **Hepatitis type B is transmitted by blood or blood products. The hemodialysis and routine transfusions needed for a client in end-stage renal failure constitute a high risk for exposure.

The nurse is reviewing the client's health history. With which diagnosis is a client most likely to exhibit hemoptysis? 1. Anemia 2. Pneumonia 3. Tuberculosis 4. Leukocytosi

3. Tuberculosis *Hemoptysis is expectoration of blood-stained sputum derived from the lungs, bronchi, or trachea; this is a clinical manifestation of tissue erosion caused by tuberculosis

A client comes to the emergency department because of minimal urinary output despite drinking adequate fluid. The client's blood pressure is 190/94. For what additional clinical manifestation associated with this data, should the nurse assess the client? 1. Thirst 2. Urinary retention 3. Weight gain 4. Urinary hesitancy

3. Weight gain

A client is suspected of having late-stage (tertiary) syphilis. When obtaining a health history, the nurse determines that the client statement that most supports this diagnosis is:

4 "I'm having trouble keeping my balance."

A client with chronic kidney disease is admitted to the hospital with severe infection and anemia. The client is depressed and irritable. The client's spouse asks the nurse about the anticipated plan of care. What is an appropriate nursing response?

4 "The intake of meat, eggs, and cheese will be restricted so the kidneys can clear the body of waste products."

A client is scheduled to receive general anesthesia during an upcoming surgery. The nurse provides education about common side effects of general anesthesia. The nurse concludes that the teaching has been effective when the client states, "Immediately after surgery I may experience:

4 A sore throat."

A client with acute kidney failure becomes confused and irritable. Which does the nurse determine is the most likely cause of this behavior?

4 An increased blood urea nitrogen level

A client is admitted with suspected atelectasis. Which clinical manifestation does the nurse expect to identify when assessing this client?

4 Diminished breath sounds

A nurse evaluates that a client understands the side effects of hydrochlorothiazide (HCTZ) therapy when the client states, "I should call my health care provider if I develop:

4 Generalized weakness."

A client with chronic obstructive pulmonary disease has increased hemoglobin and hematocrit levels. The nurse concludes that the altered blood levels are caused by:

4 Increased erythrocyte production as a result of chronic hypoxia

A client with chronic osteomyelitis in the leg is scheduled to have a debridement of the infected bone. What should the nurse include in the postoperative plan of care?

4 Inform the client that the leg may be immobilized in a cast or splint

A client arrives for a vaccination at an influenza prevention clinic. A nursing assessment identifies a current febrile illness with a cough. The nurse should:

4 Reschedule administration of the vaccine for the next month

A client with a head injury is admitted to the hospital. Which client response indicates increasing intracranial pressure?

4 Widening pulse pressure

A nurse is caring for an alert client who has diabetes and is receiving an 1800-calorie American Diabetic Association diet. The client's blood glucose level is 60 mg/dL. The health care provider's protocol calls for treatment of hypoglycemia with 15 g of a simple carbohydrate. The nurse should: 1. Provide 12 ounces of non-diet soda 2. Give 25 mL dextrose 50% by slow intravenous (IV) push 3. Have the client drink 8 ounces of fruit juice 4. Ask the client to ingest one tube of glucose gel

4. Ask the client to ingest one tube of glucose gel *One tube of glucose gel contains 15 g of carbohydrate and is the most appropriate intervention in this situation

A client is admitted to the emergency department with crushing chest pain. A diagnosis of acute coronary syndrome is suspected. The nurse expects that the client's initial treatment will include which medication? 1. Gabapentin (Neurontin) 2. Midazolam HCI (Versed) 3. Alprazolam (Xanax) 4. Aspirin (ASA)

4. Aspirin (ASA)

A client with diabetes is given instructions about foot care. The nurse determines that the instructions are understood when the client states, "I will: 1. Cut my toenails before bathing." 2. Soak my feet daily for one hour." 3. Examine my feet using a mirror at least once a week." 4. Break in my new shoes over the course of several weeks."

4. Break in my new shoes over the course of several weeks."

A client is admitted to the hospital for medical treatment of bronchopneumonia. What test result should the nurse examine to help determine the effectiveness of the client's therapy? 1. Bronchoscopy 2. Pulse oximetry 3. Pulmonary function studies 4. Culture and sensitivity tests of sputum

4. Culture and sensitivity tests of sputum *The aim of therapy is to eliminate the causative agent, which is determined from culture and sensitivity tests of sputum.

Which clinical manifestation should a nurse expect a client with diabetes insipidus to exhibit? 1. Increased blood glucose 2. Decreased serum sodium 3. Increased specific gravity 4. Decreased urine osmolarity

4. Decreased urine osmolarity

A nurse assesses a newly admitted client with renal colic to determine the signs and symptoms that are present. The nurse assesses the client for which primary subjective symptom? 1. Uremia 2. Nausea 3. Voiding at night 4. Flank discomfort

4. Flank discomfort

A client is being discharged after an acute episode of hepatitis. The nurse expects the primary health care provider to prescribe which type of diet for this client? 1. Low calorie, high protein, low carbohydrate, low fat 2. High calorie, low protein, high carbohydrate, high fat 3. Low calorie, low protein, low carbohydrate, moderate fat 4. High calorie, high protein, high carbohydrate, moderate fat

4. High calorie, high protein, high carbohydrate, moderate fat

A nurse is caring for a client with a diagnosis of chronic kidney failure who has just been told by the health care provider that hemodialysis is necessary. Which clinical manifestation indicates the need for hemodialysis? 1. Ascites 2. Acidosis 3. Hypertension 4. Hyperkalemia

4. Hyperkalemia **Protein breakdown liberates cellular potassium ions, leading to hyperkalemia, which can cause a cardiac dysrhythmia and standstill.

A client with a tentative diagnosis of Cushing syndrome has an increased cortisol level. For what response should the nurse assess this client? 1. Hypovolemia 2. Hyperkalemia 3. Hypoglycemia 4. Hypernatremia

4. Hypernatremia

A nurse is caring for a client who was admitted to the hospital with a diagnosis of Addison disease. The nurse should assess the client for what signs related to this disorder? 1. Diarrhea and pyrexia 2. Edema and hypertension 3. Moon face and hirsutism 4. Hypoglycemia and hypotension

4. Hypoglycemia and hypotension

A nurse is developing a discharge plan for a client who was hospitalized with severe cirrhosis of the liver. The plan should include the: 1. Need for a high protein diet 2. Use of a sedative for relaxation 3. Need to increase fluids 4. Importance of reporting personality changes to the health care provider

4. Importance of reporting personality changes to the health care provider

A nurse is preparing to insert a nasogastric tube. During insertion, which response indicates that the client is experiencing difficulty? 1. Gagging 2. Discomfort 3. Flushed face 4. Inability to speak

4. Inability to speak

A nurse is completing the admission assessment of a client with peripheral arterial disease. What assessments are consistent with this diagnosis? (Select all that apply.)

Absence of hair on the toes Reports of pain associated with exercising

A client with a tentative diagnosis of pernicious anemia is scheduled for a Schilling test. Which body process associated with vitamin B12 is assessed with the Shilling test?

Absorption

A nurse is caring for a client admitted to the hospital for diabetic ketoacidosis. Which clinical findings related to this event should the nurse document in the client's clinical record? (Select all that apply.)

Acetone breath Decreased arterial carbon dioxide level

In the postanesthesia care unit after a below-the-knee amputation, a client begins crying after feeling for the affected lower leg. How should the nurse respond?

Allow the client to ventilate feelings of loss.

The nurse is assessing a client with hyperthyroidism. For which signs and symptoms should the nurse assess the client? (Select all that apply.)

Amenorrhea Flushed appearance Short attention span

The cardiac monitor reveals a series of premature ventricular complexes (PVCs). The nurse anticipates that the client will be receiving a prescription for:

Amiodarone (Cordarone)

A nurse is caring for a client with cirrhosis of the liver. Which laboratory test should the nurse monitor that, when abnormal, might identify a client who may benefit from neomycin enemas?

Ammonia level

A client has a diagnosis of hemorrhoids. Which signs and symptoms does the nurse expect the client to report? (Select all that apply.)

Anal itching Blood in stool Rectal pressure Pain when defecating

A client who has been taking digoxin (Lanoxin) for 20 years is hospitalized. The client exhibits signs of dehydration and laboratory results identify the presence of hypokalemia. The nurse should monitor the client for which clinical finding indicating digoxin toxicity?

Blurred vision

A nurse provides dietary instruction to a client who has iron deficiency anemia. Which food choices by the client does the nurse consider most desirable? (Select all that apply.)

Boiled spinach Sweet potatoes

For which client response should the nurse monitor when assessing for complications of hyperparathyroidism?

Bone pain

A nurse assesses a client with the diagnosis of an intestinal obstruction in the descending colon. When auscultating the midabdomen, the nurse expects to hear:

Borborygmi

A nurse is caring for a postoperative client who had a gastrectomy. What early client response indicates that peristalsis has returned?

Borborygmi are auscultated

Which patients are at risk of developing health care-associated infections (HAIs)? Select all that apply. A. A patient with an arm fracture B. A patient with a very high fever C. A patient with laryngeal cancer D. A patient with diabetes mellitus E. A patient with an indwelling urinary catheter

C. A patient with laryngeal cancer D. A patient with diabetes mellitus E. A patient with an indwelling urinary catheter

After a surgical thyroidectomy a client exhibits carpopedal spasm and some tremors. The client complains of tingling in the fingers and around the mouth. What medication should the nurse expect the primary health care provider to prescribe after being notified of the client's adaptations?

Calcium gluconate

A client with arterial insufficiency of both lower extremities is visited by the home health care nurse. An essential nursing intervention is to teach the client to:

Check pulses in the legs regularly

A nurse is caring for a client who just returned from the postanesthesia care unit after having a thyroidectomy. Which action has priority during the first 24 hours after surgery when the nurse is concerned about thyroid storm?

Checking vital signs every two hours after they stabilize

A nurse asks a client with ischemic heart disease to identify the foods that are most important to restrict. The nurse determines that the client understands the dietary instructions when the client identifies the following foods. (Select all that apply.)

Chicken broth Enriched whole milk Red meats, such as beef Liver and other glandular organ meats

A nurse is caring for a client with a diagnosis of cholecystitis. For which clinical manifestation of obstructive jaundice should the nurse assess the client?

Clay-colored stools

A nurse is administering an enema to a client who is scheduled for gastrointestinal surgery. What should the nurse do when the client complains of abdominal cramps during the enema?

Close the lumen of the tubing and wait until the discomfort subsides.

A self-help group of clients with irritable bowel syndrome have invited a nurse to present a program on nutrition. Which substance should the nurse teach the clients to minimize in the diet to decrease gastrointestinal (GI) irritability?

Cola drinks

A nurse is teaching a group of clients with peripheral vascular disease about a smoking cessation program. Which physiological effect of nicotine should the nurse explain to the group?

Constriction of the peripheral vessels increases the force of flow.

A nurse identifies that clients with cancer often lose weight and may become cachectic. What common response do clients with cancer experience regardless of the site of the cancer that accounts for this weight loss?

CorrectTumor necrosis factor affects the satiety center

A client has a body mass index (BMI) of 35 and verbalizes the need to lose weight. The nurse encourages the client to lose weight safely by changing which dietary habits?

Decrease portion size and fat intake

When preparing a client for discharge after a thyroidectomy, the nurse teaches the signs of hypothyroidism. The nurse evaluates that the client understands the teaching when the client says, "I should call my health care provider if I develop:

Dry hair and an intolerance to cold."

A nurse is assessing a client with diabetic ketoacidosis. Which clinical manifestations should the nurse expect? (Select all that apply.)

Dry skin Abdominal pain Kussmaul respirations

A nurse is assessing a client with diabetic ketoacidosis. Which clinical manifestations should the nurse expect? (Select all that apply.

Dry skin Abdominal pain Kussmaul respirations

A nurse teaches a client who has had a thyroidectomy for thyroid cancer to observe for signs of surgically induced hypothyroidism. What should be included in the teaching plan? (Select all that apply.)

Dry skin Lethargy Sensitivity to cold

A client with a parotid tumor and enlarged lymph nodes in the neck is undergoing radiation therapy on an outpatient basis. For what physiological response to the radiation should the nurse assess the client during the return visit to the radiology department?

Dysphagia

When assessing a client with the diagnosis of left ventricular failure, the nurse expects to identify:

Dyspnea on exertion

What clinical indicators are the nurse most likely to identify when taking the admission history of a client with right ventricular failure? (Select all that apply.)

Edema 4 Dyspnea

During administration of a whole blood transfusion, the client begins to complain of shortness of breath. The nurse notes the presence of jugular venous distension, bibasilar crackles, and tachycardia. Prioritize the following nursing actions.

Elevate the head of the bed to 45 degrees Apply oxygen via nasal cannula Reduce the flow rate of the transfusion Administer furosemide (Lasix) per provider prescription Document findings in the client record

A nurse is evaluating the results of treatment with erythropoietin (Epogen). Which client response is considered significant?

Elevation in hematocrit level

A nurse is caring for a client with a history of hypertension and aphasia. A family member states that a complete occlusion of the branches of the middle cerebral artery resulted in the client's aphasia. What is a common cause of this type of occlusion?

Emboli associated with atrial fibrillation

A client with advanced bone cancer is experiencing cachexia. The nurse discusses the nutritional aspect of palliative care with the family. Why is it important to explain these nutritional interventions to the family?

Enhance the quality of the client's life

A client is admitted for malignant melanoma that was discovered during a routine eye examination. For which preferred treatment does the nurse expect the client to be scheduled?

Enucleation

A client is admitted for malignant melanoma that was discovered during a routine eye examination. For which preferred treatment does the nurse expect the client to be scheduled? 1 Radiation 2 Enucleation 3 Cryosurgery 4 Chemotherapy

Enucleation

A nurse is formulating a teaching plan for a client recently diagnosed with type 2 diabetes. What interventions should the nurse include that will decrease the risk of complications? (Select all that apply.)

Examining the feet daily Wearing well-fitting shoes Performing regular exercise

The nurse is providing teaching to a client who recently has been diagnosed with type 1 diabetes. The nurse reinforces the importance of monitoring for ketoacidosis. What are the signs and symptoms of ketoacidosis? (Select all that apply.)

Excessive thirst Fruity-scented breath Confusion

The nurse develops a teaching plan for a client with diabetes who has been diagnosed with lower extremity arterial disease (LEAD). The nurse includes measures to increase arterial blood flow to the extremities, including:

Exercises that promote muscular activity

A paracentesis is prescribed for a client recently admitted to a medical unit. The nurse recalls that the procedure is performed for what reasons? (Select all that apply.)

Extract peritoneal fluid Improve respiratory status Obtain peritoneal fluid for culture

A nurse is evaluating a client who has been receiving medical intervention for the diagnosis of Crohn's disease. Which expected outcome is most important for this client?

Gains a half pound per week

While a nurse is teaching a client with diabetes about food choices, the client states, "I do not like broccoli." The nurse suggests that a food that can be substituted for the broccoli is:

Green beans

Which actions contribute to the transmission of human immunodeficiency virus (HIV) infection from an infected to a healthy person? Select all that apply.

Having sexual intercourse Receiving blood transfusions

A nurse is caring for a client recently diagnosed with type 1 diabetes. For what signs and symptoms of an insulin reaction should the nurse assess this client? (Select all that apply.)

Headache Diaphoresis Nervousness

A nurse is caring for a client recently diagnosed with type 1 diabetes. For what signs and symptoms of an insulin reaction should the nurse assess this client? (Select all that apply.)

Headache Diaphoresis Nervousness

The nurse concludes that a client with type 1 diabetes is experiencing hypoglycemia. Which responses support this conclusion? (Select all that apply.)

Headache Tachycardia Cool clammy skin

What should the nurse identify as the primary cause of the pain experienced by a client with a coronary occlusion?

Heart muscle ischemia

A nurse is caring for several postoperative clients who had abdominal surgery. What independent nursing intervention can help prevent the development of thrombophlebitis?

Helping the client to perform in-bed exercises.

A nurse is caring for a client who is recovering from an acute episode of alcoholism. Which component of a therapeutic diet should the nurse encourage the client to consume?

High protein

A client has a paracentesis during which 1500 mL of fluid is removed. The nurse should monitor the client carefully for

Hypovolemic shock

A client newly diagnosed as having type 1 diabetes is taught to exercise on a regular basis primarily because exercise has been shown to:

Improve the cellular uptake of glucose

A client with jaundice associated with hepatitis expresses concern over the change in skin color. The nurse explains that this color change is a result of:

Inability of the liver to remove normal amounts of bilirubin from the blood

A nurse is caring for two clients; one has polycythemia and the other has prolonged anemia. What do these clients have in common?

Increased cardiac workload

A nurse is caring for a client who had a hypophysectomy. For which complication specific to this surgery should the nurse assess the client for early clinical manifestations?

Increased intracranial pressure

A client with diabetes asks how exercise will affect insulin and dietary needs. The nurse should respond, "Exercise:

Increases the need for carbohydrates and decreases the need for insulin."

Famotidine (Pepcid) is prescribed for a client with peptic ulcer disease. The client asks the nurse what this medication does. The nurse responds, "It:

Inhibits gastric acid secretion

A nurse concludes that a client has a hypoglycemic reaction to insulin. Which clinical findings support this conclusion? (Select all that apply.)

Irritability Heart palpitations

The nurse is teaching a diabetic client about the advantages of using an insulin pump. What information should the nurse include? (Select all that apply.)

It can improve A1c levels Clients can exercise without eating more carbohydrates

A client who recently immigrated to the United States has a chronic vitamin A deficiency. What information about vitamin A should the nurse consider when assessing this client for clinical indicators of this deficiency?

It is a necessary element of rhodopsin, which controls responses to light and dark environments.

Which clinical indicators can the nurse expect when assessing a client with Cushing syndrome? (Select all that apply.)

Lability of mood Slow wound healing

A client describes abdominal discomfort following ingestion of milk. Which enzyme, as a result of a genetic deficiency, should the nurse consider to be the cause of the client's discomfort?

Lactase

A client with a history of hypertension develops dyspnea on exertion. What does the nurse conclude is the most likely cause of the client's dyspnea? Incorrect1 Cor pulmonale

Left heart failure

A client receiving morphine is being monitored by the nurse for signs and symptoms of overdose. Which clinical findings support a conclusion of overdose? (Select all that apply.)

Lethargy Bradycardia Slow respirations

A nurse is obtaining a history and performing a physical assessment of a client who has cancer of the tongue. Which clinical findings should the nurse expect to identify? (Select all that apply.)

Leukoplakia Alterations in taste Enlarged cervical lymph nodes

A client with burns over 35% of the body complains of chilling. To promote client comfort, the nurse should:

Limit Room drafts

A client is taking furosemide (Lasix) and digoxin (Lanoxin) for heart failure. Why does the nurse advise the client to drink a glass of orange juice every day?

Maintaining potassium levels

A client is diagnosed with acute lymphoid leukemia and is receiving chemotherapy. The nurse should monitor what thrombocytopenic side effects of chemotherapy? (Select all that apply.)

Melena Purpura Hematuria

A client is admitted to the hospital with a diagnosis of cirrhosis of the liver. For which classic signs of hepatic coma should the nurse assess this client? (Select all that apply.)

Mental confusion Flapping hand tremors

A client with type 2 diabetes develops gout, and allopurinol (Zyloprim) is prescribed. The client is also taking metformin (Glucophage) and an over-the-counter nonsteroidal anti-inflammatory drug (NSAID). When teaching about the administration of allopurinol, what should the nurse instruct the client to do?

Monitor blood glucose levels more frequently.

A nurse is caring for a client who is experiencing an underproduction of thyroxine (T4). Which client response is associated with an underproduction of thyroxine?

Myxedema

A client has been experiencing extreme fatigue lately. The nurse suspects anemia and examines the client to identify additional clinical manifestations to support this inference. What locations on the client's body should the nurse assess? (Select all that apply.)

Nail beds Lining of eyelids Palms of hands

A client with diabetes mellitus complains of difficulty seeing. The nurse concludes that the causative factor is:

Neovascularization of the retina

A client has been diagnosed with hyperthyroidism. The nurse expects the client to exhibit which clinical manifestations? (Select all that apply.)

Nervousness Increased appetite

A client with gastroesophageal reflux disease (GERD) receives a prescription for an H2 receptor antagonist. Which medications are within the classification of an H2 receptor antagonist? (Select all that apply.)

Nizatidine (Axid) Ranitidine (Zantac) Famotidine (Pepcid)

Within the first 2½ hours after a radical neck dissection, 40 mL of medium red, bloody fluid is collected in the portable wound drainage system. What should the nurse do first?

Obtain the vital signs.

A client develops internal bleeding after an abdominal surgery. Which signs and symptoms of hemorrhage should the nurse expect the client to exhibit? (Select all that apply.)

Pallor Tachycardia

A nurse is caring for a client whose laboratory values indicate the presence of hyponatremia. For which risk factors should the nurse assess the client that most likely may have caused the hyponatremia? (Select all that apply.)

Profuse diaphoresis Rapid IV infusion of 5% dextrose in water

A nurse is caring for a client who is receiving total parenteral nutrition (TPN) after extensive colon surgery. The nurse concludes that the client understands teaching about the purpose of TPN when the client states, "TPN:

Provides total nutrition when gastrointestinal (GI) function is questionable.

A client with diabetes states, "I cannot eat big meals; I prefer to snack throughout the day." What information should the nurse include in a response to this client's statement?

Regulated food intake is basic to control.

How should the nurse make the bed of a client who is in the acute phase after a myocardial infarction?

Replace the top linen and only the necessary bottom linen.

A nurse is providing dietary teaching for a client with celiac disease. Which foods should the nurse teach the client to avoid when following a gluten-free diet? (Select all that apply.)

Rye Oats Wheat

A nurse is caring for a client with a diagnosis of necrotizing fasciitis. Which is the primary concern of the nurse when caring for this client?

Skin integrity

A client with a history of cardiac dysrhythmias is admitted to the hospital with dehydration. What does the nurse expect to be listed on the client's plan of care?

Small, frequent intake of juices, broth, or milk.

A nurse is caring for a client after a thyroidectomy. Because of concerns about potential nerve injury associated with this type of surgery, the nurse should assess for which functional ability?

Speaking

A client admitted to the emergency department has ketones in the blood and urine. Which situation associated with this physiological finding should be the nurse's focus when collecting additional data about this client?

Starvation

A client newly diagnosed with type 2 diabetes is receiving glyburide (Micronase) and asks the nurse how this drug works. The nurse explains that glyburide:

Stimulates the pancreas to produce insulin

A client with esophageal varices is admitted with hematemesis, and two units of packed red blood cells are prescribed. The client complains of flank pain halfway through the first unit of blood. The nurse's first action is to:

Stop the transfusion

A nurse is caring for a client who is cachectic. What information about the function of adipose tissue in fat metabolism is necessary to better address the needs of this client?

Stores triglycerides for energy reserves

A nurse evaluates that a client with diabetes understands the teaching about the treatment of hypoglycemia when the client says, "If I become hypoglycemic I initially should eat:

Sugar and a slice of bread.

A client is discharged with a prescription for sustained-release nitroglycerin. What should the nurse teach the client about sustained-release nitroglycerin?

Swallow the capsule whole

A client with varicose veins asks a nurse what is involved when ligation and stripping are performed rather than sclerotherapy. What should the nurse consider when planning a response in language the client will understand?

The dilated saphenous veins are removed.

Megadoses of vitamin A are taken by a client. Why should the nurse question this practice?

The liver has a great storage capacity for vitamin A, even to toxic amounts.

An older adult is hospitalized for weight loss and dehydration because of nutritional deficits. What should the nurse consider when caring for this client?

The nutritional needs of an older adult are unchanged except for a decreased need for calories.

A client has surgery to replace a prolapsed mitral valve. What should the nurse teach the client?

The possible need for prophylactic antibiotic therapy before dental work

A client who is scheduled to have surgery to remove an aldosterone-secreting adenoma asks the nurse what will happen if surgery is not performed. On what information should the nurse base a response?

The tumor must be removed to prevent heart and kidney damage.

A nurse plans to monitor for signs of autonomic dysreflexia in a client who sustained a spinal cord injury at the T2 level. This is necessary because:

There is damage above the sixth thoracic vertebra.

A client is cautioned to avoid vitamin D toxicity while increasing protein intake. Which nutrient selected by the client indicates to the nurse that the dietary teaching is understood?

Tofu

A client with gastric ulcer disease asks the nurse why the health care provider has prescribed metronidazole (Flagyl). The nurse explains, "Antibiotics are prescribed to:

Treat Helicobacter pylori infection."

A client is admitted to the hospital with the diagnosis of myocardial infarction. The nurse should monitor this client for which signs and symptoms associated with heart failure? (Select all that apply.)

Unusual fatigue Dependent edema Nocturnal dyspnea

After a bilateral inguinal hernia repair (herniorrhaphy) the nurse should assess the client for the development of:

Urinary retention

A client is taught how to recognize indications of a hypoglycemic reaction. Which signs and symptoms identified by the client indicate to the nurse that the teaching was effective? (Select all that apply.)

Weakness Nervousness Increased perspiration

A nurse is caring for a client after a thyroidectomy. For which signs of thyroid storm should the client be monitored? (Select all that apply.)

heart rate Increased temperature

What clinical indicator is the nurse most likely to identify when completing a history and physical assessment of a client with complete heart block?

syncope

On the third postoperative day after a subtotal gastrectomy, a client reports having severe abdominal pain. The nurse palpates the client's abdomen and determines rigidity. What should be the nurse's first action?

Obtain the client's vital signs.

During a routine examination, an enlarged thyroid gland is discovered in a client, and hyperthyroidism is suspected. What clinical findings should the nurse expect to identify when completing a nursing admission history and physical for this client? (Select all that apply.)

Palpitations Tachycardia

A client is admitted to the hospital with severe burns. What client response should the nurse anticipate when caring for the client during the acute phase of burn recovery?

Stable vital signs

A nurse is assessing a client with a diagnosis of psoriasis. Which clinical findings should the nurse expect to observe? (Select all that apply.) 1 Scaly lesions 2 Pruritic lesions 3 Reddened papules 4 Multiple petechiae 5 Erythematous maculesg

Scaly lesions Pruritic lesions Reddened papules

A nurse is caring for a client with varicose veins. Which clinical manifestations should the nurse expect with this diagnosis? (Select all that apply.)

Presence of ankle edema Increased muscle fatigue Report of nocturnal leg cramps

A client is admitted to the hospital with a long history of hypertension. The nurse should assess the client for which complication?

Kidney failure

A nurse is assisting a health care provider to perform a sigmoidoscopy. In which position should the nurse place the client for this procedure?

Knee-to-chest

A client arrives at the outpatient clinic with a painful leg ulcer, and the nurse performs a physical assessment. Which clinical findings in the lower extremity support a diagnosis of an arterial ulcer? (Select all that apply.)

Lack of hair Thickened toenails Pain at the ulcer site Diminished pedal pulse

Potassium supplements are prescribed for a client receiving diuretic therapy. What client statement indicates that the teaching about potassium supplements is understood?

"I will report any abdominal distress."

Which statement by a client who is scheduled for bariatric surgery indicates to the nurse that further preoperative teaching is necessary?

"I'm going to have a figure like a model in about a year."

Which instructions should the nurse include in the teaching plan for a client with hyperlipidemia who is being discharged with a prescription for cholestyramine (Questran)?

"Increase your intake of fiber and fluid.

A client develops a non-healing ulcer of a lower extremity and complains of leg cramps after walking short distances. The client asks the nurse what causes these leg pains. The nurse's best response is:

"Pain occurs in the legs while walking because there is a lack of oxygen to the muscles."

A client with diabetes asks the nurse whether the new forearm stick glucose monitor gives the same results as a fingerstick. What is the nurse's best response to this question?

"There is no difference between readings."

A health care provider prescribes oral loperamide (Maalox) and intravenous ranitidine (Zantac) for a client with burns and crushing injuries caused by an accident. The client asks how these medications work. The nurse's best response is:

"They limit acidity in the gastrointestinal tract."

A client who was admitted with severe abdominal pain and vomiting states, "I know I am very sick. Do you think I have cancer?" What is the best response by the nurse?

"What are your feelings about the diagnosis of cancer?"

A client who had abdominal surgery asks the nurse about when the client can return to work after discharge. The most appropriate response by the nurse is:

"What type of work did you have in mind?"

A client is admitted to the hospital with ascites. The client reports drinking a quart of vodka mixed in orange juice every day for the past three months. To assess the potential for withdrawal symptoms, which question would be appropriate for the nurse to ask the client?

"When was your last drink of vodka?

A client who was diagnosed recently with type 1 diabetes states, "I feel bad. I don't think I even want to go home. My spouse doesn't care about my diabetes." What is the most appropriate nursing response?

"You are unhappy. Have you tried to talk with your spouse?"

A client newly diagnosed with rheumatoid arthritis is admitted to the hospital with bilateral painful knee and wrist joints. The nurse identifies impaired physical mobility related to painful, swollen joints. What should the nurse teach the client to do during the acute phase of the disease?

1 Avoid exercises to the involved joints

The health care provider prescribes peak and trough levels of an antibiotic for a client who is receiving the medication intravenous piggyback (IVPB). For peak levels the nurse should have the laboratory obtain a blood sample from the client:

1 Between 30 and 60 minutes after the IVPB

The nurse reviews the medical records of four male clients and concludes that the client that is at highest risk of developing prostate cancer is the:

1 Black 55-year-old

The nurse is caring for a client who recently returned from another country who exhibits signs and symptoms suspicious of severe acute respiratory syndrome (SARS). Which clinical manifestations support this diagnosis? (Select all that apply.)

1 Dry cough 4 Shortness of breath 5 Fever greater than 100.4° F

A client with scleroderma reports having difficulty chewing and swallowing. What should the nurse recommend to safely facilitate eating?

1 Eat a mechanical soft diet

What instructions should the nurse provide to a client after a long leg cast is removed?

1 Elevate the extremity when sitting.

The most essential nursing intervention for a client with a nephrostomy tube is to:

1 Ensure free drainage of urine

A nurse is caring for a client who had a kidney transplant. What sign indicates that the client may be rejecting the transplanted kidney?

1 Fever

During a client's routine physical examination, a chest x-ray film reveals a lesion in the right upper lobe. What information in the client's history supports the health care provider's diagnosis of pulmonary tuberculosis? (Select all that apply.)

1 Fever 3 Night sweats 6 Blood-tinged sputum

A nurse is concerned about the public health implications of gonorrhea diagnosed in a 16-year-old adolescent. Which should be of most concern to the nurse?

1 Finding the client's contacts

A client arrives on the nursing unit unconscious and exhibiting decerebrate posturing. When assessing the client, the nurse expects to observe:

1 Hyperextension of both the upper and lower extremities

The nurse considers that sensory restriction in a client who is blind can:

1 Increase the use of daydreaming and fantasy

A client with a history of benign prostatic hypertrophy asks whether cranberry juice prevents bladder infections. The nurse replies that cranberry juice may be helpful because it:

1 Increases acidity of the urine

After emergency surgery, the nurse teaches a client how to use an incentive spirometer. What client behavior indicates to the nurse that the spirometer is being used correctly?

1 Inhales deeply through the mouthpiece, relaxes, and then exhales

What clinical manifestations does a nurse expect a client with systemic lupus erythematosus (SLE) most likely to exhibit? (Select all that apply.)

1 Joint pain 2 Facial rash 3 Pericarditis

Carbidopa-levodopa (Sinemet) is prescribed for a client with Parkinson disease. The nurse assesses for which adverse responses that are associated with this medication? (Select all that apply.)

1 Nausea 5 Emotional changes

An obese client who is mildly hypertensive is hospitalized with a diagnosis of ureteral colic and hematuria. What is the immediate focus of nursing care for this client?

1 Pain

A client with terminal cancer signs a do-not-resuscitate (DNR) order upon admission to the hospital. When the client goes into respiratory arrest a week later, the client is not resuscitated. Which factor does the nurse determine is most relevant to the legal aspects of a DNR order?

1 Policies of the agency establish the status of DNR orders

A nurse is teaching the importance of annual physical examinations to an adult health and wellness class. The nurse reinforces that it is important for men who are middle-aged and older to have what laboratory test annually?

1 Prostate-specific antigen (PSA)

A client is admitted with the diagnosis of tetanus. For which clinical indicators should the nurse assess the client? (Select all that apply.)

1 Restlessness 2 Muscular rigidity 3 Respiratory tract spasms 4 Spastic voluntary muscle contractions

A client with a history of tuberculosis reports difficulty hearing. Which medication should the nurse consider is related to this response?

1 Streptomycin

A client who is a pipe smoker is diagnosed with cancer of the tongue. A hemiglossectomy and right radical neck dissection are performed. To ensure airway patency during the first hours after surgery, the nurse should:

1 Suction as needed

A nurse is preparing a teaching plan for a client with syphilis. The nurse includes that syphilis is not considered contagious in the:

1 Tertiary stage

A client is hospitalized with a diagnosis of emphysema. The nurse provides teaching and should begin with which aspect of care?

1 The disease process and breathing exercises

A client with a long history of asthma is scheduled for surgery. What information should be included in preoperative teaching?

1 There is an increased risk of respiratory tract infections.

A client with tuberculosis is to begin Rifater (combination of isoniazid [INH], rifampin [RIF], and pyrazinamide [PZA]), and streptomycin sulfate (streptomycin) therapy. The client says, "I've never had to take so much medication for an infection before." The nurse should explain:

1 This type of organism is difficult to destroy."

After a gastroscopy, the nurse assesses the client for the return of the gag reflex by:

1 Touching the pharynx with a tongue depressor

After a nephrectomy a client arrives in the post-anesthesia care unit in the supine position. Which action should be employed by the nurse to assess the client for signs of hemorrhage?

1 Turn the client to observe the dressings.

A client with chronic obstructive pulmonary disease (COPD) reports chest congestion, especially upon wakening in the morning. The nurse should suggest that the client:

1 Use a humidifier in the bedroom

A client is receiving phenytoin (Dilantin) for a seizure disorder and heparin for a deep vein thrombosis. Warfarin (Coumadin) is added in preparation for discontinuing the heparin. Why must the nurse observe the client closely during the initial days of treatment with warfarin?

1 Warfarin affects the metabolism of phenytoin.

Which statement regarding treatment with interferon indicates that the client understands the nurse's teaching? 1. "I will drink two to three quarts of fluid a day." 2. "Any reconstituted solution must be discarded in one week." 3. "I can continue driving my car as long as I have the stamina." 4. "While taking this medicine I should be able to continue my usual activity."

1. "I will drink two to three quarts of fluid a day."

A client with diabetes asks the nurse whether the new forearm stick glucose monitor gives the same results as a fingerstick. What is the nurse's best response to this question? 1. "There is no difference between readings." 2. "These types of monitors are meant for children." 3. "Readings are on a different scale for each monitor." 4. "Faster readings can be obtained from a fingerstick."

1. "There is no difference between readings."

During discharge teaching, a client with an ileal conduit asks how frequently the urine pouch should be emptied. The best reply by the nurse is: 1. "To prevent leakage and pulling of the pouch from the skin, it should be emptied every few hours." 2. "To prevent skin irritation, it should be emptied every hour if any urine has collected in the bag." 3. "To reduce the risk of infection, the system should be opened as little as possible; two times a day is adequate." 4. "To reduce the cost of drainage pouches, it should be emptied once the system is switched to a bedside collection bag."

1. "To prevent leakage and pulling of the pouch from the skin, it should be emptied every few hours."

A client in thyroid storm tells the nurse, "I know I'm going to die. I'm very sick." What is the nurse's best response? 1. "You must feel very sick and frightened." 2. "Tell me why you feel you are going to die." 3. "I can understand how you feel, although people do not die from this problem." 4. "If you would like, I will call your family and tell them to come to the hospital."

1. "You must feel very sick and frightened."

A client who was admitted to the hospital with metastatic cancer has a temperature of 100.4° F, a distended abdomen, and abdominal pain. The client asks the nurse, "Do you think that I'm going to have surgery?" How should the nurse respond? 1. "You seem concerned about having surgery." 2. "Some people with your problem do have surgery." 3. "I'll find out for you. Your record will show if surgery is scheduled." 4. "I don't know about any surgery. You'll have to ask your health care provider."

1. "You seem concerned about having surgery."

A nurse is taking the blood pressure of a client with hypertension. The first sound is heard at 140 mm Hg, the second sound is a swishing sound heard at 130 mm Hg, a tapping sound is heard at 100 mm Hg, a muffled sound is heard at 90 mm Hg, and the sound disappears at 72 mm Hg. When recording just the systolic and diastolic readings, what is the diastolic pressure? 1. 72 mm Hg 2. 90 mm Hg 3. 100 mm Hg 4. 130 mm Hg

1. 72 mm Hg

Which clinical indicators identified by the nurse support the probable presence of a fecal impaction in a client? (Select all that apply.) 1. Abdominal cramps 2. Fecal liquid seepage 3. Hyperactive bowel sounds 4. Bright red blood in the stool 5. Decreased number of bowel movements

1. Abdominal cramps 2. Fecal liquid seepage 3. Hyperactive bowel sounds

A nurse is completing the admission assessment of a client with peripheral arterial disease. What assessments are consistent with this diagnosis? (Select all that apply.) 1. Absence of hair on the toes 2 . Superficial ulcer with irregular edges 3. Pitting edema of the lower extremities 4 . Reports of pain associated with exercising 5 . Increased pigmentation of the medial malleolus area

1. Absence of hair on the toes 4 . Reports of pain associated with exercising

A client is scheduled for an adrenalectomy. Which nursing intervention should the nurse anticipate will be prescribed for this client? 1. Administer intravenous (IV) steroids. 2. Provide a high protein diet. 3. Collect a 24-hour urine specimen. 4. Withhold all medications for 48 hours

1. Administer intravenous (IV) steroids.

A client with multiple myeloma who is receiving chemotherapy has a temperature of 102.2° F. The temperature was 99.2° F when it was taken six hours ago. A priority nursing intervention is to: 1. Administer the prescribed antipyretic and notify the charge nurse or primary health care provider 2. Obtain the respirations, pulse, and blood pressure; recheck the temperature in one hour 3. Assess the amount and color of urine; obtain a specimen for a urinalysis 4. Note the consistency of respiratory secretions and obtain a specimen for culture

1. Administer the prescribed antipyretic and notify the charge nurse or primary health care provider

The nurse is assessing a client with hyperthyroidism. For which signs and symptoms should the nurse assess the client? (Select all that apply.) 1. Amenorrhea 2. Hypotension 3. Facial edema 4. Flushed appearance 5. Short attention span

1. Amenorrhea 4. Flushed appearance 5. Short attention span

A client comes to the ambulatory surgery unit on the morning of an elective surgical procedure. The client reports shortness of breath, dizziness, and palpitations. The nurse observes profuse diaphoresis and is concerned that the client may be having either a panic attack or a myocardial infarction. Which assessments support the conclusion that the client may be experiencing a myocardial infarction? (Select all that apply.) 1. Anxiety 2. Chest pain 3. Irregular pulse 4. Fear of losing control 5. Feelings of depersonalization

1. Anxiety 2. Chest pain 3. Irregular pulse

What nursing action will limit hypoxia when suctioning a client's airway? 1. Apply suction only after catheter is inserted. 2. Limit suctioning with catheter to half a minute. 3. Lubricate the catheter with saline before insertion. 4. Use a sterile suction catheter for each suctioning episode.

1. Apply suction only after catheter is inserted.

A client who was in an automobile collision is now in hypovolemic shock. Why is it important for the nurse to take the client's vital signs frequently during the compensatory stage of shock? 1. Arteriolar constriction occurs. 2. The cardiac workload decreases. 3. Contractility of the heart decreases. 4. The parasympathetic nervous system is triggered.

1. Arteriolar constriction occurs. **The early compensation of shock is cardiovascular and is reflected in changes in pulse, blood pressure, and pulse pressure; blood is shunted to vital organs, particularly the heart and brain.

A female client who is scheduled for a thyroidectomy is concerned that the surgery will interfere with her ability to become pregnant. The nurse should base a response on the understanding that: 1. As long as medication is continued, ovulation will occur 2. Hyperthyroidism can cause abortions and fetal anomalies 3. Pregnancy is not advisable for the client with a thyroidectomy 4. Pregnancy affects metabolism and will require decreased thyroid hormone

1. As long as medication is continued, ovulation will occur

A client enters the emergency department reporting shortness of breath and epigastric distress. What should be the triage nurse's first intervention 1. Assess vital signs 2. Insert a saline lock 3. Place client on oxygen 4. Draw blood for troponins

1. Assess vital signs

A client has chronic asthma. For which complication should the nurse monitor this client? 1. Atelectasis 2. Pneumothorax 3. Pulmonary edema 4. Respiratory alkalosis

1. Atelectasis

A nurse is caring for clients with a variety of problems. Which health problem does the nurse determine poses the greatest risk factor for the development of a pulmonary embolus? 1. Atrial fibrillation 2. Forearm laceration 3. Migraine headache 4. Respiratory infection

1. Atrial fibrillation

A nurse is providing discharge instructions for a client with a diagnosis of gastroesophageal reflux disease (GERD). What should the nurse advise the client to do to limit symptoms of GERD? (Select all that apply.) 1. Avoid heavy lifting. 2. Lie down after eating. 3. Avoid drinking alcohol. 4 .Eat small, frequent meals. 5 .Increase fluid intake with meals. 6 . Wear an abdominal binder or girdle

1. Avoid heavy lifting. 3. Avoid drinking alcohol. 4 .Eat small, frequent meals.

A client is admitted to the hospital with a tentative diagnosis of pneumonia. The client has a high fever and is short of breath. Bed rest, oxygen via nasal cannula, an intravenous antibiotic, and blood and sputum specimens for culture and sensitivity (C&S) are prescribed. Place these interventions in the order in which they should be implemented. 1. Bed rest 2. Administration of an antibiotic 3. Oxygen via nasal cannula 4. Specimens for C&S

1. Bed rest 3. Oxygen via nasal cannula 4. Specimens for C&S 2. Administration of an antibiotic

The health care provider prescribes peak and trough levels of an antibiotic for a client who is receiving the medication intravenous piggyback (IVPB). For peak levels the nurse should have the laboratory obtain a blood sample from the client: 1. Between 30 and 60 minutes after the IVPB 2. Halfway between two IVPB administrations 3. Immediately before administering the IVPB 4. Anytime it is convenient for the client and laboratory

1. Between 30 and 60 minutes after the IVPB

A client is admitted to the hospital for acute gastritis and ascites secondary to alcoholism and cirrhosis. It is most important for the nurse to assess this client for: 1. Blood in the stool 2. Food intolerances 3. Complaints of nausea 4. Hourly urinary output

1. Blood in the stool

A client is receiving hydrochlorothiazide (HCTZ). What should the nurse monitor to best determine the effectiveness of the client's hydrochlorothiazide therapy? 1. Blood pressure 2. Decreasing edema 3. Serum sodium level 4. Urine specific gravity

1. Blood pressure

A nurse is caring for a client who has a 20-year history of type 2 diabetes. The nurse should assess for what physiological changes that are associated with a long history of diabetes? 1. Blurry, spotty, or hazy vision 2. Arthritic changes in the hands 3. Hyperactive knee and ankle jerk reflexes 4. Dependent pallor of the feet and lower legs

1. Blurry, spotty, or hazy vision

A nurse is planning to teach facts about hyperglycemia to a client with the diagnosis of diabetes. What information should the nurse include in the discussion about what causes diabetic acidosis? 1. Breakdown of fat stores for energy 2. Ingestion of too many highly acidic foods 3. Excessive secretion of endogenous insulin 4. Increased amounts of cholesterol in the extracellular compartment

1. Breakdown of fat stores for energy

A frantic parent calls stating their child has swallowed dish soap. What would you advise? 1. Call poison control. 2. Induce vomiting immediately. 3. Give syrup of ipecac, one tablespoon. 4. Give activated charcoal, and expect black stools for 24 hours.

1. Call poison control.

A nurse is providing education about excellent food sources of vitamin A for a client who is deficient in this vitamin. Which foods should the nurse include in the teaching? (Select all that apply.) 1. Carrots 2. Oranges 3. Tomatoes 4 . Skim milk 5. Leafy greens

1. Carrots 5. Leafy greens

A beta-blocker, atenolol (Tenormin), is prescribed for a client with moderate hypertension. What information should the nurse include when teaching the client about this medication? (Select all that apply.) 1. Change positions slowly 2. Take the medication before going to bed 3. Count the pulse before taking the medication 4. Mild weakness and fatigue are common side effects 5. It is safe to take concurrent over-the-counter (OTC) medications

1. Change positions slowly 3. Count the pulse before taking the medication 4. Mild weakness and fatigue are common side effects

What should the nurse do when collecting a 24-hour urine specimen? 1. Check to verify if a preservative is needed. 2. Weigh the client before starting the collection. 3. Discard the last voided specimen of the 24-hour period. 4. Assess the client's intake and output (I&O) for the previous 24-hour period.

1. Check to verify if a preservative is needed.

A client on a telemetry unit demonstrates a regular sinus rhythm (RSR) with an occasional premature atrial contraction (PAC). What action should the nurse take? 1. Continue to monitor the client. 2. Notify the health care provider. 3. Ensure that a defibrillator is close by. 4. Administer lidocaine intravenously as per protocol.

1. Continue to monitor the client.

A client diagnosed with asthma has received a prescription for an inhaler. The nurse teaches the client how to determine when the inhaler is empty, instructing the client to: 1. Count the number of doses taken 2. Taste the medication when sprayed into the air 3. Shake the canister 4. Place the canister in water to see if it floats

1. Count the number of doses taken

A client has a body mass index (BMI) of 35 and verbalizes the need to lose weight. The nurse encourages the client to lose weight safely by changing which dietary habits? 1. Decrease portion size and fat intake 2. Increase protein and vegetable intake 3. Decrease carbohydrate and fat intake 4. Increase fruits and limit fluid intake

1. Decrease portion size and fat intake

A nurse instructs a client with a history of frequent urinary tract infections to drink cranberry juice to: 1. Decrease the urinary pH 2. Exert a bactericidal effect 3. Improve glomerular filtration 4. Relieve the symptoms of dysuria

1. Decrease the urinary pH

A client with acute renal failure moves into the diuretic phase after one week of therapy. For which signs during this phase should the nurse assess the client? (Select all that apply.) 1. Dehydration 2. Hypovolemia 3. Hyperkalemia 4. Metabolic acidosis 5. Skin rash

1. Dehydration 2. Hypovolemia

A nurse is caring for a client with right-sided heart failure. Which are key features of right-sided heart failure? (Select all that apply.) 1. Dependent edema 2. Distended abdomen 3. Polyuria at night 4. Collapsed neck veins 5. Cool extremities

1. Dependent edema 2. Distended abdomen 3. Polyuria at night

When assessing the client with peripheral arterial disease, the nurse anticipates the presence of which clinical manifestations? (Select all that apply.) 1. Dependent rubor 2. Warm extremities 3. Ulcers on the toes 4. Thick, hardened skin 5 . Delayed capillary refill

1. Dependent rubor 3. Ulcers on the toes 5 . Delayed capillary refill **Peripheral arterial disease affects arterial circulation and results in delayed and impaired circulation to the extremities. As a result, the extremities exhibit rubor while in the dependent position and pallor while elevated, ulcers on the feet and toes, cool skin, and capillary refill greater than three seconds

What therapeutic effect should the nurse identify as the reason for administration of neomycin sulfate to a client before colon surgery? 1. Destroy intestinal bacteria. 2. Increase production of vitamin K. 3. Decrease the incidence of any secondary infection. 4. Prevent the possibility of postoperative urinary tract infection.

1. Destroy intestinal bacteria

Which clinical findings should the nurse expect when assessing a client with hyperthyroidism? (Select all that apply.) 1. Diarrhea 2. Listlessness 3. Weight loss 4. Bradycardia 5. Decreased appetite

1. Diarrhea 3. Weight loss

A client with heart failure is digitalized and placed on a maintenance dose of digoxin (Lanoxin) 0.25 mg by mouth daily. What responses does the nurse expect the client to exhibit when a therapeutic effect of digoxin is achieved? 1. Diuresis and decreased pulse rate 2. Increased blood pressure and weight loss 3. Regular pulse rhythm and stable fluid balance 4. Corrected heart murmur and decreased pulse pressure

1. Diuresis and decreased pulse rate

Trimethoprim-sulfamethoxazole (Septra) is prescribed for a client with cystitis. When teaching about the medication, the nurse instructs the client to: 1. Drink 8 to 10 glasses of water daily 2. Drink two glasses of orange juice daily 3. Take the medication with meals 4. Take the medication until symptoms subside

1. Drink 8 to 10 glasses of water daily

When preparing a client for discharge after a thyroidectomy, the nurse teaches the signs of hypothyroidism. The nurse evaluates that the client understands the teaching when the client says, "I should call my health care provider if I develop: 1. Dry hair and an intolerance to cold." 2. Muscle cramping and sluggishness." 3. Fatigue and an increased pulse rate." 4. Tachycardia and an increase in weight."

1. Dry hair and an intolerance to cold."

A nurse is assessing a client with diabetic ketoacidosis. Which clinical manifestations should the nurse expect? (Select all that apply.) 1. Dry skin 2. Abdominal pain 3. Kussmaul respirations 4. Absence of ketones in the urine 5. Blood glucose level of less than 100 mg/dL

1. Dry skin 2. Abdominal pain 3. Kussmaul respirations

A nurse teaches a client who has had a thyroidectomy for thyroid cancer to observe for signs of surgically induced hypothyroidism. What should be included in the teaching plan? (Select all that apply.) 1. Dry skin 2. Lethargy 3. Insomnia 4. Tachycardia 5. Sensitivity to cold

1. Dry skin 2. Lethargy 5. Sensitivity to cold

A nurse is caring for a client who is admitted to the hospital with the diagnosis of primary hyperparathyroidism. Which action should be included in this client's plan of care? 1. Ensuring a large fluid intake 2. Providing a high-calcium diet 3. Instituting seizure precautions 4. Encouraging complete bed rest

1. Ensuring a large fluid intake

A nurse is formulating a teaching plan for a client recently diagnosed with type 2 diabetes. What interventions should the nurse include that will decrease the risk of complications? (Select all that apply.) 1. Examining the feet daily 2. Wearing well-fitting shoes 3. Performing regular exercise 4. Powdering the feet after showering 5. Visiting the health care provider weekly 6. Testing bathwater with the toes before bathing

1. Examining the feet daily 2. Wearing well-fitting shoes 3. Performing regular exercise

A nurse is assessing a client with a diagnosis of diabetes insipidus. For which signs indicative of diabetes insipidus should the nurse assess the client? (Select all that apply.) 1. Excessive thirst 2. Increased blood glucose 3. Dry mucous membranes 4. Increased blood pressure 5. Decreased serum osmolarity 6. Decreased urine specific gravity

1. Excessive thirst 3. Dry mucous membranes 6. Decreased urine specific gravity

Before a client's discharge after a thyroidectomy, the nurse teaches the client to observe for signs of surgically induced hypothyroidism. What clinical indicators identified by the client provide evidence that the nurse's instructions are understood? (Select all that apply.) 1. Fatigue 2. Dry skin 3. Insomnia 4. Intolerance to heat 5. Progressive weight loss

1. Fatigue 2. Dry skin

A client is admitted to the hospital with diabetic ketoacidosis. The nurse concludes that the client's elevated ketone level is caused by incomplete oxidation of which nutrient? 1. Fats 2. Protein 3. Potassium 4. Carbohydrates

1. Fats *Incomplete oxidation of fat results in fatty acids that further break down to ketones. Protein metabolism produces nitrogenous waste, causing elevated blood urea nitrogen (BUN), not ketones

A client is to be transferred from the coronary care unit to a progressive care unit. The client asks the nurse, "Are you sure I'm ready for this move?" From this statement the nurse determines that the client most likely is experiencing: 1. Fear 2. Depression 3. Dependency 4. Ambivalence

1. Fear

A client develops iron deficiency anemia. Which of the client's laboratory test results should the nurse expect to be decreased? 1. Ferritin level 2. Platelet count 3. White blood cell count 4. Total iron-binding capacity

1. Ferritin level **Ferritin, a form of stored iron, is reduced with iron deficiency anemia.

A client who had surgery for a ruptured appendix develops peritonitis. What clinical findings related to peritonitis should the nurse expect the client to exhibit? (Select all that apply.) 1. Fever 2. Hyperactivity 3. Extreme hunger 4. Urinary retention 5. Abdominal muscle rigidity

1. Fever 5. Abdominal muscle rigidity

A nurse is concerned about the public health implications of gonorrhea diagnosed in a 16-year-old adolescent. Which should be of most concern to the nurse? 1. Finding the client's contacts 2. Interviewing the client's parents 3. Instructing the client about birth control measures 4. Determining the reasons for the client's promiscuity

1. Finding the client's contacts

An active adolescent is admitted to the hospital for surgery for an ileostomy. When planning a teaching session about self-care, the nurse includes sports that should be avoided by a client with an ileostomy. Which should be included on the list of sports to be avoided? (Select all that apply.) 1. Football 2. Swimming 3. Ice hockey 4. Track events 5. Cross-country skiing

1. Football 3. Ice hockey

When advising a college student about dietary choices, the nurse should consider the caloric value of the most commonly ordered fast foods eaten by active young adults. List the following foods in order from the one with the least number of calories to the one with the most number of calories. 1. Garden salad 2. One slice of French toast 3. French fries 4.Six pieces of chicken tenders 5. Hamburger with cheese

1. Garden salad 2. One slice of French toast 4.Six pieces of chicken tenders 3. French fries 5. Hamburger with cheese

Which food selections by a client with malabsorption syndrome indicate that the nurse's dietary teaching was successful? (Select all that apply.) 1. Green beans 2. Baked potato 3. Noodle pudding 4. Turkey sandwich 5. Whole wheat cereal

1. Green beans 2. Baked potato

A nurse is caring for a client recently diagnosed with type 1 diabetes. For what signs and symptoms of an insulin reaction should the nurse assess this client? (Select all that apply.) 1. Headache 2. Diaphoresis 3. Nervousness 4. Excessive thirst 5. Kussmaul respirations

1. Headache 2. Diaphoresis 3. Nervousness

A nurse is assessing a client for dehydration, The client has had diarrhea and vomiting for 48 hours. What are indicators of dehydration? (Select all that apply.) 1. Headache 2. Protruding eyeballs 3. The client reporting drinking an average of two glasses of water daily 4. The skin on the client's forehead remains tented after being pinched 5. Within four days, the client lost two pounds of weight

1. Headache 4. The skin on the client's forehead remains tented after being pinched

A female client with the diagnosis of Crohn's disease tells the nurse that her boyfriend dates other women. She believes that this behavior causes an increase in her symptoms. What should the nurse do first when counseling this client? 1. Help the client explore attitudes about herself. 2. Educate the client's boyfriend about her illness. 3. Suggest the client should not see her boyfriend for a while. 4. Schedule the client and her boyfriend for a counseling session.

1. Help the client explore attitudes about herself

A client is diagnosed as having kidney failure. During the oliguric phase the nurse should assess the client for: 1. Hyperkalemia 2. Hypocalcemia 3. Hypernatremia 4. Hypoproteinemia

1. Hyperkalemia

A nurse is caring for a client who had an adrenalectomy. For what clinical response should the nurse monitor while steroid therapy is being regulated? 1. Hypotension 2.Hyperglycemia 3. Sodium retention 4. Potassium excretion

1. Hypotension

A client's diet is modified to eliminate foods that act as cardiac stimulants. The nurse should teach the client to avoid what foods? (Select all that apply.) 1. Iced tea 2. Red meat 3. Club soda 4. Hot cocoa 5 . Chocolate pudding

1. Iced tea 4. Hot cocoa 5 . Chocolate pudding

A nurse concludes that a client has a hypoglycemic reaction to insulin. Which clinical findings support this conclusion? (Select all that apply.) 1. Irritability 2. Glycosuria 3. Dry, hot skin 4. Heart palpitations 5. Fruity odor of breath

1. Irritability 4. Heart palpitations

A client is admitted with the diagnosis of acute pancreatitis. For which clinical manifestations should a nurse assess the client? (Select all that apply.) 1. Jaundice 2. Acute pain 3. Hypertension 4. Hypoglycemia 5. Increased amylase

1. Jaundice 2. Acute pain 5. Increased amylase

What clinical manifestations does a nurse expect a client with systemic lupus erythematosus (SLE) most likely to exhibit? (Select all that apply.) 1. Joint pain 2. Facial rash 3. Pericarditis 4. Weight gain 5. Hypotension

1. Joint pain 2. Facial rash 3. Pericarditis

The nurse is caring for a client diagnosed with Cushing syndrome. The nurse expects that the client will exhibit: 1. Lability of mood 2. Hair thinning 3. Increased skin thickness 4. Ectomorphism

1. Lability of mood

Which clinical indicators can the nurse expect when assessing a client with Cushing syndrome? (Select all that apply.) 1. Lability of mood 2. Slow wound healing 3. A decrease in the growth of hair 4. Ectomorphism with a moon face 5. An increased resistance to bruising

1. Lability of mood 2. Slow wound healing

A client comes to the emergency department reporting symptoms of the flu. When the health history reveals intravenous drug use and multiple sexual partners, acute retroviral syndrome is suspected, and a test for the human immunodeficiency virus (HIV) is performed. Which clinical responses are associated most commonly with this syndrome? (Select all that apply.) 1. Malaise 2. Confusion 3. Constipation 4. Swollen lymph glands 5. Oropharyngeal candidiasis

1. Malaise 4. Swollen lymph glands

A nurse is auscultating a client's heart sounds. Which valves close when the first heart sounds are produced? 1. Mitral and tricuspid 2. Aortic and tricuspid 3. Mitral and pulmonic 4. Aortic and pulmonic

1. Mitral and tricuspid

A client with a 20-year history of excessive alcohol use is admitted to the hospital with jaundice and ascites. A priority nursing action during the first 48 hours after the client's admission is to: 1. Monitor the client's vital signs 2. Increase the client's fluid intake 3. Improve the client's nutritional status 4. Determine the client's reasons for drinking

1. Monitor the client's vital signs

A client is receiving total parenteral nutrition solution. Potassium has not been added to the solution. The nurse monitors the client for which signs of hypokalemia? (Select all that apply.) 1. Muscle weakness 2. Metabolic alkalosis 3. Cardiac dysrhythmias 4. Serum potassium of 5.5 mEq/L 5. Respiratory rate of 24 or higher

1. Muscle weakness 3. Cardiac dysrhythmias

A client with gastroesophageal reflux disease (GERD) receives a prescription for an H2 receptor antagonist. Which medications are within the classification of an H2 receptor antagonist? (Select all that apply.) 1. Nizatidine (Axid) 2. Ranitidine (Zantac) 3. Famotidine (Pepcid) 4. Lansoprazole (Prevacid) 5. Metoclopramide (Reglan)

1. Nizatidine (Axid) 2. Ranitidine (Zantac) 3. Famotidine (Pepcid)

A client who is 60 pounds more than the ideal body weight is admitted to the hospital with a diagnosis of type 1 diabetes. Which concept should the nurse include in teaching about diabetes when discussing strategies to lose weight? 1. Obesity leads to insulin resistance 2. Surplus fat causes excretion of insulin 3. Fat cells absorb insulin and prevent its circulation to other cells 4. Lipids accumulate in the pancreas and interfere with insulin production

1. Obesity leads to insulin resistance

A chronically ill, older client tells the home care nurse that the daughter with whom the client lives seems run-down and disinterested in her own health, as well as the health of her children, who are 5, 7, and 12 years old. The client tells the nurse that the daughter coughs a good deal and sleeps a lot. Why is it important that the nurse pursue the daughter's condition for potential case finding? 1. Older adults with chronic illness are more susceptible to tuberculosis. 2. Tuberculosis has been rising dramatically in the general population. 3. There is a high incidence of tuberculosis in children less than 12 years of age. 4. Death from tuberculosis has been generally on the decrease in the United States.

1. Older adults with chronic illness are more susceptible to tuberculosis.

A client is admitted to the emergency department with a stab wound of the left thorax. The nurse should position the client: 1. On the left side with the head of the bed elevated 2. In the Trendelenburg position with knees gatched 3. In the high-Fowler position with the left side supported 4. On the right side flat in bed with a pillow supporting the left arm

1. On the left side with the head of the bed elevated

A nurse is caring for a client who is admitted with ureteral colic and hematuria. The client also has stage 1 hypertension and is overweight. Which assessment finding should the nurse be most concerned about at this time? 1. Pain 2. Weight 3. Hematuria 4. Blood pressure of 120/64

1. Pain

A client with terminal cancer signs a do-not-resuscitate (DNR) order upon admission to the hospital. When the client goes into respiratory arrest a week later, the client is not resuscitated. Which factor does the nurse determine is most relevant to the legal aspects of a DNR order? 1. Policies of the agency establish the status of DNR orders 2. Age is an important factor in the decision not to resuscitate 3. Decisions regarding resuscitation reside with the client's primary health care provider 4. Once a DNR order is signed, it remains in force for the entire hospitalization

1. Policies of the agency establish the status of DNR orders

A client who has always been active is diagnosed with atherosclerosis and hypertension. The client is interested in measures that will help promote and maintain health. What recommendation by the nurse will help the client maintain blood vessel patency? 1. Practice relaxation techniques 2. Lead a more sedentary lifestyle 3. Decrease the amount of exercise 4. Increase saturated fats in the diet

1. Practice relaxation techniques

A client with laryngeal cancer has a partial laryngectomy and tracheostomy. To best facilitate communication postoperatively, the nurse should: 1. Provide a means for the client to write. 2. Allow the client more time for articulation. 3. Use visual clues, such as gestures and objects. 4. Face the client and speak slowly and distinctly

1. Provide a means for the client to write.

A client admitted to the hospital for chest pain is diagnosed with angina. The nurse should teach the client that the most common characteristic of anginal pain is that it is: 1. Relieved by rest 2. Precipitated by light activity 3. Described as sharp or knifelike 4. Unaffected by the administration of vasodilators

1. Relieved by rest **Anginal pain commonly is relieved by immediate rest because rest decreases the cardiac workload.

A client is recovering from an acute episode of alcoholism that included esophageal involvement. What are the components of a therapeutic diet that are most appropriate for the nurse to include in the teaching plan for this client? (Select all that apply.) 1. Soft diet 2. Regular diet 3. Low protein diet 4. High protein diet 5. Low carbohydrate diet 6. High carbohydrate diet

1. Soft diet 4. High protein diet 6. High carbohydrate diet

A client with type 1 diabetes comes to the clinic because of concerns regarding erratic control of blood glucose with the prescribed insulin therapy. The client has been experiencing a sudden fall in the blood glucose level, followed by a sudden episode of hyperglycemia. Which complication of insulin therapy should the nurse conclude that the client is experiencing? 1. Somogyi effect 2. Dawn phenomenon 3. Diabetic ketoacidosis 4. Hyperosmolar nonketotic syndrome

1. Somogyi effect

A nurse is caring for a client after a thyroidectomy. Because of concerns about potential nerve injury associated with this type of surgery, the nurse should assess for which functional ability? 1. Speaking 2. Swallowing 3. Pursing the lips 4. Turning the head

1. Speaking

A client who is a pipe smoker is diagnosed with cancer of the tongue. A hemiglossectomy and right radical neck dissection are performed. To ensure airway patency during the first hours after surgery, the nurse should: 1. Suction as needed 2. Apply an ice collar 3. Maintain a high-Fowler position 4. Encourage expectoration of secretions

1. Suction as needed

Hydrocortisone (Cortef) is prescribed for a client with Addison disease. Before discharge, the nurse teaches the client about this medication. What did the nurse include as a therapeutic effect of the drug? 1. Supports a better response to stress 2. Promotes a decrease in blood pressure 3. Decreases episodes of shortness of breath 4. Controls an excessive loss of potassium from the body

1. Supports a better response to stress

A nurse is caring for a client who just had a thyroidectomy. For which client response should the nurse assess the client when concerned about an accidental removal of the parathyroid glands during surgery? 1. Tetany 2. Myxedema 3. Hypovolemic shock 4. Adrenocortical stimulation

1. Tetany

A client is hospitalized with a diagnosis of emphysema. The nurse provides teaching and should begin with which aspect of care? 1. The disease process and breathing exercises 2. How to control or prevent respiratory infections 3. Using aerosol therapy, especially nebulizers 4. Priorities in carrying out everyday activities

1. The disease process and breathing exercises

A client who is scheduled to have surgery to remove an aldosterone-secreting adenoma asks the nurse what will happen if surgery is not performed. On what information should the nurse base a response? 1. The tumor must be removed to prevent heart and kidney damage. 2. Surgery will prevent the tumor from metastasizing to other organs. 3. Radiation therapy can be just as effective as surgery if the tumor is small. 4. Chemotherapy is as reliable as surgery for the treatment of adenomas of this type in some people

1. The tumor must be removed to prevent heart and kidney damage.

The nurse provides education related to manifestations of hyperglycemia to a client with type 1 diabetes. Which signs and symptoms identified by the client indicate that the teaching was effective? (Select all that apply.) 1. Thirst 2. Headache 3. Nervousness 4. Fruity breath odor 5. Excessive urination

1. Thirst 4. Fruity breath odor 5. Excessive urination

A nurse is caring for a client who was diagnosed with a myocardial infarction. While caring for the client two days after the event, the nurse identifies that the client's temperature is elevated. The nurse concludes that this increase in temperature is most likely the result of: 1. Tissue necrosis 2. Venous thrombosis 3. Pulmonary infarction 4. Respiratory infection

1. Tissue necrosis **The body's inflammatory response to myocardial necrosis causes an elevation of temperature as well as leukocytosis within 24 to 48 hours after the event.

A client has been taking levothyroxine (Synthroid) for hypothyroidism for three weeks. The nurse suspects that a decrease in dosage is needed when the client exhibits which clinical manifestations? (Select all that apply.) 1. Tremors 2. Bradycardia 3. Somnolence 4. Heat intolerance 5. Decreased blood pressure

1. Tremors 4. Heat intolerance

A nurse is teaching a health class to older adult women about heart disease. The nurse discusses the most common prodromal symptom reported by women with acute coronary heart disease that usually is not experienced by men. What response indicates that a woman understood the teaching? 1. Unusual fatigue 2. Shortness of breath 3. Crushing pain in the chest 4. Substernal pressure radiating to the neck

1. Unusual fatigue

A nurse is evaluating a client's response to fluid replacement therapy. Which clinical finding indicates adequate tissue perfusion to vital organs? 1. Urinary output of 30 mL in an hour 2. Central venous pressure reading of 2 mm Hg 3. Baseline pulse rate of 120 that decreases to 110 beats/min within a 15-minute period 4. Baseline blood pressure of 50/30 that increases to 70/40 mm Hg within a 30-minute period

1. Urinary output of 30 mL in an hour **A urinary output rate of 30 mL/hour is considered adequate for perfusion of the kidneys, heart, and brain.

A client is admitted to the hospital with a tentative diagnosis of pneumonia. The client has a high fever and is short of breath. Bed rest, oxygen via nasal cannula, an intravenous antibiotic, and blood and sputum specimens for culture and sensitivity (C&S) are prescribed. Place these interventions in the order in which they should be implemented.

1.Oxygen via nasal cannula 3.Bed rest 2.Specimens for C&S 4.Administration of an antibiotic

A client is admitted to the hospital from the emergency department with a diagnosis of urolithiasis. The nurse reviews the client's clinical record and performs an admission assessment. What is the priority nursing action?

2 Administer the prescribed morphine.

A client with a history of chronic kidney disease is hospitalized. The nurse assesses the client for signs of related kidney insufficiency, which include:

2 Edema and pruritus

The nurse is caring for a client with type 1 diabetes. For which signs or symptoms of insulin reaction should the nurse particularly be observant? (Select all that apply.) 1. Lethargy 2. Headache 3. Diaphoresis 4. Excessive thirst 5. Deep respirations

2 Headache 3 Diaphoresis

A client with acute kidney failure is to receive peritoneal dialysis and asks why the procedure is necessary. The nurse's best response is, "It:

2 Helps perform some of the work usually done by the kidneys."

A nurse is caring for a client who is being treated with continuous ambulatory peritoneal dialysis (CAPD) for chronic glomerulonephritis. What dietary need should the nurse discuss with the client?

2 High-quality protein

Postoperatively a client who had a thyroidectomy complains of tingling and numbness of the fingers and toes, and the nurse observes muscle twitching. Which complication does the nurse suspect the client is experiencing? 1 Hypokalemia 2 Hypocalcemia 3 Thyrotoxic crisis 4 Hypovolemic shock

2 Hypocalcemia

A client manifests right-sided hemianopsia as a result of a brain attack (CVA). The nurse develops a plan of care and includes:

2 Instruct the client to scan surroundings

A nurse is assessing a client who is scheduled for a liver biopsy. What assessment finding needs to be reported immediately because it warrants a postponement of the liver biopsy?

2 International normalized ratio (INR) of 4.0

. The nurse teaches a premenopausal obese client about strategies to prevent osteoporosis. Which strategy identified by the client indicates that the teaching is effective

2 Joins a tennis league and practices every day.

To reduce the risk of recurrent painful gout attacks, the nurse teaches the client to avoid which foods? (Select all that apply.)

2 Liver 5 shellfish

A client sustains a complex comminuted fracture of the tibia with soft tissue injuries after being hit by a car while riding a bicycle. Surgical placement of an external fixator is performed to maintain the bone in alignment. Postoperatively it is most essential for the nurse to:

2 Perform a neurovascular assessment of both lower extremities

A postmenopausal woman who has cancer of the breast decides to have a lumpectomy followed by chemotherapy. After receiving chemotherapy for several weeks, she says to a nurse at the clinic, "I don't feel well." The nurse reviews the medical record. Based on this information, what does the nurse conclude is the client's priority need?

2 Preventing infection

A nurse is caring for a client with chronic kidney failure. What should the nurse teach the client to limit the intake of to help control uremia associated with end stage renal disease (ESRD)?

2 Protein

A client is recuperating from a spinal cord injury at the T4 level and depends on a wheelchair for mobility. What should the nurse teach the client to prepare for use of a wheelchair?

2 Push-ups to strengthen arm muscles

A nurse is caring for a client who has a radium implant for cancer of the cervix. What is the priority nursing intervention?

2 Restrict visitors to a 10-minute stay.

Which test result should a nurse review to determine if the antibiotic prescribed for the client will be effective?

2 Sensitivity test

A nurse is caring for a client who had a kidney transplant. Which test is most important for determining whether a client's newly transplanted kidney is working effectively?

2 Serum creatinine

A nurse is providing postoperative care to a client who had a kidney transplant. What assessment is the best indicator of the functioning of the newly transplanted kidney?

2 Serum creatinine

After surgery for cancer of the posterior pharynx, a client is receiving gavage feedings through a nasogastric tube. A family member asks why this is necessary. What is the nurse's best response?

2 Tube feedings promote healing by reducing the risk for infection.

The nurse determines that which genitourinary factor contributes to urinary incontinence in older adults

2 Urinary tract infection

A nurse is planning to transfer a client who is experiencing pain from the bed to a chair. Place the following steps in the order in which they should be implemented.

2 Verify the client's activity prescription. 1 Identify factors that may impact the ability to transfer. 3 Explain the steps of the transfer. 4 Ensure that the wheels on the bed are locked. 5 Position the client in functional body alignment before transferring

A client with type 1 diabetes self-administers Novolin N insulin every morning at 8 AM. The nurse evaluates that the client understands the action of the insulin when the client says, "I should be alert for signs of hypoglycemia between:

2 pm and 8 pm.

A client is scheduled for a transurethral resection of the prostate. What should the nurse tell the client to expect after surgery? 1. "Urinary control may be permanently lost to some degree." 2. "An indwelling urinary catheter is required for at least a day." 3. "Your ability to perform sexually will be impaired permanently." 4. "Burning on urination will last while the cystotomy tube is in place."

2. "An indwelling urinary catheter is required for at least a day."

A client with tuberculosis is started on a chemotherapy protocol that includes rifampin (RIF). The nurse evaluates that the teaching about rifampin is effective when the client states: 1. "I need to drink a lot of fluid while I take this medication." 2. "I can expect my urine to turn orange from this medication." 3. "I should have my hearing tested while I take this medication." 4. "I might get a skin rash because it is an expected side effect of this medication."

2. "I can expect my urine to turn orange from this medication."

A client is hospitalized with a tentative diagnosis of pancreatic cancer. On admission the client asks the nurse, "Do you think I have anything serious, like cancer?" What is the nurse's best reply? 1. "What makes you think you have cancer?" 2. "I don't know if you do; let's talk about it." 3. "Why don't you discuss this with your health care provider?" 4. "You needn't worry now; we won't know the answer for a few days."

2. "I don't know if you do; let's talk about it."

A client is learning alternate site testing (AST) for glucose monitoring. Which client statement indicates to the nurse that additional teaching is necessary? 1. "I need to rub my forearm vigorously until warm before testing at this site." 2. "The fingertip is preferred for glucose monitoring if hyperglycemia is suspected." 3. "Alternate site testing is unsafe if I am experiencing a rapid change in glucose levels." 4. "I have to make sure that my current glucose monitor can be used at an alternative site."

2. "The fingertip is preferred for glucose monitoring if hyperglycemia is suspected."

A client has a permanent colostomy. During the first 24 hours, there is no drainage from the colostomy. The nurse concludes that this is a result of the: 1. Edema after the surgery 2. Absence of intestinal peristalsis 3. Decrease in fluid intake before surgery 4. Effective functioning of the nasogastric tube

2. Absence of intestinal peristalsis

A client is admitted with a tentative diagnosis of pancreatitis. The medical and nursing measures for this client are aimed toward maintaining nutrition, promoting rest, maintaining fluid and electrolyte balance, and decreasing anxiety. Which interventions should the nurse implement? (Select all that apply.) 1. Provide a low fat diet. 2. Administer analgesics. 3. Teach relaxation exercises. 4. Encourage walking in the hall. 5. Monitor cardiac rate and rhythm. 6. Observe for signs of hypercalcemia

2. Administer analgesics. 3. Teach relaxation exercises. 5. Monitor cardiac rate and rhythm.

The nurse is caring for a 75-year-old client that had radical head and neck surgery. Thirty minutes after awakening from anesthesia, the client becomes agitated, disoriented, and confused. The nurse should: 1. Notify the health care provider immediately of the findings 2. Administer the prescribed oxygen 3. Record the observations and continue to observe the client 4. Administer the prescribed antianxiety medication

2. Administer the prescribed oxygen

Discharge instructions for the client diagnosed with cirrhosis with varices should include information about the importance of: (Select all that apply.) 1. Adhering to a low carbohydrate diet 2. Avoiding aspirin and aspirin containing products 3. Limiting alcohol consumption to two drinks weekly 4. Avoiding acetaminophen and products containing acetaminophen 5. Avoiding coughing, sneezing, and straining to have a bowel movement

2. Avoiding aspirin and aspirin containing products 4. Avoiding acetaminophen and products containing acetaminophen 5. Avoiding coughing, sneezing, and straining to have a bowel movement

A nurse provides dietary instruction to a client who has iron deficiency anemia. Which food choices by the client does the nurse consider most desirable? (Select all that apply.) 1. Raw carrots 2. Boiled spinach 3. Sweet potatoes 4. Brussels sprouts 5. Asparagus spears

2. Boiled spinach 3. Sweet potatoes

A nurse educator of a college health course is discussing tattoos with the class. Which type of hepatitis associated with tattoos should the nurse include in the teaching plan? 1. A 2. C 3. D 4. E

2. C **hepatitis C is a bloodborne pathogen that can be transmitted via contaminated tattoo needles. Hepatitis A is not a bloodborne pathogen; it is spread through contaminated food or water. Although hepatitis D is a bloodborne pathogen, it can be produced only when the hepatitis B virus is present. Also, hepatitis D is not the main virus associated with contaminated tattoo needles. Hepatitis E is believed to be transmitted via the fecal-oral route; it is spread through contaminated food or water.

After a surgical thyroidectomy a client exhibits carpopedal spasm and some tremors. The client complains of tingling in the fingers and around the mouth. What medication should the nurse expect the primary health care provider to prescribe after being notified of the client's adaptations? 1. Potassium iodide 2. Calcium gluconate 3. Magnesium sulfate 4. Potassium chloride

2. Calcium gluconate **The client is exhibiting signs and symptoms of hypocalcemia, which occurs with accidental removal of the parathyroid glands; calcium gluconate is administered to treat hypocalcemia. Potassium iodide is prescribed for hyperthyroidism because it inhibits the release of thyroid hormones. Magnesium sulfate is prescribed for hypomagnesemia or to treat pregnant women who have preeclampsia. Potassium chloride is prescribed for hypokalemia, not hypocalcemia.

The nurse identifies that the dietary teaching provided for a client with diabetes is understood when the client states, "My diet: 1. Should be rigidly controlled to avoid emergencies." 2. Can be planned around a wide variety of commonly used foods." 3. Is based on nutritional requirements that are the same for all people." 4. Must not include eating any combination dishes and processed foods."

2. Can be planned around a wide variety of commonly used foods."

A client is diagnosed with emphysema. For what long-term problem should the nurse monitor this client? 1. Localized tissue necrosis 2. Carbon dioxide retention 3. Increased respiratory rate 4. Saturated hemoglobin molecules

2. Carbon dioxide retention

The nurse is conducting a nutrition class for a group of clients with congestive heart failure (CHF). It would be most important for the nurse to explain the importance of: 1. Restricting fluid intake 2. Choosing fresh or frozen vegetables instead of canned ones 3. Eating a low-calorie diet to reduce weight 4. Recognizing which products are high in cholesterol

2. Choosing fresh or frozen vegetables instead of canned ones **The key principle to teach CHF clients is the importance of decreasing sodium in their diet and which foods contain sodium.

When monitoring a client for hyponatremia, what clinical findings should the nurse consider significant? (Select all that apply.) 1. Thirst 2. Confusion 3. Tachycardia 4. Pale coloring 5. Poor tissue turgor

2. Confusion 5. Poor tissue turgor

A nurse is teaching a group of clients with peripheral vascular disease about a smoking cessation program. Which physiological effect of nicotine should the nurse explain to the group? 1. Constriction of the superficial vessels dilates the deep vessels. 2. Constriction of the peripheral vessels increases the force of flow. 3. Dilation of the superficial vessels causes constriction of collateral circulation. 4. Dilation of the peripheral vessels causes reflex constriction of visceral vessels

2. Constriction of the peripheral vessels increases the force of flow.

The nurse is caring for a client that is admitted with the diagnosis of mild chronic heart failure. The nurse expects to hear what lung sounds? 1. Stridor 2. Crackles 3. Wheezes 4. Friction rubs

2. Crackles **Left-sided heart failure causes fluid accumulation in the capillary network of the lungs; fluid eventually enters alveolar spaces and causes crackling sounds at the end of inspiration.

A low-residue diet is recommended for a client. Which food should the nurse encourage the client to select from a menu? 1. Steamed broccoli 2. Creamed potatoes 3. Raw spinach salad 4. Baked sweet potato

2. Creamed potatoes

Thick mucous gland secretions, elevated sweat electrolytes, meconium ileus, and difficulty maintaining and gaining weight are associated with this autosomal recessive disorder: 1. Cerebral palsy 2. Cystic fibrosis 3. Muscular dystrophy 4. Multiple sclerosis

2. Cystic fibrosis

What is the primary focus of nursing care for a client admitted with tetanus caused by a puncture wound? 1. Monitoring urinary output 2. Decreasing external stimuli 3. Maintaining body alignment 4. Encouraging high intake of fluid

2. Decreasing external stimuli

The nurse is providing postoperative care for a client who has received a prescription for nalbuphine (Nubain) for pain. For which side effects or adverse reactions should the nurse assess this client after administering this medication? (Select all that apply.) 1. Oliguria 2. Dry mouth 3. Palpitations 4. Constipation 5. Urinary retention 6 . Orthostatic hypotension

2. Dry mouth 3. Palpitations 4. Constipation 6 . Orthostatic hypotension

The nurse is providing postoperative care eight hours after a client had a total cystectomy and the formation of an ileal conduit. What assessment finding should be reported immediately? 1. Edematous stoma 2. Dusky-colored stoma 3. Absence of bowel sounds 4. Pink-tinged urinary drainage

2. Dusky-colored stoma

A nurse is caring for several postoperative clients. For what clinical manifestations of a pulmonary embolus should the nurse monitor these clients? (Select all that apply.) 1. Apathy 2. Dyspnea 3. Hemoptysis 4. Bronchial wheezes 5. Feeling of impending doom

2. Dyspnea 3. Hemoptysis 5. Feeling of impending doom

A health care provider prescribes psyllium (Metamucil) 3.5 g twice a day for constipation. What is most important for the nurse to teach this client? 1. Urine may be discolored. 2. Each dose should be taken with a full glass of water. 3. Use only when necessary because it can cause dependence. 4. Daily use may inhibit the absorption of some fat-soluble vitamins.

2. Each dose should be taken with a full glass of water.

A client with a history of chronic kidney disease is hospitalized. The nurse assesses the client for signs of related kidney insufficiency, which include: 1. Facial flushing 2. Edema and pruritus 3. Dribbling after voiding 4. Diminished force and caliber of stream

2. Edema and pruritus

A client with extensive bone and soft tissue injuries to the right leg is on bed rest. When positioning the client, the nurse should: 1. Keep the right leg resting straight on the bed, parallel to the left leg 2. Elevate the entire right leg with pillows, keeping the foot higher than the knee 3. Maintain both legs on the bed and use an abduction pillow to keep them separated 4. Attach a padded ankle sling to a Balkan frame to support the right foot and elevate the leg

2. Elevate the entire right leg with pillows, keeping the foot higher than the knee

A nurse is administering erythropoietin (Epogen) three times a week to a client receiving chemotherapy for cancer. Which client response is considered most expected? 1. Elevated liver enzymes 2. Elevated hematocrit level 3. Increase in the white blood cell (WBC) count 4. Increase in Kaposi's sarcoma lesions

2. Elevated hematocrit level

When assessing a client's abdomen, the nurse palpates the area directly above the umbilicus. This area is known as the: 1. Iliac area 2. Epigastric area 3. Hypogastric area 4. Suprasternal area

2. Epigastric area

A nurse is caring for a client who has a prescription for a diuretic, 2-gram sodium diet, and an oral fluid restriction of 1200 mL daily. The most recent laboratory results are blood urea nitrogen (BUN) level 42 mg/dL and creatinine 1.1 mg/dL. Considering the assessment findings, what is the most appropriate intervention by the nurse? 1. Sending the client's urine for analysis 2. Expecting an increase in the oral fluid intake 3. Placing the client on strict intake and output measurements 4. Notifying a nutritionist/dietitian so that sodium can be restricted further

2. Expecting an increase in the oral fluid intake

The nurse reviews a client's medication history, which includes a cholinergic medication. The client states, "I take that for some kind of urinary problem." The nurse recalls that cholinergic medications are prescribed primarily for what type of urinary condition? 1. Kidney stones 2. Flaccid bladder 3. Spastic bladder 4. Urinary tract infections

2. Flaccid bladder

In addition to Pneumocystis jiroveci, a client with acquired immunodeficiency syndrome (AIDS) also has an ulcer 4 cm in diameter on the leg. Considering the client's total health status, the most critical concern is: 1. Skin integrity 2. Gas exchange 3. Social isolation 4. Nutritional status

2. Gas exchange *P. jiroveci , now believed to be a fungus, causes pneumonia in immunosuppressed hosts; it can cause death in 60% of the clients. The client's respiratory status is the priority.

A client is diagnosed with Cushing syndrome. Which clinical manifestation does the nurse expect to increase in a client with Cushing syndrome? 1. Urine output 2. Glucose level 3. Serum potassium 4. Immune response

2. Glucose level

A nurse is caring for a client with a diagnosis of acute salpingitis. Which condition most commonly causing inflammation of the fallopian tubes should the nurse include when planning a teaching program for this client? 1. Syphilis 2. Gonorrhea 3. Hydatidiform mole 4. Spontaneous abortion

2. Gonorrhea

A client develops increased respiratory secretions because of radiation therapy to the lung, and the health care provider prescribes postural drainage. What client assessment leads the nurse to determine that the postural drainage is effective? 1. Is free of crackles 2. Has a productive cough 3. Is able to expectorate saliva 4. Can breathe deeply through the nose

2. Has a productive cough

conclusion? (Select all that apply.) 1. Vomiting 2. Headache 3. Tachycardia 4. Cool clammy skin 5. Increased respirations

2. Headache 3. Tachycardia 4. Cool clammy skin

A client who had a transurethral resection of the prostate is transferred to the post-anesthesia care unit with an IV and a urinary retention catheter. For which major complication is it most important for the nurse to assess during the immediate postoperative period? 1. Sepsis 2. Hemorrhage 3. Leakage around the catheter 4. Urinary retention with overflow

2. Hemorrhage

An acute, life-threatening complication for which a nurse should assess a client in the early postoperative period after a radical nephrectomy is: 1. Sepsis 2. Hemorrhage 3. Renal failure 4. Paralytic ileus

2. Hemorrhage

A mother with the diagnosis of acquired immunodeficiency disease (AIDS) states that she has been caring for her baby even though she has not been feeling well. What important information should the nurse determine? 1. If she has kissed the baby 2. If the baby is breastfeeding 3. When the baby last received antibiotics 4. How long she has been caring for the baby

2. If the baby is breastfeeding

Steroid therapy is prescribed for a client with an exacerbation of ulcerative colitis. The nurse evaluates that teaching is effective when the client says, "I should take this medicine: 1. At bedtime with a snack." 2. In the early morning with food." 3. One hour before or two hours after eating." 4. By dividing it into equal parts for each meal."

2. In the early morning with food."

A nurse is caring for a client who is scheduled for cystoscopy. What should the nurse include in the client's postprocedure teaching plan? 1. Remain flat in bed for the first 24 hours 2. Increase fluid intake for three to four days postoperatively 3. Notify the nurse if there is any drainage on the dressing 4. Bear down when attempting to void during the first six hours

2. Increase fluid intake for three to four days postoperatively

A client has coronary artery bypass graft (CABG) surgery for the second time via a sternal incision. What should the nurse teach the client to expect when returning home? 1. No further drainage from the incisions 2. Increased edema in the leg that provided the donor graft 3. Mild incisional pain and tenderness for three to four weeks 4. Extreme fatigue and a mild fever occurring for several weeks

2. Increased edema in the leg that provided the donor graft

The nurse is caring for a client with acute renal failure. The most serious complication for this client is: 1. Anemia 2. Infection 3. Weight loss 4. Platelet dysfunction

2. Infection **Infection is responsible for one third of the traumatic or surgically induced deaths of clients with acute renal failure, as well as for medically induced acute renal failure.

A nurse is caring for a client with chronic kidney failure. Which clinical findings should the nurse expect when assessing this client? (Select all that apply.) 1. Polyuria 2. Lethargy 3. Hypotension 4. Muscle twitching 5. Respiratory acidosis

2. Lethargy 4. Muscle twitching

A nurse is caring for a client with a diagnosis of renal calculi secondary to hyperparathyroidism. Which type of diet should the nurse explore with the client when providing discharge information? 1. Low purine 2. Low calcium 3. High phosphorus 4. High alkaline ash

2. Low calcium

A nurse is caring for a client with hepatic cirrhosis. Which elements are important to include in this client's diet? (Select all that apply.) 1. High fat 2. Low protein 3. Low sodium 4. High vitamins 5 . Low carbohydrates

2. Low protein 3. Low sodium 4. High vitamins

A client's urine specific gravity is being measured. For what condition should the nurse conduct a focused assessment when a client's specific gravity is increased? 1. Fluid overload 2. Low-grade fever 3. Diabetes insipidus 4. Chronic kidney disease

2. Low-grade fever

A client with an ileal conduit is being prepared for discharge. As part of the discharge teaching, the nurse instructs the client to: 1. Abstain from beer and alcohol consumption 2. Maintain fluid intake of at least 2 L daily 3. Notify the health care provider if the stoma size decreases 4. Avoid getting soap and water on the peristomal skin

2. Maintain fluid intake of at least 2 L daily

A client is admitted to the hospital with urinary retention, and an indwelling urinary catheter is prescribed by the health care provider. What should the nurse do to help prevent the client from developing a urinary tract infection? 1. Assess urine specific gravity 2. Maintain the prescribed hydration 3. Collect a weekly urine specimen 4. Empty the drainage bag frequently

2. Maintain the prescribed hydration

A client is receiving dexamethasone (Decadron) for adrenocortical insufficiency. To monitor for a negative side effect of the medication, the nurse should: 1. Auscultate for bowel sounds 2. Measure blood glucose levels 3. Culture respiratory secretions 4. Assess deep tendon reflexes

2. Measure blood glucose levels *Corticosteroids, such as dexamethasone, have a hyperglycemic effect, and blood glucose levels should be monitored routinely.

A client has been experiencing extreme fatigue lately. The nurse suspects anemia and examines the client to identify additional clinical manifestations to support this inference. What locations on the client's body should the nurse assess? (Select all that apply.) 1. Sclera 2. Nail beds 3. Lining of eyelids 4. Palms of hands 5. Bony prominences

2. Nail beds 3. Lining of eyelids 4. Palms of hands **Nail beds lose their pink coloration because of reduced hemoglobin. A reduced amount of hemoglobin decreases pink color of the lining of the eyelids. Palms of the hands will become pale because of the decreased hemoglobin.

A client with diabetes mellitus complains of difficulty seeing. The nurse concludes that the causative factor is: 1. Lack of glucose in the retina 2. Neovascularization of the retina 3. Inadequate glucose supply to rods and cones 4. Destructive effect of ketones on retinal metabolism

2. Neovascularization of the retina

A client who had a suprapubic prostatectomy returns from the post-anesthesia care unit and accidentally pulls out the urethral catheter. What should the nurse do first? 1. Reinsert a new catheter. 2. Notify the health care provider. 3. Check for bleeding by irrigating the suprapubic tube. 4. Take no immediate action if the suprapubic tube is draining.

2. Notify the health care provider.

A client is diagnosed with calcium oxalate renal calculi. Which nutrients should the nurse teach the client to avoid? (Select all that apply.) 1. Milk 2. Nuts 3. Liver 4. Spinach 5. Rhubarb

2. Nuts 4. Spinach 5. Rhubarb

A nurse is caring for a male client with a diagnosis of Cushing syndrome. Which clinical manifestations does the nurse expect to identify? (Select all that apply.) 1. Polyuria 2. Obese trunk 3. Hypotension 4. Sleep disturbance 5. Thin arms and legs

2. Obese trunk 4. Sleep disturbance 5. Thin arms and legs

A client who had a subtotal thyroidectomy returns to the unit from the postanesthesia care unit. What is the priority nursing action at this time? 1. Monitor for hypoglycemia. 2. Observe for signs of tetany. 3. Place a sandbag under the neck. 4. Teach the need to support the head.

2. Observe for signs of tetany.

On the third postoperative day after a subtotal gastrectomy, a client complains of severe abdominal pain. The nurse palpates the client's abdomen and identifies rigidity. What action should the nurse take? 1. Assist the client to ambulate 2. Obtain the client's vital signs 3. Administer the prescribed analgesic 4. Encourage the use of spirometry

2. Obtain the client's vital signs

A nurse is caring for an older bedridden male client who is incontinent of urine. What nursing intervention is the most satisfactory initial approach to managing urinary incontinence? 1. Restricting fluid intake 2. Offering the urinal regularly 3. Applying incontinence pants 4. Inserting an indwelling urinary catheter

2. Offering the urinal regularly

When a client returns from the postanesthesia care unit after a kidney transplant, the nurse should plan to measure the client's urinary output every: 1. 15 minutes 2. One hour 3. Two hours 4. Three hours

2. One hour

A client develops subcutaneous emphysema after a chest injury with suspected pneumothorax. What assessment by the nurse is the best method for detecting this complication? 1. Percussing the neck and chest. 2. Palpating the neck or face. 3. Auscultating for abnormal breath sounds. 4. Observing for asymmetry of chest movement.

2. Palpating the neck or face. *Subcutaneous emphysema refers to the presence of air in the tissue that surrounds an opening in the normally closed respiratory tract; the tissue appears puffy, and a crackling sensation is detected when trapped air is compressed between the nurse's palpating fingertips and the client's tissue.

A nurse is assessing a client with a diagnosis of hypoglycemia. What clinical manifestations support this diagnosis? (Select all that apply.) 1. Thirst 2. Palpitations 3. Diaphoresis 4. Slurred speech 5. Hyperventilation

2. Palpitations 3. Diaphoresis 4. Slurred speech

Which responses should a nurse expect a client experiencing hypoglycemia to exhibit? (Select all that apply.) 1. Nausea 2. Palpitations 3. Tachycardia 4. Nervousness 5. Warm, dry skin 6. Increased respirations

2. Palpitations 3. Tachycardia 4. Nervousness

A client is admitted to the hospital with a diagnosis of chronic kidney disease. Which responses should the nurse expect the client to exhibit? (Select all that apply.) 1. Polyuria 2. Paresthesias 3. Hypertension 4. Metabolic alkalosis 5. Widening pulse pressure

2. Paresthesias 3. Hypertension

A client develops heart failure. Which response should the nurse expect when assessing the client? 1. Weight loss 2. Peripheral edema 3. Decreased heart rate 4. Increased urinary output

2. Peripheral edema

A nurse is caring for a postoperative client who has diabetes. Which is the most common cause of diabetic ketoacidosis that the nurse needs to consider when caring for this client? 1. Emotional stress 2. Presence of infection 3. Increased insulin dose 4. Inadequate food intake

2. Presence of infection

A client with a history of alcoholism and cirrhosis is admitted with severe dyspnea as a result of ascites. The nurse concludes that the ascites is most likely the result of increased: 1. Secretion of bile salts 2. Pressure in the portal vein 3. Interstitial osmotic pressure 4. Production of serum albumin

2. Pressure in the portal vein **The enlarged cirrhotic liver impinges on the portal system, causing increased hydrostatic pressure and resulting in ascites.

A client with severe Crohn's disease develops a small bowel obstruction. Which clinical finding should the nurse expect the client to report? 1. Bloody vomitus 2. Projectile vomiting 3. Bleeding with defecation 4. Pain in the left lower quadrant

2. Projectile vomiting **Nausea and vomiting, accompanied by diffuse abdominal pain, commonly occur in clients with small bowel obstruction; the vomiting may be projectile and may contain bile or fecal material.

The nurse who is teaching the client with chronic obstructive pulmonary disease (COPD) to use pursed-lip breathing (PLB) is aware that this is beneficial for the client through which mechanism? 1. Increased respiratory rate to improve arterial oxygenation 2. Prolonged exhalation to decrease air trapping 3. Shortened inhalation to reduce bronchial swelling 4. Use of the diaphragm to increase the amount of inspired air

2. Prolonged exhalation to decrease air trapping

A client with Crohn's disease is admitted to the hospital with a history of chronic, bloody diarrhea, weight loss, and signs of general malnutrition. The client has anemia, a low serum albumin level, and signs of negative nitrogen balance. The nurse concludes that the client's health status is related to a major deficiency of: 1. Iron 2. Protein 3. Vitamin C 4. Linoleic acid

2. Protein

Morphine via an epidural catheter is prescribed for a client after abdominal surgery. The client asks the nurse why this medicine is necessary. What primary rationale does the nurse give for the administration of an opioid analgesic after abdominal surgery? 1. Facilitates oxygen use 2. Relieves abdominal pain 3. Decreases anxiety and restlessness 4. Dilates coronary and peripheral blood vessels

2. Relieves abdominal pain

A client who has acromegaly and insulin-dependent diabetes undergoes a hypophysectomy. The nurse identifies that further teaching about the hypophysectomy is necessary when the client states, "I know I will: 1. Be sterile for the rest of my life." 2. Require larger doses of insulin than I did preoperatively." 3. Have to take cortisone or a similar drug for the rest of my life." 4. Have to take thyroxine or a similar medication for the rest of my life."

2. Require larger doses of insulin than I did preoperatively."

A nurse is caring for a client who has a radium implant for cancer of the cervix. What is the priority nursing intervention? 1. Store urine in lead-lined containers. 2. Restrict visitors to a 10-minute stay. 3. Wear a lead-lined apron when giving care. 4. Avoid giving injections in the gluteal muscle.

2. Restrict visitors to a 10-minute stay.

A nurse is caring for a client with an infection caused by group A beta-hemolytic streptococci. The nurse should assess this client for responses associated with which illness? 1. Hepatitis A 2. Rheumatic fever 3. Spinal meningitis 4. Rheumatoid arthritis

2. Rheumatic fever

A sexually active client presents with a sore throat and a generalized rash. The client states that a chancre that had been present healed approximately three months ago. The physical assessment and the serologic test findings indicate a diagnosis of syphilis. The nurse recognizes that the client is experiencing what stage of syphilis? 1. Primary 2. Secondary 3. Latent 4. Tertiary

2. Secondary

Which test result should a nurse review to determine if the antibiotic prescribed for the client will be effective? 1. Serologic test 2. Sensitivity test 3. Serum osmolality 4. Sedimentation rate

2. Sensitivity test

A client has an open reduction and internal fixation of a fractured hip. To prevent the most common complication after this type of surgery, the nurse expects the client's postoperative plan of care to include: 1. Routinely turning the client from side to side 2. Sequential compression stockings 3. Isometric exercises to the extremities 4. Passive range of motion (ROM) to the affected extremity

2. Sequential compression stockings

A nurse is caring for a client who had a kidney transplant. Which test is most important for determining whether a client's newly transplanted kidney is working effectively? 1 Renal scan 2. Serum creatinine 3. 24-hour urine output 4. White blood cell (WBC) count

2. Serum creatinine

A client with a history of cardiac dysrhythmias is admitted to the hospital with dehydration. What does the nurse expect to be listed on the client's plan of care? 1. A glass of water every hour until hydrated. 2. Small, frequent intake of juices, broth, or milk. 3. Short-term nasogastric (NG) replacement of fluids and nutrients. 4. A rapid intravenous (IV) infusion of an electrolyte and glucose solution.

2. Small, frequent intake of juices, broth, or milk.

A client who has been on hemodialysis for several weeks asks the nurse what substances are being removed by the dialysis. The nurse informs the client that one of the substances passing through the membrane is: 1. Blood 2. Sodium 3. Glucose 4. Bacteria

2. Sodium

A nurse evaluates that a client with diabetes understands the teaching about the treatment of hypoglycemia when the client says, "If I become hypoglycemic I initially should eat: 1. Fruit juice and a lollipop. 2. Sugar and a slice of bread. 3 .Chocolate candy and a banana. 4. Peanut butter crackers and a glass of milk.

2. Sugar and a slice of bread.

A client with Crohn's disease is admitted to the hospital with abdominal pain, fever, poor skin turgor, and diarrhea, with 10 stools in the past 24 hours. Which signs are evidence that the client probably is dehydrated? (Select all that apply.) 1. Moist skin 2. Sunken eyes 3. Decreased apical pulse 4. Dry mucous membranes 5. Increased blood pressure

2. Sunken eyes 4. Dry mucous membranes

A client, visiting the health center, reports feeling nervous, irritable, and extremely tired. The client says to the nurse, "Although I eat a lot of food, I have frequent bouts of diarrhea and am losing weight." The nurse observes a fine hand tremor, an exaggerated reaction to external stimuli, and a wide-eyed expression. What laboratory tests may be prescribed to determine the cause of these signs and symptoms? 1. Partial thromboplastin time (PTT) and prothrombin time (PT) 2. T3, T4, and thyroid-stimulating hormone (TSH) 3. Venereal disease research laboratory (VDRL) test and complete blood count (CBC) 4. Adrenocorticotropic hormone (ACTH), antidiuretic hormone ADH, and corticotropin-releasing factor (CRF)

2. T3, T4, and thyroid-stimulating hormone (TSH)

A client is admitted to the emergency department after vomiting bright red blood. After the vomiting ceases and the vital signs are stabilized, the client is transferred to a medical-surgical unit. To assess for bleeding, the nurse on the medical-surgical unit should monitor the client for: 1. Lethargy 2. Tachycardia 3. Deep breathing 4. Abdominal pain

2. Tachycardia

Which clinical findings should the nurse expect when assessing a client with hyperthyroidism? (Select all that apply.) 1. Lethargy 2. Tachycardia 3. Weight gain 4. Constipation 5. Exophthalmos

2. Tachycardia 5. Exophthalmos

A nurse provides instruction when the beta-blocker atenolol (Tenormin) is prescribed for a client with moderate hypertension. What action identified by the client indicates to the nurse that the client needs further teaching? 1. Move slowly when changing positions. 2. Take the medication before going to bed. 3. Expect to feel drowsy when taking this drug. 4. Count the pulse before taking the medication.

2. Take the medication before going to bed. **Beta blockers (BBs) should not be taken at night because the blood pressure usually decreases when sleeping.

A client with varicose veins asks a nurse what is involved when ligation and stripping are performed rather than sclerotherapy. What should the nurse consider when planning a response in language the client will understand? 1. Plaque from within the veins is abraded. 2. The dilated saphenous veins are removed. 3. Superficial veins are anastomosed to deep veins. 4. An umbrella filter is placed in the large affected veins.

2. The dilated saphenous veins are removed.

A client has surgery to replace a prolapsed mitral valve. What should the nurse teach the client? 1. The signs and symptoms of pericarditis 2. The possible need for prophylactic antibiotic therapy before dental work 3. That cardiac surgery will have to be done eventually for the other valves 4. That pregnancy and childbirth are too stressful when one has this problem

2. The possible need for prophylactic antibiotic therapy before dental work **Antibiotic therapy before invasive procedures, such as dental work, is often prescribed to prevent endocarditis because these situations may introduce infectious agents systemically.

A nurse plans to set up emergency equipment at the bedside of a client in the immediate postoperative period after a thyroidectomy. What should the nurse include in the bedside setup? 1. Crash cart with bed board 2. Tracheostomy set and oxygen 3. Ampule of sodium bicarbonate 4. Airway and nonrebreather mask

2. Tracheostomy set and oxygen

The laboratory values of a client with renal calculi reveal a serum calcium within expected limits and an elevated serum purine. The nurse concludes that the stone probably is composed of: 1. Cystine 2. Uric acid 3. Calcium oxalate 4. Magnesium ammonium phosphate

2. Uric acid

The nurse determines that which genitourinary factor contributes to urinary incontinence in older adults? 1. Sensory deprivation 2. Urinary tract infection 3. Frequent use of diuretics 4. Inaccessibility of a bathroom

2. Urinary tract infection

A client is hospitalized with joint pain, loss of hair, yellow pigmentation of the skin, and an enlarged liver. The nurse suspects what type of toxicity? 1. Thiamine 2. Vitamin A 3. Vitamin C 4. Pyridoxine

2. Vitamin A **These adaptations, as well as anemia, irritability, pruritus, and an enlarged spleen, occur with vitamin A toxicity.

A client's laboratory report indicates hyperkalemia. Which responses should the nurse expect the client to exhibit? (Select all that apply.) 1. Anorexia 2. Vomiting 3. Constipation 4 . Muscle weakness 5 . Irregular heart rate

2. Vomiting 4 . Muscle weakness 5 . Irregular heart rate

When performing a peritoneal dialysis procedure, the nurse should: 1. Place the client in a side-lying position 2. Warm dialysate solution slightly before instillation 3. Infuse the dialysate solution slowly over several hours 4. Withhold the routine medications until after the procedure

2. Warm dialysate solution slightly before instillation

A client is to have hemodialysis. What must the nurse do before this treatment? 1. Obtain a urine specimen to evaluate kidney function. 2. Weigh the client to establish a baseline for later comparison. 3. Administer medications that are scheduled to be given within the next hour. 4. Explain that the peritoneum serves as a semipermeable membrane to remove wastes.

2. Weigh the client to establish a baseline for later comparison.

After surgical clipping of a cerebral aneurysm, the client develops the syndrome of inappropriate secretion of antidiuretic hormone (ADH). For which manifestations of excessive levels of ADH should the nurse assess the client? (Select all that apply.) 1. Polyuria 2. Weight gain 3. Hypotension 4. Hyponatremia 5. Decreased specific gravity

2. Weight gain 4. Hyponatremia

What clinical indicators should a nurse expect when assessing a client with hyperthyroidism? (Select all that apply.) 1. Dry skin 2. Weight loss 3. Tachycardia 4. Restlessness 5. Constipation 6. Exophthalmos

2. Weight loss 3. Tachycardia 4. Restlessness 6. Exophthalmos

A client is admitted to the hospital for a laparoscopic cholecystectomy. What should the nurse encourage the client to add to the diet to help normalize bowel function after surgery? 1. Vitamins 2. Whole bran 3. Cod liver oil 4. Amino acids

2. Whole bran

A client is admitted for treatment of partial- and full-thickness burns of the entire right lower extremity and the anterior portion of the right upper extremity. Performing an immediate appraisal, using the Rule of Nines, what is the percent of body surface area burned? 1 18% 2 22.5% 3 27% 4 36.5%

22.5%

The nurse is completing an assessment on a couple seeking genetic counseling for sickle cell anemia. Both prospective parents carry sickle cell traits. The nurse recognizes that the couple has what chance of having a child who develops the disease?

25%

A client with multiple sclerosis is informed that this is a chronic, progressive neurological condition. The client asks the nurse, "Will I experience excruciating pain?" What is the nurse's best response?

3 "Pain is not a characteristic symptom of this disease process."

A client with a recent history of sinusitis develops meningitis and demonstrates a positive Brudzinski sign. The priority nursing care is: 1. Controlling intracranial pressure 2. Adding pads to the side of the bed 3. Administering prescribed antibiotics 4. Hydrating the client with hypotonic saline

3 Administering prescribed antibiotics

The nurse auscultates fine crackles in a client who has been in respiratory distress. When the nurse is providing information to the client about crackles, which would be appropriate to include?

3 Crackles are located in the smaller air passages.

A client who has been immobile for a prolonged time develops hypercalcemia. Which findings are consistent with this condition? (Select all that apply.)

3 Deep bone pain 5 Depressed deep tendon reflexes

A nurse is caring for a client who is hospitalized because of injuries sustained in a major automobile collision. As the client is describing the accident to a friend, the client becomes very restless, and his pulse and respirations increase sharply. Which factor probably is related to the client's physical responses?

3 Delayed psychological response to trauma

Twenty-four hours after a penile implant the client's scrotum is edematous and painful. The nurse should:

3 Elevate the scrotum using a soft support

A nurse teaches the signs of organ rejection to a client who had a kidney transplant. What should be included in the education?

3 Elevated blood pressure

Polycythemia frequently is associated with chronic obstructive pulmonary disease (COPD). When assessing for this complication, the nurse should monitor for:

3 Elevated hemoglobin

A client is diagnosed with gastroenteritis. What does the nurse determine is the basic intention underlying the unique dietary management for this client?

3 Eliminate chemical, mechanical, and thermal irritation.

A nurse obtains a health history from a client with the diagnosis of renal calculi. The nurse concludes that the factor that most likely contributed to the calculi development is the client's:

3 Excess ingestion of antacids

A client is receiving furosemide (Lasix) to relieve edema. The nurse should monitor the client for which response to the medication?

3 Excessive loss of potassium ions

A nurse uses the Glasgow Coma Scale to assess a client's status after a head injury. When the nurse applies pressure to the nail bed of a finger, which movement of the client's upper arm should cause the most concern?

3 Extending

An older adult client states, "I walk 2 miles a day for exercise, but now that the weather is hot, I am worried about becoming dehydrated." What should the nurse teach the client?

3 Fluids should be increased if the urine is getting darker.

A client who has been diagnosed with Lyme disease is started on doxycycline (Vibramycin) as part of the therapy. What should the nurse do when administering this drug?

3 Give the medication an hour before milk products are ingested.

A nurse is caring for a client with a diagnosis of acquired immunodeficiency syndrome (AIDS). The IV infiltrates and needs to be restarted. What is necessary to protect the nurse when restarting the IV? (Select all that apply.)

3 Gloves 5 Hand hygiene

A jogger sustains multiple fractures of the femur after being hit by a motor vehicle. A nurse responds to the scene of the accident to assist with care. The nurse recalls that, for this type of fracture, immediate life-threatening systemic complications can be minimized by:

3 Handling and transporting the client gently

A nurse is reviewing the physical examination and laboratory tests of a client with malaria. For which important clinical indicators should the nurse be alert when reviewing data about this client? (Select all that apply.)

3 Hyperthermia 4 Splenomegaly

While on a hike, a rusty nail pierces the sole of an adolescent's foot and the adolescent is brought to the emergency department of a local hospital. Tetanus immune globulin is prescribed because the adolescent does not know when the last tetanus immunization was received. The nurse administers the prescribed dose of tetanus immune globulin and explains that it provides:

3 Immediate passive short-term immunity

During a client's immediate postoperative period after a laryngectomy, a nursing priority is to:

3 Keep the trachea free of secretions

A nurse is caring for a client with severe dyspnea who is receiving oxygen via a Venturi mask. What should the nurse do when caring for this client?

3 Monitor oxygen saturation levels when eating.

The nurse is caring for a client that has a lesion in the right upper lobe. A diagnosis of tuberculosis (TB) has been made. What are the clinical manifestations of tuberculosis?

3 Night sweats and blood-tinged sputum

A client is admitted to the hospital for a surgical resection of the lower left lobe of the lung. After surgery the client has a chest tube to a closed-chest drainage system. What should the nurse do to determine if the chest tube is patent?

3 Observe for fluctuations of the fluid in the water-seal chamber

A client arrives in the emergency department with multiple crushing wounds of the chest, abdomen, and legs. The priority nursing assessments are:

3 Quality of respirations and presence of pulses

A nurse is caring for a client after abdominal surgery and encourages the client to turn from side to side and to engage in deep-breathing exercises. What complication is the nurse trying to prevent?

3 Respiratory acidosis

A client is diagnosed with Parkinson disease and receives a prescription for levodopa (L-dopa) therapy. The nurse concludes that the medication is appropriate for this client because it:

3 Restores the dopamine levels in the brain

A nurse raises three of four of the client's bedside rails at night. Which psychosocial outcome does the nurse hope to achieve through the use of side rails?

3 Support a sense of security

A nurse is caring for a client with complications associated with peritoneal dialysis. For which signs and symptoms should the nurse monitor the client? (Select all that apply.)

3 Tachycardia 4 Cloudy outflow 5 Abdominal pain

A client who is to begin continuous ambulatory peritoneal dialysis (CAPD) asks the nurse what this treatment entails. What information should the nurse include in the explanation?

3 The peritoneal membrane allows passage of toxins into the dialysate.

A nurse working in the health services center of a college is reviewing the vaccination records of a young adult who plans to enroll. Which immunizations are required to meet admission criteria according to the American Academy of Pediatrics?

3 Three doses of diphtheria toxoid and oral poliomyelitis vaccine, and one dose of live measles, live rubella, and mumps vaccine.

When auscultating a client's chest, the nurse hears swishing sounds of normal breathing. The nurse should document:

3 Vesicular breath sounds

A client is diagnosed with herpes genitalis. What should the nurse do to prevent cross-contamination?

3 Wear a gown and gloves when giving direct care.

A client with intermittent claudication has been instructed to stop smoking. The nurse explains that the reason that the client should quit smoking is because: 1. "The policy states that the hospital is a smoke-free environment." 2. "Nicotine causes arteries to go into spasm, which decreases circulation." 3. "Cigarette smoking is not suggested for clients like you that have vascular problems." 4. "The health care provider may allow you to begin smoking again after you are feeling better."

3. "Cigarette smoking is not suggested for clients like you that have vascular problems."

A nurse is reviewing preoperative instructions with a client who is scheduled for orthopedic surgery at 8 AM the next day. The nurse advises the client to: 1. "Have your dinner completed by 6 PM tonight and then no food or fluids after that." 2. "Drink whatever liquids you want tonight and then only clear liquids tomorrow morning." 3. "Consume a light evening meal tonight and then no food or fluids after midnight." 4. "Eat lunch today and then do not drink or eat anything until after your surgery."

3. "Consume a light evening meal tonight and then no food or fluids after midnight."

An insulin pump is instituted for a client with type 1 diabetes. The nurse plans discharge instructions. Which short-term goal is the priority for this client? 1. "Adhere to the medical regimen." 2. "Remain normoglycemic for three weeks." 3. "Demonstrate correct use of the insulin pump." 4. "List three self-care activities that help control the diabetes."

3. "Demonstrate correct use of the insulin pump."

Isoniazid (INH) is prescribed as a prophylactic measure for a client whose spouse has active tuberculosis (TB). What statements by the client indicate that there is a need for further teaching? (Select all that apply.) 1. "I plan to start taking vitamin B6 (NesTrex) with breakfast." 2. "I'll still be taking this drug six months from now." 3. "I sometimes allow our children to sleep in our bed at night." 4. "I know I also have tuberculosis because the skin test was positive." 5. "I'll be skipping the wine but enjoying the cheese at my neighbor's party."

3. "I sometimes allow our children to sleep in our bed at night." 4. "I know I also have tuberculosis because the skin test was positive." 5. "I'll be skipping the wine but enjoying the cheese at my neighbor's party."

A nurse is preparing to discharge a client who had a transurethral prostatectomy for benign prostatic hyperplasia. The nurse evaluates that the client understands the discharge teaching when the client states: 1. "I will drink 6-8 cups of fluid daily and no fluids near bedtime." 2. "Now I don't have to go back to my health care provider's office." 3. "I will use stool softeners regularly for the next one to two months." 4. "I plan to go home and have sexual intercourse with my spouse."

3. "I will use stool softeners regularly for the next one to two months."

Which client should a nurse consider the greatest risk for developing hypernatremia? 1. 52-year-old who is receiving 0.45% NaCl intravenously 2. 76-year-old who developed syndrome of inappropriate antidiurectic hormone secretion (SIADH) as a result of head trauma 3. 63-year-old who has had watery diarrhea since traveling abroad 4. 48-year-old who is admitted with a diagnosis of Addison disease

3. 63-year-old who has had watery diarrhea since traveling abroad

A client has a suspected peptic ulcer in the duodenum. What should the nurse expect the client to report when describing the pain associated with this disease? 1. An ache radiating to the left side 2. An intermittent colicky flank pain 3. A gnawing sensation relieved by food 4. A generalized abdominal pain intensified by moving

3. A gnawing sensation relieved by food

The nurse is providing care to a client who is being treated for bacterial cystitis. Before discharge, it is most important for the client to: 1. Understand the need to drink 4 L of water per day, an essential measure to prevent dehydration 2. Be able to identify dietary restrictions and plan menus 3. Achieve relief of symptoms and to maintain kidney function 4. Recognize signs of bleeding, a complication associated with this type of procedure

3. Achieve relief of symptoms and to maintain kidney function

An older adult client is demonstrating mild confusion after surgical repair of a hernia. What should the nurse do to provide for this client's safety? 1. Use a nightlight in the client's room. 2. Secure a prescription for a soft vest restraint. 3. Activate the position-sensitive bed alarm. 4. Raise the four side rails on the client's bed.

3. Activate the position-sensitive bed alarm.

On the first day after a thyroidectomy, a client tolerates a full-liquid/fluid diet. When the diet is progressed to a soft diet the next day, the client complains of a sore throat when swallowing. How should the nurse respond? 1. Place the client on a full liquid/fluid diet to prevent choking 2. Notify the health care provider immediately 3. Administer prescribed analgesics before meals 4. Assist the client to gargle with saline to moisten mucous membranes

3. Administer prescribed analgesics before meals

During the early postoperative period after a subtotal thyroidectomy, the nursing priority is to assess for: 1. Hemorrhage 2. Thyrotoxic crisis 3. Airway obstruction 4. Hypocalcemic tetany

3. Airway obstruction

A client with heart failure is on a drug regimen of digoxin (Lanoxin) and furosemide (Lasix). The client dislikes oranges and bananas. Which fruit should the nurse encourage the client to eat? 1. Apples 2. Grapes 3. Apricots 4. Cranberries

3. Apricots

For which clinical indicator should a nurse monitor a client with end-stage renal disease? 1. Polyuria 2. Jaundice 3. Azotemia 4. Hypotension

3. Azotemia **Azotemia is an increase in nitrogenous waste (particularly urea) in the blood, which is common with end-stage renal disease.

Which factor identified by the nurse while obtaining the client's health history predisposes a client to type 2 diabetes? 1. Having diabetes insipidus 2. Eating low cholesterol foods 3. Being twenty pounds overweight 4. Drinking a daily alcoholic beverage

3. Being twenty pounds overweight

For which client response should the nurse monitor when assessing for complications of hyperparathyroidism? 1. Tetany 2. Seizures 3. Bone pain 4. Graves disease

3. Bone pain

A client admitted to the hospital in the oliguric phase of acute renal failure estimates that the urine output for the last 12 hours was less than 240 mL. The nurse reviews the plan of care and notes a prescription for 900 mL of water to be given orally over the next 24 hours. The nurse concludes that this amount of fluid was prescribed to: 1. Equal the expected urinary output for the next 24 hours 2. Prevent the development of hypostatic pneumonia and fever 3. Compensate for both insensible and expected output over the next 24 hours 4. Prevent hyperkalemia, which can lead to life-threatening cardiac dysrhythmias

3. Compensate for both insensible and expected output over the next 24 hours **Insensible losses are 400 to 500 mL in 24 hours; the measured output is about 400 mL in 24 hours based on the available history.

A nurse is caring for two clients newly diagnosed with diabetes. One client has type 1 diabetes and the other client has type 2 diabetes. The nurse determines that the main difference between newly diagnosed type 1 and type 2 diabetes is that in type 1 diabetes: 1. Onset of the disease is slow. 2. Excessive weight is a contributing factor. 3. Complications are not present at the time of diagnosis. 4. Treatment involves diet, exercise, and oral medications

3. Complications are not present at the time of diagnosis.

A health care provider prescribes oxygen given in low concentration rather than in high concentration to a client with chronic obstructive pulmonary disease (COPD). What does this prevent? 1. Decrease in red cell formation 2. Rupture of emphysematous bullae 3. Depression of the respiratory center 4. Excessive drying of the respiratory mucosa

3. Depression of the respiratory center

When assessing the oral cavity of a newly admitted client with acquired immunodeficiency syndrome (AIDS), the nurse identifies areas of white plaque on the client's tongue and palate. What is the nurse's initial response? 1. Scrape an area of one of the lesions and send the specimen for a biopsy. 2. Instruct the client to perform meticulous oral hygiene at least once daily. 3. Document the presence of the lesions, describing their size, location, and color. 4. Consider that these lesions are universally found in clients with AIDS and require no treatment

3. Document the presence of the lesions, describing their size, location, and color.

A client with type 2 diabetes is admitted for elective surgery. The health care provider prescribes regular insulin even though oral antidiabetics were adequate before the client's hospitalization. The nurse concludes that regular insulin is needed because the: 1. Client will need a higher serum glucose level while on bed rest 2. Possibility of acidosis is greater when a client is on oral hypoglycemics 3. Dosage can be adjusted to changing needs during recovery from surgery 4. Stress of surgery may precipitate uncontrollable periods of hypoglycaemia

3. Dosage can be adjusted to changing needs during recovery from surgery

A nurse is teaching a client who has pancreatitis about dietary restrictions. What should the education include? 1. Use unsaturated fats 2. Season foods sparingly 3. Eat small meals frequently 4. Limit coffee to three cups per day

3. Eat small meals frequently

A nurse plans to teach the signs of rejection to a client who just had a transplanted kidney. What sign of rejection should the nurse include? 1. Weight loss 2. Subnormal temperature 3. Elevated blood pressure 4. Increased urinary output

3. Elevated blood pressure

Polycythemia frequently is associated with chronic obstructive pulmonary disease (COPD). When assessing for this complication, the nurse should monitor for: 1. Pallor and cyanosis 2. Dyspnea on exertion 3. Elevated hemoglobin 4. Decreased hematocrit

3. Elevated hemoglobin

A client is diagnosed with gastroenteritis. What does the nurse determine is the basic intention underlying the unique dietary management for this client? 1. Provide optimal amounts of all important nutrients. 2. Increase the amount of bulk and roughage in the diet. 3. Eliminate chemical, mechanical, and thermal irritation. 4. Promote psychological support by offering a wide variety of foods.

3. Eliminate chemical, mechanical, and thermal irritation.

A nurse is advising a client about the risks associated with failing to seek treatment for acute pharyngitis caused by beta-hemolytic streptococcus. For what health problem is the client at risk? 1. Asthma 2. Anemia 3. Endocarditis 4. Reye syndrome

3. Endocarditis

A client who is receiving methotrexate for acute lymphocytic leukemia (ALL) develops a temperature of 101° F. The nurse notifies the health care provider. Aspirin 650 mg every four hours as needed for temperature equal to or greater than 101° F is prescribed. What should the nurse do regarding this prescription? 1. Express concern about the dosage prescribed. 2. Request a prescription for an antacid. 3. Express concern about the type of antipyretic prescribed. 4. Ask if the frequency should be every six hours instead.

3. Express concern about the type of antipyretic prescribed.

A client with uremic syndrome has the potential to develop many complications. Which complication should the nurse anticipate? 1. Hypotension 2. Hypokalemia 3. Flapping hand tremors 4. Elevated hematocrit values

3. Flapping hand tremors

Four hours after surgery the blood glucose level of a client who has type 1 diabetes is elevated. The nurse can expect to: 1. Administer an oral hypoglycemic 2. Institute urine glucose monitoring 3. Give supplemental doses of regular insulin 4. Decrease the rate of the intravenous infusion

3. Give supplemental doses of regular insulin

A client who has been diagnosed with Lyme disease is started on doxycycline (Vibramycin) as part of the therapy. What should the nurse do when administering this drug? 1. Administer the medication with meals or a snack. 2. Provide orange or other citrus fruit juice with the medication. 3. Give the medication an hour before milk products are ingested. 4. Offer antacids 30 minutes after administration if gastrointestinal side effects occur.

3. Give the medication an hour before milk products are ingested.

A client with acute glomerulonephritis reports feeling thirsty. What should the nurse offer the client? 1. Ginger ale 2. Milkshake 3. Hard candy 4. Chicken broth

3. Hard candy

A client has a glycosylated hemoglobin measurement of 6%. What should the nurse conclude about this client when planning a teaching plan based on the results of this laboratory test? 1. Is experiencing a rebound hyperglycemia 2. Needs the insulin changed to a different type 3. Has followed the treatment plan as prescribed 4. Requires further teaching regarding nutritional guidelines

3. Has followed the treatment plan as prescribed **The expected range of glycosylated hemoglobin (HbA1c ) is 4.4% to 6.4%. A value of 6% is within the expected range.

During a client's paracentesis, 1500 mL of fluid is removed. The nurse monitors the client for which sign of a potentially severe response? 1. Abdominal girth decrease 2. Mucous membranes becoming drier 3. Heart rate increases from 80 to 135 4. Blood pressure rises from 130/70 to 190/80

3. Heart rate increases from 80 to 135

What response provides evidence that a client with chronic obstructive pulmonary disease (COPD) understands the nurse's instructions about an appropriate breathing technique? 1. Inhales through the mouth. 2. Increases the respiratory rate. 3. Holds each breath for a second at the end of inspiration. 4. Progressively increases the length of the inspiratory phase.

3. Holds each breath for a second at the end of inspiration. *Holding each breath for a second at the end of inspiration allows added time for gaseous exchange at alveolar capillary beds.

A nurse is reviewing the physical examination and laboratory tests of a client with malaria. For which important clinical indicators should the nurse be alert when reviewing data about this client? (Select all that apply.) 1. Polyuria 2. Leukopenia 3. Hyperthermia 4. Splenomegaly 5. Erythrocytosis

3. Hyperthermia 4. Splenomegaly

While on a hike, a rusty nail pierces the sole of an adolescent's foot and the adolescent is brought to the emergency department of a local hospital. Tetanus immune globulin is prescribed because the adolescent does not know when the last tetanus immunization was received. The nurse administers the prescribed dose of tetanus immune globulin and explains that it provides: 1. Lifelong passive immunity 2. Long-lasting active protection 3. Immediate passive short-term immunity 4. Stimulation for the production of antibodies

3. Immediate passive short-term immunity

A client newly diagnosed as having type 1 diabetes is taught to exercise on a regular basis primarily because exercise has been shown to: 1. Decrease insulin sensitivity 2. Stimulate glucagon production 3. Improve the cellular uptake of glucose 4. Reduce metabolic requirements for glucose

3. Improve the cellular uptake of glucose

A nurse is caring for a client with type 1 diabetes, and the health care provider prescribes one tube of glucose gel. What is the primary reason for the administration of glucose gel to this client? 1. Diabetic acidosis 2. Hyperinsulin secretion 3. Insulin-induced hypoglycemia 4. Idiosyncratic reactions to insulin

3. Insulin-induced hypoglycemia

A client experiences difficulty in voiding after an indwelling urinary catheter is removed. The nurse determines that this difficulty most likely is related to: 1. Fluid imbalance 2. Sedentary lifestyle 3. Interruption in previous voiding habits 4. Nervous tension following the procedure

3. Interruption in previous voiding habits

Valsartan (Diovan), an angiotensin II receptor antagonist, is prescribed for a client. For which possible side effects should the nurse monitor the client? (Select all that apply.) 1. Constipation 2. Hypokalemia 3. Irregular pulse rate 4. Change in visual acuity 5 . Orthostatic hypotension

3. Irregular pulse rate 5 . Orthostatic hypotension **Dysrhythmias, including second-degree heart block, are cardiovascular side effects of valsartan .

The nurse is teaching a diabetic client about the advantages of using an insulin pump. What information should the nurse include? (Select all that apply.) 1. It prevents ketoacidosis 2. It helps cause weight loss 3. It can improve A1c levels 4. An insulin pump costs less than subcutaneous injections 5. Clients can exercise without eating more carbohydrates

3. It can improve A1c levels 5. Clients can exercise without eating more carbohydrates

A nurse is counseling a client who has gonorrhea. What additional fact about gonorrhea, besides the fact that it is highly infectious, should the nurse teach this client? 1. It is easily cured. 2. It occurs very rarely. 3. It can produce sterility. 4. It is limited to the external genitalia.

3. It can produce sterility.

A nurse is caring for a client who just had surgery for a parotid tumor. Which nursing intervention is the priority in the immediate postoperative period? 1. Offering psychological support 2. Monitoring the client's fluid balance 3. Keeping the client's respiratory passages patent 4. Providing a pad and pencil for writing messages

3. Keeping the client's respiratory passages patent

A client is admitted to the hospital with a long history of hypertension. The nurse should assess the client for which complication? 1. Cataracts 2. Esophagitis 3. Kidney failure 4. Diabetes mellitus

3. Kidney failure

The nurse provides education to a client with cancer of the tongue who will begin gastrostomy feedings at home. The nurse concludes that the teaching was effective when the client states, "Before I start the procedure, I will: 1. Don sterile gloves" 2. Obtain my body weight" 3. Measure the residual volume" 4. Instill one ounce of a carbonated liquid"

3. Measure the residual volume"

Which medication should the nurse question when it is prescribed for a client with acute pancreatitis? 1. Ranitidine (Zantac) 2. Cimetidine (Tagamet) 3. Meperidine (Demerol) 4. Promethazine HCl (Phenergan)

3. Meperidine (Demerol)

A nurse is caring for a client after radioactive iodine is administered for Graves disease. What information about the client's condition after this therapy should the nurse consider when providing care? 1. Not radioactive and can be handled as any other individual 2. Highly radioactive and should be isolated as much as possible 3. Mildly radioactive but should be treated with routine safety precautions 4. Not radioactive but may still transmit some dangerous radiations and must be treated with precautions

3. Mildly radioactive but should be treated with routine safety precautions

A client is admitted to the hospital for a surgical resection of the lower left lobe of the lung. After surgery the client has a chest tube to a closed-chest drainage system. What should the nurse do to determine if the chest tube is patent? 1. Milk the chest tube toward the drainage unit 2. Check the amount of bubbling in the suction control chamber 3. Observe for fluctuations of the fluid in the water-seal chamber 4. Assess for extent of chest expansion in relation to breath sounds

3. Observe for fluctuations of the fluid in the water-seal chamber *Fluctuations of the fluid in the water-seal chamber indicate effective communication between the pleural cavity and the drainage system.

A client scheduled for surgery has a history of methicillin-resistant Staphylococcus aureus (MRSA) since developing an infection in a surgical site nine months ago. The site is healed and the client reports having received antibiotics for the infection. What should the nurse do to determine if the infecting organism is still present? 1. Notify the infection control officer 2. Inform the operating room of the MRSA 3. Obtain a prescription to culture the client's blood 4. Call the surgeon for an infectious disease consultation

3. Obtain a prescription to culture the client's blood

An older African-American client with hypertension is admitted to the hospital. Which data from the client's history and diagnostic workup represent risk factors for hypertension? (Select all that apply.) 1. Increased high-density lipoprotein (HDL) 2. Taking an aspirin a day 3. Occasional cocaine use 4. Reduced hemoglobin level 5. African-American heritage

3. Occasional cocaine use 5. African-American heritage

A nurse is assessing a client with the diagnosis of primary hypertension. What clinical finding does the nurse identify as an indicator of primary hypertension? 1. Mild but persistent depression 2. Transient temporary memory loss 3. Occipital headache in the morning 4. Cardiac palpitation during periods of stress

3. Occipital headache in the morning

What criteria should the nurse consider when determining if an infection should be categorized as a health care-associated infection? 1. Originated primarily from an exogenous source. 2. Is associated with a drug resistant microorganism. 3. Occurred in conjunction with treatment for an illness. 4. Still has the infection despite completing the prescribed therapy

3. Occurred in conjunction with treatment for an illness.

What criteria should the nurse consider when determining if an infection should be categorized as a health care-associated infection? 1. Originated primarily from an exogenous source. 2. Is associated with a drug resistant microorganism. 3. Occurred in conjunction with treatment for an illness. 4. Still has the infection despite completing the prescribed therapy.

3. Occurred in conjunction with treatment for an illness.

A nurse is monitoring a client with renal failure for signs of fluid excess. Which finding does the nurse identify as inconsistent with fluid excess? 1. Increased weight 2. Distended neck veins 3. Orthostatic hypotension 4. Abnormal breath sounds

3. Orthostatic hypotension **Hypertension, not hypotension, is an indicator of fluid volume excess.

A client is admitted to the hospital for an emergency cardiac catheterization. What is the most common adaptation that the client is most likely to complain about after this procedure? 1. Fear of dying 2. Skipped heartbeats 3. Pain at the insertion site 4. Anxiety in response to intensive monitoring

3. Pain at the insertion site

During a routine physical examination, an abdominal aortic aneurysm is diagnosed. The client immediately is admitted to the hospital, and surgery is scheduled for the next morning. Which clinical finding should the nurse expect when performing an assessment of this client? 1 .Severe radiating abdominal pain 2. Pattern of visible peristaltic waves 3. Palpable pulsating abdominal mass 4 . Cyanosis with other symptoms of shock

3. Palpable pulsating abdominal mass

A client has circumgastric banding, a bariatric surgical procedure. The nurse provides discharge teaching about signs and symptoms of dumping syndrome and includes what physiological response? 1. Fever 2. Vomiting 3. Palpitations 4. Constipation

3. Palpitations

When caring for a client who is recovering from a gastrectomy, a nurse is concerned about the potential development of pernicious anemia. What should the nurse conclude may be the cause of this complication? 1. Vitamin B12 is just absorbed in the stomach. 2. Hemopoietic factor is secreted in the stomach. 3. Parietal cells of the stomach secrete the intrinsic factor. 4. Chief cells in the stomach promote the secretion of the extrinsic factor.

3. Parietal cells of the stomach secrete the intrinsic factor.

The left foot of a client with a history of intermittent claudication becomes increasingly cyanotic and numb. Gangrene of the left foot is diagnosed, and because of the high level of arterial insufficiency, an above-the-knee amputation (AKA) is scheduled. The response that demonstrates emotional readiness for the surgery is when the client: 1. Explains the goals of the procedure 2. Displays few signs of anticipatory grief 3. Participates in learning perioperative care 4. Verbalizes acceptance of future dependency needs

3. Participates in learning perioperative care

A male client who is receiving prolonged steroid therapy complains of always being thirsty and urinating frequently. What is the nurse's best initial action? 1. Have the client assessed for an enlarged prostate. 2. Obtain a urine specimen from the client to test for ketonuria. 3. Perform a finger stick to test the client's blood glucose level. 4. Assess the client's lower extremities for the presence of pitting edema.

3. Perform a finger stick to test the client's blood glucose level.

Blood studies are being performed on a client with the potential diagnosis of hyperparathyroidism. What serum blood level should the nurse expect to be decreased when reviewing this client's hematological studies? 1. Calcium 2. Chloride 3. Phosphorus 4. Parathormone

3. Phosphorus *Because of its inverse relationship with calcium, when serum calcium levels increase, serum phosphorous levels decrease (greater than 3 mg/dL; greater than 0.1 mmol/L).

Which relationship does the nurse consider reflective of the relationship of naloxone (Narcan) to morphine sulfate? 1. Aspirin to warfarin (Coumadin) 2. Amoxicillin (Amoxil) to systemic infection 3. Protamine sulfate to parenteral heparin 4. Enoxaparin (Lovenox) to dalteparin (Fragmin)

3. Protamine sulfate to parenteral heparin **Protamine sulfate is the antidote for heparin overdose and naloxone will reverse the effects of opioids such as morphine.

A client with a hiatal hernia comes to the community health clinic to attend a class about nutrition. The client reports frequently waking up at night with heartburn. Which suggestion by the nurse may help to reduce symptoms of heartburn? 1. Eat a large meal at noontime 2. Take an intestinal sedative at night 3. Raise the head of the bed on blocks 4. Have a light snack with orange juice

3. Raise the head of the bed on blocks **Elevating the head of the bed helps prevent reflux of gastric contents into the esophagus, minimizing heartburn.

An unresponsive older adult is admitted to the emergency department on a hot, humid day. The initial nursing assessment reveals hot, dry skin, a respiratory rate of 36 breaths/min, and a heart rate of 128 beats/min. What is the initial nursing action? 1. Offer cool fluids. 2. Suction the airway. 3. Remove the clothing. 4. Prepare for intubation

3. Remove the clothing.

A client has edema in the lower extremities during the day, which disappears at night. With which medical problem does the nurse conclude this clinical finding is consistent? 1. Pulmonary edema 2. Myocardial infarction 3. Right ventricular heart failure 4. Chronic obstructive lung disease

3. Right ventricular heart failure

What data about the fluid in the water-seal chamber of a closed chest drainage system provide support for the nurse's conclusion that the system is functioning correctly? 1. Contains many small air bubbles. 2. Bubbles vigorously on inspiration. 3. Rises with inspiration and falls with expiration. 4. Remains at a consistent level during the respiratory cycle

3. Rises with inspiration and falls with expiration. *During inspiration, negative pressure in the pleural space increases, causing fluid to rise in the chamber; during expiration, negative pressure in the pleural space decreases, causing fluid to drop in the chamber.

The menu for a client with malabsorption syndrome must be limited because of a sensitivity to gluten. Which foods cannot be served to this client? (Select all that apply.) 1. Cheese omelet 2. Creamed spinach 3. Roast beef sandwich 4. Chicken noodle soup 5. Spaghetti and meatballs

3. Roast beef sandwich 4. Chicken noodle soup 5. Spaghetti and meatballs **Bread contains gluten, which is irritating to the gastrointestinal mucosa and should be avoided. Noodles are made from flour and are high in gluten, which is irritating to the gastrointestinal mucosa and should be avoided. Pasta is made from flour and is high in gluten, which is irritating to the gastrointestinal mucosa and should be avoided.

A nurse teaches a client with chronic renal failure that salt substitutes cannot be used in the diet because: 1. A person's body tends to retain fluid when a salt substitute is included in the diet. 2. Limiting salt substitutes in the diet prevents a buildup of waste products in the blood. 3. Salt substitutes contain potassium, which must be limited to prevent abnormal heartbeats. 4. A substance in the salt substitute interferes with the transfer of fluid across capillary membranes, resulting in anasarca.

3. Salt substitutes contain potassium, which must be limited to prevent abnormal heartbeats.

A client, complaining of fatigue, is admitted to the hospital with a diagnosis of chronic obstructive pulmonary disease (COPD). To prevent fatigue, the nurse should: 1. Provide small, frequent meals 2. Encourage pursed-lip breathing 3. Schedule nursing activities to allow for rest 4. Encourage bed rest until energy level improves

3. Schedule nursing activities to allow for rest

A client admitted to the hospital with a diagnosis of chronic obstructive pulmonary disease (COPD) has received a prescription for a medication that is delivered via a nebulizer. When teaching about use of the nebulizer, the nurse should teach the client to: 1. Hold the breath while spraying the medication into the mouth 2. Position the lips loosely around the mouthpiece and take rapid, shallow breaths 3. Seal the lips around the mouthpiece and breathe in and out taking slow, deep breaths 4. Inhale the medication from the nebulizer, remove the mouthpiece from the mouth, and then exhale

3. Seal the lips around the mouthpiece and breathe in and out taking slow, deep breaths *Sealing the lips around the mouthpiece ensures that medication is delivered on inspiration; slow, deep breaths promote better deposition and efficacy of medication deep into the lungs.

When providing discharge teaching for a young female client who had a pneumothorax, it is important that the nurse include the signs and symptoms of a recurring pneumothorax. What is the most important symptom that the nurse should teach the client to report to the healthcare provider? 1. Substernal chest pain 2. Episodes of palpitation 3. Severe shortness of breath 4. Dizziness when standing up

3. Severe shortness of breath

A client with a 40-year history of drinking two alcoholic beverages and smoking two packs of cigarettes daily comes to the outpatient clinic with an ischemic left foot. It is determined that the cause is arterial insufficiency. The nurse concludes that the pain in the client's foot is a result of inadequate blood supply, which may be diminished further by: 1. Drinking alcohol 2. Lowering the limb 3. Smoking cigarettes 4. Consuming excessive fluid

3. Smoking cigarettes

A nurse is caring for a client with complications associated with peritoneal dialysis. For which signs and symptoms should the nurse monitor the client? (Select all that apply.) 1 . Pruritus 2. Oliguria 3. Tachycardia 4. Cloudy outflow 5. Abdominal pain

3. Tachycardia 4. Cloudy outflow 5. Abdominal pain

The nurse cares for a client who develops pyrexia three days after surgery. The nurse should monitor the client for which signs and symptoms commonly associated with pyrexia? (Select all that apply.) 1. Dyspnea 2. Chest pain 3. Tachypnea 4. Increased pulse rate 5. Elevated blood pressure

3. Tachypnea 4. Increased pulse rate

After abdominal surgery a client is to receive a progressive postsurgical diet. The nurse explains to the client that this diet is characterized by progressive alterations in the: 1. Caloric content of food 2. Nutritional value of food 3. Texture and digestibility of food 4. Variety of fluids and food

3. Texture and digestibility of food

A client who is to begin continuous ambulatory peritoneal dialysis (CAPD) asks the nurse what this treatment entails. What information should the nurse include in the explanation? 1. Peritoneal dialysis is done in an ambulatory care clinic. 2. Hemodialysis and peritoneal dialysis are provided continuously. 3. The peritoneal membrane allows passage of toxins into the dialysate. 4. A quarter of a liter of dialysate is maintained inter- and intraperitoneally.

3. The peritoneal membrane allows passage of toxins into the dialysate.

A client who has repeated episodes of cystitis is scheduled for a cystoscopy to determine the possibility of urinary tract abnormalities. The client asks the nurse to describe the procedure. The nurse's most appropriate response is, "This procedure is: 1. A computerized scan that outlines the bladder and surrounding tissue." 2. An x-ray film of the abdomen, kidneys, ureters, and 3. The visualization of the inside of the bladder with an instrument connected to a source of light." 4. The visualization of the urinary tract through ureteral catheterization and the use of radiopaque material."

3. The visualization of the inside of the bladder with an instrument connected to a source of light."

A client who has had a subtotal thyroidectomy does not understand how hypothyroidism can develop when the problem was initially hyperthyroidism. The nurse bases a response on the fact that: 1. Hypothyroidism is a gradual slowing of the body's function 2. There will be a decrease in pituitary thyroid-stimulating hormone (TSH) 3. There may not be enough thyroid tissue to supply adequate thyroid hormone 4. Atrophy of tissue remaining after surgery reduces secretion of thyroid hormones

3. There may not be enough thyroid tissue to supply adequate thyroid hormone

A nurse providing care to a client who had major abdominal surgery monitors the client for postoperative complications. Which clinical findings are indicators of impending hypovolemic shock? 1. Diuresis, irritability, and fever 2. Lethargy, cold skin, and hypertension 3. Thirst, cool skin, and orthostatic hypotension 4. Bounding pulse, restlessness, and slurred speech

3. Thirst, cool skin, and orthostatic hypotension

A nurse is caring for a client with acute kidney failure who is receiving a protein-restricted diet. The client asks why this diet is necessary. What information should the nurse include in a response to the client's questions? 1. A high-protein intake ensures an adequate daily supply of amino acids to compensate for losses. 2. Essential and nonessential amino acids are necessary in the diet to supply materials for tissue protein synthesis. 3. This supplies only essential amino acids, reducing the amount of metabolic waste products, thus decreasing stress on the kidneys. 4. Urea nitrogen cannot be used to synthesize amino acids in the body, so the nitrogen for amino acid synthesis must come from the dietary protein.

3. This supplies only essential amino acids, reducing the amount of metabolic waste products, thus decreasing stress on the kidneys.

A client with cancer of the thyroid is scheduled for a thyroidectomy. What should the nurse teach the client? 1. The dietary intake of carbohydrates must be restricted. 2. Chemotherapy may be used in conjunction with the surgery. 3. Thyroxine replacement therapy will be required indefinitely. 4. A tracheostomy requires an alternate means of communication.

3. Thyroxine replacement therapy will be required indefinitely.

A nurse is caring for a client who has been taking several antibiotic medications for a prolonged time. Because long-term use of antibiotics interferes with the absorption of fat, the nurse anticipates a prescription for: 1. High fat diet 2. Supplemental cod liver oil 3. Total parenteral nutrition (TPN) 4. Water-soluble forms of vitamins A and E

3. Total parenteral nutrition (TPN)

A nurse is planning to administer a prescribed intravenous solution that contains potassium chloride. What assessment should be brought to the health care provider's attention before administration of the intravenous (IV) line? 1. Uncharacteristic irritability 2. Poor tissue turgor with tenting 3. Urinary output of 200 mL during the previous 8 hours 4. Oral fluid intake of 300 mL during the previous 12 hours

3. Urinary output of 200 mL during the previous 8 hours

A client has been in a coma for two months and is maintained on bed rest. At what angle should the nurse adjust the head of the bed to prevent the effects of shearing force?

30 degrees

A client who has been in a coma for two months is being maintained on bed rest. The nurse concludes that to prevent the effects of shearing force, the head of the bed should be maintained at an angle of:

30 degrees

After surgery a client is to receive an antibiotic by intravenous (IV) piggyback in 50 mL of a diluent. The piggyback is to infuse in 20 minutes. The drop factor of the IV set is 15 gtts/mL. The nurse should set the piggyback to flow at how many gtts/min? Record your answer using a whole number. __________ gtts/min

38 gtts/min is the flow rate to deliver the correct amount of medication. Solve the problem by dividing the total number of drops (50 x 15 = 750) by the total number of minutes (20); because a half drop cannot be administered, the answer must be rounded up to the next number.

A client has been diagnosed with hyperthyroidism. The nurse expects the client to exhibit which clinical manifestations? (Select all that apply.) 1. Dry skin 2. Slow pulse 3. Weight gain 4 . Nervousness 5 . Increased appetite

4 . Nervousness 5 . Increased appetite

To facilitate micturition in a male client, the nurse should instruct him to:

4 Assume the standing position for voiding

A nurse is caring for a client who sustained a transection of the spinal cord. The nurse continually monitors this client for what medical emergency?

4 Autonomic hyperreflexia

A client with the diagnosis of chronic kidney disease develops hypocalcemia. Which clinical manifestations should the nurse expect the client with hypocalcemia to exhibit? (Select all that apply.)

4 Chvostek sign 5 Muscle cramps

Which nursing action can best prevent infection from a urinary retention catheter?

4 Cleansing around the meatus routinely

When a client who had an above-the-knee amputation (AKA) complains of phantom limb sensations, the nursing staff should:

4 Describe the neurological mechanisms in language that the client understands

A client who has been told she needs a hysterectomy for cervical cancer is upset about being unable to have a third child. What is the next nursing action?

4 Ensure that other treatment options for her will be explored.

A nurse is preparing to insert a nasogastric tube. During insertion, which response indicates that the client is experiencing difficulty?

4 Inability to speak

A client with bronchial pneumonia is having difficulty maintaining airway clearance because of retained thick secretions. To decrease the amount of secretions retained, the nurse plans to:

4 Increase fluid intake to at least 2 L a day

The nurse reinforces instructions about how to use a nebulizer to a client with chronic obstructive pulmonary disease (COPD). The nurse concludes that additional teaching is needed when the client:

4 Inhales with the lips tightly sealed around the mouthpiece of the nebulizer

The nurse providing postoperative care for a client who had kidney surgery reviews the client's urinalysis results. The nurse concludes that the presence of what substance in the urine needs to be reported to the health care provider?

4 Large proteins

A nurse is caring for a client who just had an above-the-knee amputation. What is the key factor in assessing how the client will cope with the body image change?

4 Personal perception of the change

A nurse is teaching a preoperative client about postoperative breathing exercises. What information should the nurse include? (Select all that apply.)

4 Place a hand on the abdomen while feeling it rise 5 Hold the breath for several seconds at the height of inspiration

A nurse is reviewing the laboratory reports of a client with a diagnosis of end-stage renal disease. What test result should the nurse anticipate?

4 Potassium of 6.3 mEq/L

Clients are encouraged to perform deep-breathing exercises after most types of surgery. The nurse teaches clients that the reason for these exercises is to help:

4 Prevent the buildup of carbon dioxide in the body

What should the nurse expect when assessing a client with pleural effusion?

4 Reduced or absent breath sounds at the base of the lung

A nurse works with a large population of immigrant clients and is concerned about the debilitating effects of influenza. Which action is the first line of defense against an emerging influenza pandemic?

4 Reporting surveillance findings to appropriate public health officials

A client is admitted to the hospital with a diagnosis of acute Guillain-Barré syndrome. What frequent assessment does the nurse determine is most important for a client with this syndrome?

4 Respiratory exchange

A client has a chest tube inserted to treat a right hemopneumothorax. In which position should the nurse place the client to facilitate chest drainage?

4 Right side-lying

A skier skied off the marked trail into the woods and collided with a tree. After several hours, the skier was found by the ski patrol and brought to the emergency department of the hospital. Moderate hypothermia (temperature range 87° to 90° F) is diagnosed. What clinical findings specific to moderate hypothermia should the nurse expect the client to exhibit?

4 Rigidity and slowed respiratory rate

A nurse is caring for a client with glomerulonephritis. What should the nurse instruct the client to do to prevent recurrent attacks?

4 Seek early treatment for respiratory tract infections.

A client with emphysema has a history of smoking two packs of cigarettes a day. What is the best approach for the nurse to help the client stop smoking?

4 Suggest that the client limit smoking to one pack of cigarettes a day

A lithotripsy to break up renal calculi is unsuccessful, and a nephrolithotomy is performed. Which postoperative clinical indicator should the nurse report to the health care provider?

4 Urine output of 20 to 30 mL/hr

When taking the blood pressure of a client who has acquired immunodeficiency syndrome (AIDS), the nurse must:

4 Wash the hands thoroughly

A client with type 2 diabetes travels frequently and asks how to plan meals during trips. The nurse's most appropriate response is: 1. "You can order diabetic foods on most airlines and in restaurants." 2. "Plan your food ahead and carry it with you from home." 3. "Monitor your blood glucose level frequently and eat accordingly." 4. "Choose the foods you normally do and follow your food plan wherever you are."

4. "Choose the foods you normally do and follow your food plan wherever you are."

The nurse provides education to a client about the side effects of furosemide (Lasix). Which client statements indicate that the teaching is understood? (Select all that apply.) 1. "I must not eat citrus fruits." 2 . "I should wear dark glasses." 3 . "I should avoid lying flat in bed." 4. "I should change my position slowly." 5. "I must eat a food that contains potassium every day."

4. "I should change my position slowly." 5. "I must eat a food that contains potassium every day."

A 40-year-old client scheduled for a hemi-colectomy because of ulcerative colitis asks if having a hemi-colectomy means wearing a pouch and having bowel movements in an abnormal way. Which is the best response by the nurse? 1. "Yes, hemi-colectomy is the same as a colostomy." 2. "Yes, but it will be temporary until the colitis is cured." 3. "No, that is necessary when a tumor is blocking the rectum." 4. "No, only part of the colon is removed and the rest reattached."

4. "No, only part of the colon is removed and the rest reattached."

A client with chronic kidney disease is admitted to the hospital with severe infection and anemia. The client is depressed and irritable. The client's spouse asks the nurse about the anticipated plan of care. What is an appropriate nursing response? 1. "The staff will provide total care because the infection causes severe fatigue." 2. "Mood elevators will be prescribed to improve depression and irritability." 3. "Iron will be prescribed for the anemia and the stools will be dark." 4. "The intake of meat, eggs, and cheese will be restricted so the kidneys can clear the body of waste products."

4. "The intake of meat, eggs, and cheese will be restricted so the kidneys can clear the body of waste products."

Ampicillin 250 mg by mouth every six hours is prescribed for a client who is to be discharged. Which statement indicates to the nurse that the client understands the teaching about ampicillin? 1. "I should drink a glass of milk with each pill." 2. "I should drink at least six glasses of water every day." 3. "The medicine should be taken with meals and at bedtime." 4. "The medicine should be taken one hour before or two hours after meals."

4. "The medicine should be taken one hour before or two hours after meals."

Before a transurethral resection of the prostate (TURP), a client asks about what to expect postoperatively. The most appropriate response by the nurse is: 1. "You will have an abdominal incision and a dressing." 2. "Your urine will be pink and free of clots." 3. "There will be an incision between your scrotum and rectum." 4. "There will be an indwelling urinary catheter and a continuous bladder irrigation in place."

4. "There will be an indwelling urinary catheter and a continuous bladder irrigation in place."

A client who was diagnosed recently with type 1 diabetes states, "I feel bad. I don't think I even want to go home. My spouse doesn't care about my diabetes." What is the most appropriate nursing response? 1. "What can I do to make you feel better?" 2. "It seems that you don't get along with your spouse." 3. "It's probably temporary. Your spouse needs more time to adjust." 4. "You are unhappy. Have you tried to talk with your spouse?"

4. "You are unhappy. Have you tried to talk with your spouse?"

A nurse is monitoring a client's laboratory results for a fasting plasma glucose level. Within which range of a fasting plasma glucose level does the nurse conclude that a client is considered to be diabetic? 1. 40 and 60 mg/dL 2. 80 and 99 mg/dL 3. 100 and 125 mg/dL 4. 126 and 140 mg/dL

4. 126 and 140 mg/dL

A client complains of left-sided chest pain after the client finished playing racquetball. The client is hospitalized and diagnosed with left pneumothorax. When assessing the client's left chest area, the nurse expects to identify: 1. Dull sound on percussion 2. Vocal fremitus on palpation 3. Rales with rhonchi on auscultation 4. Absence of breath sounds on auscultation

4. Absence of breath sounds on auscultation

What does the nurse determine is the most likely cause of renal calculi in clients with paraplegia? 1. High fluid intake 2. Increased intake of calcium 3. Inadequate kidney function 4. Accelerated bone demineralization

4. Accelerated bone demineralization

A health care provider prescribes enoxaprarin (Lovenox) 30 mg subcutaneously daily. To ensure client safety, which measure would the nurse take when administering this medication? 1. Remove air pocket from prepackaged syringe before administration 2. Rub site after administration 3. Push over two minutes 4. Administer in the abdomen

4. Administer in the abdomen

A client with acute kidney failure becomes confused and irritable. Which does the nurse determine is the most likely cause of this behavior? 1. Hyperkalemia 2. Hypernatremia 3. A limited fluid intake 4. An increased blood urea nitrogen level

4. An increased blood urea nitrogen level

A client had a thyroidectomy. The nurse monitors for thyrotoxic crisis, which is evidenced by: 1. An increased pulse deficit 2. A decreased blood pressure 3. A decreased heart rate and respirations 4. An increased temperature and pulse rate

4. An increased temperature and pulse rate

A client diagnosed with gastroesophageal reflux disease (GERD) is being treated with antacid therapy. When teaching the client about the therapy, the nurse reinforces that antacid tablets: 1. Are as effective as the liquid form 2. Should be taken one hour before meals 3. Should be taken only at four-hour intervals 4. Are known to interfere with the absorption of other drugs

4. Are known to interfere with the absorption of other drugs

During the postoperative period after surgery for a kidney transplant, the client's creatinine level is 3.1 mg/dL. What should the nurse do first in response to this laboratory result? 1. Notify the health care provider. 2. Check the intravenous (IV) infusion. 3. Obtain current blood test results. 4. Assess for decreased urine output.

4. Assess for decreased urine output.

A client had surgery for a ruptured appendix. Postoperatively, the health care provider prescribes an antibiotic to be administered intravenously twice a day. The nurse administers the prescribed antibiotic via a secondary line into the primary infusion of 0.9% sodium chloride. During the administration of the antibiotic, the client becomes restless and flushed, and begins to wheeze. What should the nurse do after stopping the antibiotic infusion? 1. Check the client's temperature 2. Take the client's blood pressure 3. Obtain the client's pulse oximetry 4. Assess the client's respiratory status

4. Assess the client's respiratory status

A client is admitted to the hospital with a tentative diagnosis of pneumonia. The client has a high fever and is short of breath. Bed rest, oxygen via nasal cannula, an intravenous antibiotic, and blood and sputum specimens for culture and sensitivity (C&S) are prescribed. Place these interventions in the order in which they should be implemented. Incorrect 1. Specimens for C&S 2. Oxygen via nasal cannula 3. Administration of an antibiotic 4. Bed rest

4. Bed rest 2. Oxygen via nasal cannula 1. Specimens for C&S 3. Administration of an antibiotic

A client's laboratory values demonstrate an increased serum calcium level, and further diagnostic tests reveal hyperparathyroidism. For what clinical manifestations should the nurse assess this client? (Select all that apply.) 1. Muscle tremors 2. Abdominal cramps 3. Increased peristalsis 4. Cardiac dysrhythmias 5. Hypoactive bowel sounds

4. Cardiac dysrhythmias 5. Hypoactive bowel sounds

A client just had a suprapubic prostatectomy. Which action should the nurse implement to prevent a secondary bladder infection? 1. Observe for signs of uremia 2. Attach the catheter to suction 3. Clamp off the connecting tube 4. Change the dressings frequently

4. Change the dressings frequently **After a suprapubic prostatectomy, leakage of urine generally is identified around the suprapubic tube; this creates an environment in which bacteria can flourish if the dressing is not changed frequently.

Which nursing action can best prevent infection from a urinary retention catheter? 1. Cleansing the perineum 2. Encouraging adequate fluids 3. Irrigating the catheter once daily 4. Cleansing around the meatus routinely

4. Cleansing around the meatus routinely

A nurse is administering an enema to a client who is scheduled for gastrointestinal surgery. What should the nurse do when the client complains of abdominal cramps during the enema? 1. Reduce the rate of flow of the infusion. 2. Discontinue the enema and try again later. 3. Lower the container below the level of the rectum. 4. Close the lumen of the tubing and wait until the discomfort subsides.

4. Close the lumen of the tubing and wait until the discomfort subsides.

After a successful kidney transplant for a client with end-stage kidney disease, the nurse anticipates that laboratory studies will demonstrate: 1. Increased specific gravity 2. Correction of hypotension 3. Elevated serum potassium 4. Decreasing serum creatinine

4. Decreasing serum creatinine

What breathing exercises should the nurse teach a client with the diagnosis of emphysema? 1. An inhalation that is prolonged to promote gas exchange. 2. Abdominal exercises to limit the use of accessory muscles. 3. Sit-ups to help strengthen the accessory muscles of respiration. 4. Diaphragmatic exercises to improve contraction of the diaphragm

4. Diaphragmatic exercises to improve contraction of the diaphragm

A client is scheduled for a barium swallow. How should the nurse prepare the client for this test? (Select all that apply.) 1. Ask about allergies to iodine before the test. 2. Administer cleansing enemas before the test. 3. Suggest a light breakfast on the day of the test. 4. Ensure that a laxative is prescribed after the test. 5. Instruct to withhold prescribed opioids for one day before the test.

4. Ensure that a laxative is prescribed after the test. 5. Instruct to withhold prescribed opioids for one day before the test.

A client comes to the clinic for a physical and asks to be tested for acquired immunodeficiency syndrome (AIDS). Which test should the nurse explain will be used for the initial screening for AIDS? 1. CD4 T cell count 2. Western blot test 3. Polymerase chain reaction test 4. Enzyme-linked immunosorbent assay (ELISA)

4. Enzyme-linked immunosorbent assay (ELISA) **The ELISA is the first screening test done to detect serum antibodies that bind to human immunodeficiency virus (HIV) antigens on test plates.

A nurse is caring for a client with Addison disease. Which information should the nurse include in a teaching plan as a means of encouraging this client to modify dietary intake? 1. Increased amounts of potassium are needed to replace renal losses. 2. Increased protein is needed to heal the adrenal tissue and thus cure the disease. 3. Supplemental vitamins are needed to supply energy and assist in regaining the lost weight. 4. Extra salt is needed to replace the amount being lost caused by lack of sufficient aldosterone to conserve sodium.

4. Extra salt is needed to replace the amount being lost caused by lack of sufficient aldosterone to conserve sodium.

A nurse is caring for a client who had surgery for cancer of the pancreas. Which nutrients are most influenced by the effects of this surgery? 1. Proteins and grains 2. Vitamins and minerals 3. Fluids and electrolytes 4. Fats and carbohydrates

4. Fats and carbohydrates

The nurse assesses a male client with a preliminary diagnosis of cancer of the urinary bladder. The nurse recalls that which sign or symptom is a common early sign of cancer of the urinary system: 1. Dysuria 2. Retention 3. Hesitancy 4. Hematuria

4. Hematuria

A client is admitted to the hospital with the diagnosis of cancer of the thyroid and a thyroidectomy is scheduled. What is important for the nurse to consider when caring for this client during the postoperative period? 1. Hypercalcemia may result from parathyroid damage. 2. Hypotension and bradycardia may result from thyroid storm. 3. Tetany may result from underdosage of thyroid hormone replacement. 4. Hoarseness and airway obstruction may result from laryngeal nerve damage.

4. Hoarseness and airway obstruction may result from laryngeal nerve damage.

A client with bronchial pneumonia is having difficulty maintaining airway clearance because of retained thick secretions. To decrease the amount of secretions retained, the nurse plans to: 1. Administer continuous oxygen 2. Instruct the client to gargle deep in the throat using warmed normal saline 3. Place the client in a high Fowler position 4. Increase fluid intake to at least 2 L a day

4. Increase fluid intake to at least 2 L a day *Increased fluid intake helps to liquefy respiratory secretions, which promotes expectoration.

A client with chronic obstructive pulmonary disease has increased hemoglobin and hematocrit levels. The nurse concludes that the altered blood levels are caused by: 1. Increased leukocyte development in response to infection 2. Decreased extracellular fluid volume secondary to infection 3. Decreased red blood cell proliferation because of hypercapnia 4. Increased erythrocyte production as a result of chronic hypoxia

4. Increased erythrocyte production as a result of chronic hypoxia *Hypoxia stimulates production of large quantities of erythrocytes in an attempt to compensate for the lack of oxygen. White blood cell production increases with infection; infection is not the cause of the increase in the hemoglobin and hematocrit.

The nurse provides instructions to a client who will be using an incentive spirometer postoperatively. During the client's return demonstration, the nurse concludes that the teaching has been effective when the client: 1. Coughs twice before inhaling deeply through the mouthpiece 2. Uses the incentive spirometer for 10 consecutive breaths an hour 3. Inhales deeply, seals the lips around the mouthpiece, and then exhales 4. Inhales deeply through the mouthpiece, holds the breath for two seconds, and then exhales

4. Inhales deeply through the mouthpiece, holds the breath for two seconds, and then exhales

The nurse reinforces instructions about how to use a nebulizer to a client with chronic obstructive pulmonary disease (COPD). The nurse concludes that additional teaching is needed when the client: 1. Places the tip of the mouthpiece an inch beyond the lips 2. Holds the inspired breath for at least three seconds 3. Exhales slowly through the mouth with lips pursed slightly 4. Inhales with the lips tightly sealed around the mouthpiece of the nebulizer

4. Inhales with the lips tightly sealed around the mouthpiece of the nebulizer

An older client tells the nurse, "My legs begin to hurt after walking the dog for several blocks. The pain goes away when I stop walking, but it comes back again when I resume walking." Which condition does the nurse consider as the most likely cause of the client's responses? 1. Spinal stenosis 2. Buerger disease 3. Rheumatoid arthritis 4. Intermittent claudication

4. Intermittent claudication **Pain that develops during exercise is a classic symptom of peripheral arterial occlusive disease; arterial occlusion prevents adequate blood flow to the muscles of the legs, causing ischemia and pain.

A client who recently immigrated to the United States has a chronic vitamin A deficiency. What information about vitamin A should the nurse consider when assessing this client for clinical indicators of this deficiency? 1. Vitamin A is an integral part of the retina's pigment called melanin. 2. It is a component of the rods and cones, which control color visualization. 3. Vitamin A is the material in the cornea that prevents the formation of cataracts. 4. It is a necessary element of rhodopsin, which controls responses to light and dark environments.

4. It is a necessary element of rhodopsin, which controls responses to light and dark environments.

A nurse is assisting a health care provider to perform a sigmoidoscopy. In which position should the nurse place the client for this procedure? 1. Sims 2. Prone 3. Lithotomy 4. Knee-chest

4. Knee-chest

The nurse providing postoperative care for a client who had kidney surgery reviews the client's urinalysis results. The nurse concludes that the presence of what substance in the urine needs to be reported to the health care provider? 1. Sodium 2. Potassium 3. Urea nitrogen 4. Large proteins

4. Large proteins

several days ago and has been receiving warfarin sodium (Coumadin) therapy. An international normalized ratio (INR) is performed each afternoon, and the evening warfarin sodium dose is prescribed by the health care provider on a daily basis. The nurse identifies that the afternoon INR is 4.6. The next action the nurse should take is to: 1. Contact the health care provider to request the day's dosage of warfarin sodium 2. Obtain a blood specimen to have a partial thromboplastin time performed 3. Assist with meal planning to increase the intake of foods high in vitamin K 4. Maintain the client on bed rest until the health care provider reviews the laboratory results

4. Maintain the client on bed rest until the health care provider reviews the laboratory resultsThe

When developing a plan of care for a client who had a cardiac catheterization via a femoral insertion site, the nurse should include: 1. Ambulating the client two hours after the procedure 2. Checking the vital signs every 15 minutes for eight hours 3. Keeping the client nothing by mouth for four hours after the procedure 4. Maintaining the supine position for a minimum of four hours

4. Maintaining the supine position for a minimum of four hours

A nurse is caring for a client with the clinical manifestation of hypotension associated with a diagnosis of Addison disease. Which hormone is impaired in its production as a result of this disease? 1. Estrogens 2. Androgens 3. Glucocorticoids 4. Mineralocorticoids

4. Mineralocorticoids

The nurse helps a client create a list of appropriate food choices to maintain a 2-gram sodium diet that recently has been prescribed for the client. The nurse also assesses the client's cooking habits. The client tells the nurse that, at home, all food is cooked without salt. The nurse concludes that further teaching is needed when the client places what food items on the list of appropriate food choices? 1. Soft-cooked egg, salt-free toast, jelly, and skim milk 2. Baked chicken, boiled potatoes, broccoli, and coffee 3. Fillet of sole, baked potato, lettuce and tomato salad, fresh fruit cup, and milk 4. Mixed fruit salad bowl with cottage cheese, crackers, celery, sweet pickles and tea

4. Mixed fruit salad bowl with cottage cheese, crackers, celery, sweet pickles and tea

A client with type 2 diabetes develops gout, and allopurinol (Zyloprim) is prescribed. The client is also taking metformin (Glucophage) and an over-the-counter nonsteroidal anti-inflammatory drug (NSAID). When teaching about the administration of allopurinol, what should the nurse instruct the client to do? 1. Decrease the daily dose of NSAIDs. 2. Limit fluid intake to one quart a day. 3. Take the medication on an empty stomach. 4. Monitor blood glucose levels more frequently.

4. Monitor blood glucose levels more frequently.

A client with type 2 diabetes has been receiving insulin in the hospital while being treated for sepsis. The client's infection is resolving, and the health care provider writes a prescription to discontinue the 7 AM dose of insulin and administer glyburide (Micronase) 5 mg twice daily (8 AM and 8 PM). The nurse on the day shift (8 AM to 4 PM) administers the Micronase at 8:30 AM. When recording its administration in the client's record, the nurse sees that the insulin had already been administered at 7 AM. What initial action should the nurse take? 1. Measure the vital signs. 2. Notify the health care provider. 3. Assess for signs of ketoacidosis. 4. Monitor for signs of hypoglycemia.

4. Monitor for signs of hypoglycemia.

A client has been diagnosed with hyperthyroidism. The nurse expects the client to exhibit which clinical manifestations? (Select all that apply.) 1. Dry skin 2. Slow pulse 3. Weight gain 4. Nervousness 5. Increased appetite

4. Nervousness 5. Increased appetite

A male client reports dysuria, nocturia, and difficulty starting the urinary stream. A cystoscopy and biopsy of the prostate gland have been scheduled. After the procedure the client reports an inability to void. The nurse should: 1. Limit oral fluids until the client voids 2. Assure the client that this is expected 3. Insert a urinary retention catheter 4. Palpate above the pubic symphysis

4. Palpate above the pubic symphysis

A client with impaired peripheral pulses and signs of chronic hypoxia in a lower extremity is scheduled for a femoral angiogram. What would be appropriate for the nurse to include in the postprocedure plan of care? 1. Elevate the foot of the bed 2. Perform urinary catheter care every 12 hours 3. Place in the high-Fowler position 4. Perform a neurovascular assessment every two hours

4. Perform a neurovascular assessment every two hours **Because of the trauma associated with the insertion of the catheter during the procedure, the involved extremity should be assessed for sensation, motor ability, and arterial perfusion; hemorrhage or an arterial embolus can occur.

A nurse is teaching a preoperative client about postoperative breathing exercises. What information should the nurse include? (Select all that apply.) 1. Take short, frequent breaths 2 Exhale with the mouth open wide 3. Perform the exercises twice a day 4. Place a hand on the abdomen while feeling it rise 5. Hold the breath for several seconds at the height of inspiration

4. Place a hand on the abdomen while feeling it rise 5. Hold the breath for several seconds at the height of inspiration

A nurse is assessing a malnourished client with a history of cirrhosis. The client is experiencing nausea, ascites, and gastrointestinal bleeding. The primary cause of the client's ascites is a decrease in: 1. Vitamins to maintain cell coenzyme functions 2. Iron to maintain adequate hemoglobin synthesis 3. Sodium to maintain its concentration in tissue fluid 4. Plasma protein to maintain adequate capillary-tissue circulation.

4. Plasma protein to maintain adequate capillary-tissue circulation.

A client tells the nurse during the admission history that an oral hypoglycemic agent is taken daily. For which condition does the nurse conclude that an oral hypoglycemic agent may be prescribed by the health care provider? 1. Ketosis 2. Obesity 3. Type 1 diabetes 4. Reduced insulin production

4. Reduced insulin production

A client is diagnosed with diabetic ketoacidosis. Which insulin should the nurse expect the health care provider to prescribe? 1. Insulin lispro (Humalog) 2. Insulin glargine (Lantus) 3. NPH insulin (Novolin N) 4. Regular insulin (Novolin R)

4. Regular insulin (Novolin R)

A client who has type 1 diabetes is admitted to the hospital for major surgery. Before surgery the client's insulin requirements are elevated but well controlled. Postoperatively, the nurse anticipates that the client's insulin requirements will: 1. Decrease 2. Fluctuate 3. Increase sharply 4. Remain elevated

4. Remain elevated *Emotional and physical stress may cause insulin requirements to remain elevated in the postoperative period. Insulin requirements will remain elevated rather than decrease.

A client with a spontaneous pneumothorax asks, "Why did they put this tube into my chest?" The nurse explains that the purpose of the chest tube is to: 1. Check for bleeding in the lung 2. Monitor the function of the lung 3. Drain fluid from the pleural space 4. Remove air from the pleural space

4. Remove air from the pleural space

A client is admitted to the hospital for a subtotal thyroidectomy. When discussing postoperative drug therapy with the client, the nurse should teach the client to: 1. Take the iodine daily to increase the formation of thyroid hormone 2. Understand that medication will be temporary until the body adjusts to postsurgical activities 3. Take the propylthiouracil that is prescribed to stimulate the secretion of thyroid-stimulating hormone 4. Report palpitations, nervousness, tremors, or loss of weight that may indicate an overdose of thyroid hormone

4. Report palpitations, nervousness, tremors, or loss of weight that may indicate an overdose of thyroid hormone **Excessive thyroid hormone replacement may lead to signs and symptoms of hyperthyroidism. Iodine may be administered before, not after, surgery. Thyroid hormone replacement is required for life. Propylthiouracil blocks thyroid hormone synthesis; this often is administered before, not after, surgery.

A client with cirrhosis of the liver has a prolonged prothrombin time and a low platelet count. A regular diet is prescribed. What should the nurse instruct the client to do considering the client's condition? 1. Avoid foods high in vitamin K. 2. Check the pulse several times a day. 3. Drink a glass of milk when taking aspirin. 4. Report signs of bleeding no matter how slight

4. Report signs of bleeding no matter how slight **One of the many functions of the liver is the manufacture of clotting factors; there is interference in this process with cirrhosis of the liver, resulting in bleeding tendencies.

A nurse is caring for clients with various health problems. These problems include scarlet fever, otitis media, bacterial endocarditis, rheumatic fever, and glomerulonephritis. What common factor linking these diseases should the nurse consider? 1. Are self-limiting infections caused by spirilla 2. Can be controlled through childhood vaccination 3. Are caused by parasitic bacteria that normally live outside the body 4. Result from streptococcal infections that enter via the upper respiratory tract

4. Result from streptococcal infections that enter via the upper respiratory tract

A nurse in the post-anesthesia care unit is caring for a client who just had a thyroidectomy. For which client response is it most important for the nurse to monitor? 1. Urinary retention 2. Signs of restlessness 3. Decreased blood pressure 4. Signs of respiratory obstruction

4. Signs of respiratory obstruction

The nurse should ask the client with secondary syphilis about sexual contacts during the past: 1. 21 days 2. 30 days 3. Three months 4. Six months

4. Six months **The client is in the secondary stage, which begins from six weeks to six months after primary contact; therefore, a six-month history is needed to ensure that all possible contacts are located.

A client is experiencing severe respiratory distress. What response should the nurse expect the client to exhibit? 1. Tremors 2. Anasarca 3. Bradypnea 4. Tachycardia

4. Tachycardia *The heart rate increases in an attempt to compensate for the lack of oxygen to body cells.

When performing a physical assessment, the nurse identifies bilateral varicose veins. What does the nurse expect the client to report about the legs? 1. Burning sensations in the legs. 2. Calf pain when the feet are dorsiflexed. 3. Increased sensitivity of the legs to cold. 4. Worsening ankle edema as the day progresses.

4. Worsening ankle edema as the day progresses.

When advising a college student about dietary choices, the nurse should consider the caloric value of the most commonly ordered fast foods eaten by active young adults. List the following foods in order from the one with the least number of calories to the one with the most number of calories.

A garden salad has 95 calories. One slice of French toast has 126 calories. Six chicken tenders have 236 calories. An order of French fries has 372 calories. A hamburger with cheese has 720 calories.

While obtaining a health history, a nurse expects a client admitted to the hospital with chronic heart failure to report:

A need to use three pillows at night to sleep

Which clinical indicators identified by the nurse support the probable presence of a fecal impaction in a client? (Select all that apply.)

Abdominal cramps Fecal liquid seepage Hyperactive bowel sounds

A client is to receive an intravenous (IV) antibiotic in 50 mL of 0.9% sodium chloride to be administered over 20 minutes. At what rate should the nurse set the infusion pump? Record your answer using a whole number. __________ mL/hr

An infusion device delivers a specific volume of fluid to be infused over the period of one hour; calculate the answer by using ratio and proportion: 50 mL x mL -------- = ------ 20 min 60 min 20x = 50 x 60 x = 3000 ÷ 20 x = 150 mL/hr

A nurse uses the Braden Scale to predict a client's risk for developing pressure ulcers. What data should the nurse use to determine a client's score on this scale? (Select all that apply.) 1 Age 2 Anorexia 3 Hemiplegia 4 History of diabetes 5 Urinary incontinence

Anorexia Hemiplegia Urinary incontinence

A client comes to the ambulatory surgery unit on the morning of an elective surgical procedure. The client reports shortness of breath, dizziness, and palpitations. The nurse observes profuse diaphoresis and is concerned that the client may be having either a panic attack or a myocardial infarction. Which assessments support the conclusion that the client may be experiencing a myocardial infarction? (Select all that apply.)

Anxiety Chest pain Irregular pulse

Which interventions should the nurse anticipate will be prescribed initially for a client who had a hemorrhoidectomy? (Select all that apply.)

Applying moist heat Administering stool softeners

A client with heart failure is on a drug regimen of digoxin (Lanoxin) and furosemide (Lasix). The client dislikes oranges and bananas. Which fruit should the nurse encourage the client to eat?

Apricots

A client has a diagnosis of partial-thickness burns. The nurse recalls that the client's burn is different than full-thickness burns in that partial-thickness burns:

Are often painful, reddened, and have blisters

A client who was in an automobile collision is now in hypovolemic shock. Why is it important for the nurse to take the client's vital signs frequently during the compensatory stage of shock?

Arteriolar constriction occurs.

A client is admitted to the emergency department with a blood pressure of 240/150 mm Hg. The client complains of a severe headache, blurred vision, and swelling of the ankles. In response to the clinical manifestations, the nurse should:

Assess the client's pulse and respirations

A client who has had an uncomplicated myocardial infarction asks the nurse about the resumption of sexual activity. Which parameters should the nurse consider to determine the safe resumption of sexual activity? (Select all that apply.)

At least between 4 and 6 weeks after the myocardial infarction The point at which two flights of stairs can be climbed without dyspnea

A nurse is caring for clients with a variety of problems. Which health problem does the nurse determine poses the greatest risk factor for the development of a pulmonary embolus?

Atrial fibrillation

A primary health care provider prescribes atenolol (Tenormin) 20 mg by mouth four times a day for a client who has had double coronary artery bypass surgery. What information is most important for the nurse to include in the discharge teaching plan for this client?

Avoid abruptly discontinuing the medication

A client who was hospitalized with partial- and full-thickness burns over 30% of the total body surface area is to be discharged. The client asks the nurse, "How will my spouse be able to care for me at home?" How should the nurse interpret this statement?

Beginning realization of implications for the future

A client is admitted via the emergency department with the tentative diagnosis of diverticulitis. Which test commonly is prescribed to assess for this problem?

CT scan

After an acute episode of gastrointestinal (GI) bleeding, a client is diagnosed with gastric ulcer. The client receives a prescription for ranitidine (Zantac) 150 mg twice a day to be taken with meals. The nurse contacts the health care provider about the prescription because ranitidine:

Can be administered by a variety of routes

Which response should the nurse expect when assessing a client who is dehydrated? (Select all that apply.)

Confusion Sunken eyeballs Decreased blood pressure

Several hours after administering insulin, the nurse is assessing a client for an adverse response to the insulin. Which client responses are indicative of a hypoglycemic reaction? (Select all that apply.)

Confusion Tremors Diaphoresis

A client with a history of heart failure is admitted to the hospital with the diagnosis of pulmonary edema. For which signs and symptoms specific to pulmonary edema should the nurse assess the client? (Select all that apply.)

Coughing Orthopnea Diaphoresis

A client with a history of recurrent cholecystitis is scheduled for an abdominal cholecystectomy. What should the nurse specifically emphasize when planning preoperative teaching for this client?

Coughing and deep breathing

The nurse is teaching first aid to a group of community members. A participant asks what first aid should be administered to a person that suffers extensive burns. An appropriate response by the nurse is to call 911 and: 1 Apply ice to burned areas; the intervention will decrease pain 2 Use first aid cream to burned areas 3 Do nothing; attempting to treat the burned areas may cause further damage 4 Cover the burned areas with a bed sheet

Cover the burned areas with a bed sheet

An 85-year-old client has a three-day history of nausea, vomiting, and diarrhea. The client develops weakness and confusion and is admitted to the hospital. To best monitor the client's rehydration status, the nurse should assess the client's:

Daily weight

A client's serum potassium level is below the expected range. Which clinical indicators should the nurse determine are consistent with hypokalemia? (Select all that apply.)

Decreased heart rate Decreased bowel sounds

A nurse is caring for a client with right-sided heart failure. Which are key features of right-sided heart failure? (Select all that apply.)

Dependent edema Distended abdomen Polyuria at night

When assessing the client with peripheral arterial disease, the nurse anticipates the presence of which clinical manifestations? (Select all that apply.)

Dependent rubor Ulcers on the toes Delayed capillary refil

A nurse is assessing a middle-aged female client for the presence of risk factors associated with coronary heart disease. Which risk factor carries a greater risk for women than for men?

Diabetes mellitus

While assessing a client during a routine examination, a nurse in the clinic identifies signs and symptoms of hyperthyroidism. Which signs are characteristic of hyperthyroidism? (Select all that apply.)

Diaphoresis Weight loss Protruding eyes

Which clinical findings should the nurse expect when assessing a client with hyperthyroidism? (Select all that apply.)

Diarrhea Weight loss

Amlodipine (Norvasc) is prescribed for a client with hypertension. Which response to the medication should the nurse instruct the client to report to the health care provider?

Difficulty breathing

The nurse is caring for a client with chronic obstructive pulmonary disease (COPD). During assessment, the nurse expects to auscultate:

Diminished breath sounds

A client with heart failure is digitalized and placed on a maintenance dose of digoxin (Lanoxin) 0.25 mg by mouth daily. What responses does the nurse expect the client to exhibit when a therapeutic effect of digoxin is achieved?

Diuresis and decreased pulse rate

The nurse is providing postoperative care for a client who has received a prescription for nalbuphine (Nubain) for pain. For which side effects or adverse reactions should the nurse assess this client after administering this medication? (Select all that apply.)

Dry mouth Palpitations Constipation Orthostatic hypotension

Following a major abdominal surgery, a client has a nasogastric tube attached to continuous low suction. The nurse caring for the client postoperatively monitors the client for what signs of hypokalemia? (Select all that apply.)

Dysrhythmias Muscle weakness

An older adult tells the nurse, "I read about a vitamin that may be related to aging because of its effect on the structure of cell walls. I wonder whether it is wise to take it." The nurse concludes the client probably is referring to:

E

A nurse is providing counseling to a client with the diagnosis of systemic lupus erythematosus (SLE). What recommendations are essential for the nurse to include? (Select all that apply.) 1 Eat foods high in vitamin C. 2 Take your temperature daily. 3 Balance periods of rest and activity. 4 Use a strong soap when washing the skin. 5 Expose the skin to the sun as often as possible

Eat foods high in vitamin C. Take your temperature daily. Balance periods of rest and activity.

To help prevent long-term complications associated with gastric bypass surgery, the nurse needs to educate the client. Identify the factors that should be included in the nurse's teaching plan for this client. (Select all that apply.)

Eat foods rich in calcium. Consume a diet high in protein. Receive vitamin B12 injections routinely

A nurse is caring for a client with cholelithiasis and obstructive jaundice. When assessing this client, the nurse should be alert for which common clinical indicators associated with these conditions? (Select all that apply.)

Ecchymosis Yellow sclera Pain in right upper quadrant

What clinical indicators are the nurse most likely to identify when taking the admission history of a client with right ventricular failure? (Select all that apply.)

Edema Dyspnea

The nurse is assessing a client with hyperthyroidism. Which clinical indicators are consistent with this diagnosis? (Select all that apply.)

Emotional lability Dyspnea on exertion Hyperactive deep tendon reflexes

A client is admitted with anorexia, weight loss, abdominal distention, and abnormal stools. A diagnosis of malabsorption syndrome is made. What nursing action should the nurse implement to best meet this client's needs?

Encourage the consumption of high-protein foods

A client is diagnosed with cancer of the pancreas and is apprehensive and restless. Which nursing action should be included in the plan of care?

Encouraging expression of concerns

The nurse is providing care for a client that is on bed rest. The nurse can prevent skin breakdown for this client by: 1 Massaging the bony prominences 2 Maintaining a sheepskin pad under the client 3 Promoting range-of-motion activities 4 Encouraging the client to move around as much as possible

Encouraging the client to move around as much as possible

A nurse is caring for a client who is admitted to the hospital with the diagnosis of primary hyperparathyroidism. Which action should be included in this client's plan of care?

Ensuring a large fluid intake

A health care provider prescribes a sigmoidoscopy for one client and a barium enema for another client. What is a nursing responsibility common to preparing both of these clients for these procedures?

Ensuring an understanding of the procedure

A nurse is collecting information about a client who has type 1 diabetes and is being admitted because of diabetic ketoacidotic coma. Which factors can predispose a client to this condition? (Select all that apply.)

Excessive emotional stress Running a fever with the flu

A nurse is caring for a client who has a prescription for a diuretic, 2-gram sodium diet, and an oral fluid restriction of 1200 mL daily. The most recent laboratory results are blood urea nitrogen (BUN) level 42 mg/dL and creatinine 1.1 mg/dL. Considering the assessment findings, what is the most appropriate intervention by the nurse?

Expecting an increase in the oral fluid intake

A client eats a meal that contains 13 g of fat, 31 g of carbohydrates, and 5 g of protein. What is this client's total caloric intake for this meal? Record your answer using a whole number. ______ kilocalories

Fat contains 9 kilocalories per gram; carbohydrates and proteins contain 4 kilocalories per gram; therefore, 117 + 124 + 20 = 261 kilocalories.

A client is admitted to the hospital with diabetic ketoacidosis. The nurse concludes that the client's elevated ketone level is caused by incomplete oxidation of which nutrient?

Fats

A nurse is planning a community health program about screening for cancer. Which information recommended by the American Cancer Society (ACS) should the nurse include?

Fecal occult blood testing should be performed yearly beginning at age 50 years

A nurse observes a window washer falling 25 feet to the ground. The nurse rushes to the scene and determines that the person is in cardiopulmonary arrest. What should the nurse do first?

Feel for a pulse

A nurse is collecting data from a client with varicose veins who is to have sclerotherapy. What should the nurse expect the client to report?

Feeling of heaviness in both legs

A client develops peritonitis and sepsis after the surgical repair of a ruptured diverticulum. What signs should the nurse expect when assessing the client? (Select all that apply.)

Fever Tachypnea Abdominal rigidity

An older client who lives alone was found unconscious on the floor at home. The client was admitted to the hospital with the diagnoses of a fractured hip, kidney failure, and dehydration. In the 24 hours since admission, the client received 1500 mL of intravenous fluid and the serum electrolyte value demonstrates hyponatremia. The nurse concludes that the element that most likely contributed to the hyponatremia is:

Fluid intake

An active adolescent is admitted to the hospital for surgery for an ileostomy. When planning a teaching session about self-care, the nurse includes sports that should be avoided by a client with an ileostomy. Which should be included on the list of sports to be avoided? (Select all that apply.)

Football Ice hockey

Which factors should the nurse identify that can precipitate hyponatremia? (Select all that apply.)

Gastrointestinal (GI) suction Diuretic therapy Continuous bladder irrigation

A nurse is eliciting a health history from a client with ulcerative colitis. What factor does the nurse consider to be most likely associated with the client's colitis?

Genetic predisposition

A health care provider prescribes an upper gastrointestinal (GI) series and a barium enema. The client asks, "Why do I have to have barium for these tests?" The nurse's best response is "Barium:

Gives more contrast to the soft tissue of the alimentary tract, allowing absorption of x-rays."

A nurse is caring for a newly admitted client with a diagnosis of Cushing syndrome. Why should the nurse monitor this client for clinical indicators of diabetes mellitus?

Glucocorticoids accelerate the process of gluco-neogenesis

A client is diagnosed with Cushing syndrome. Which clinical manifestation does the nurse expect to increase in a client with Cushing syndrome?

Glucose level

A nurse is caring for a client newly admitted with a diagnosis of pheochromocytoma. Which clinical findings does the nurse expect when assessing this client? (Select all that apply.)

Headache Palpitations Diaphoresis

A nurse is assessing a client for dehydration, The client has had diarrhea and vomiting for 48 hours. What are indicators of dehydration? (Select all that apply.)

Headache The skin on the client's forehead remains tented after being pinched

The nurse is caring for a client with type 1 diabetes. For which signs or symptoms of insulin reaction should the nurse particularly be observant? (Select all that apply.)

Headache Diaphoresis

During a client's paracentesis, 1500 mL of fluid is removed. The nurse monitors the client for which sign of a potentially severe response?

Heart rate increases from 80 to 135

A nurse is caring for several postoperative clients who had abdominal surgery. What independent nursing intervention can help prevent the development of thrombophlebitis?

Helping the client to perform in-bed exercises

A nurse epidemiologist is responsible for wound consults at the hospital where a client has been admitted with an infected wound. The client asks, "What is the primary role of a nurse epidemiologist?" The nurse explains that the nurse epidemiologist:

Helps health care providers to control infections

A client who was admitted with a diagnosis of acute lymphoblastic leukemia is receiving chemotherapy. Which assessment findings should alert the nurse to the possible development of the life threatening response of thrombocytopenia? (Select all that apply.)

Hematuria Ecchymosis

While receiving a blood transfusion, a client develops flank pain, chills, fever, and hematuria. What type of transfusion reaction does the nurse conclude that the client probably is experiencing?

Hemolytic

A client is admitted to the hospital with a diagnosis of peptic ulcer. The nurse should assess the client for which complication commonly associated with the diagnosis?

Hemorrhage

A nurse educator of a college health course is discussing tattoos with the class. Which type of hepatitis associated with tattoos should the nurse include in the teaching plan?

Hepatitis-C

A client was admitted with full-thickness burns two weeks ago. Since admission, the client has lost an average of a pound of weight each day. The nurse expects the client's diet to be adjusted to include: 1Low-sodium milk 2High protein drinks 3Foods that are low in potassium 4 Ten percent more calories in the form of fats

High protein drinks

A nurse is assessing a female client with Cushing syndrome. Which clinical findings can the nurse expect to identify? (Select all that apply.)

Hirsutism Buffalo hump

A nurse is caring for a client who is scheduled for a bilateral adrenalectomy. Which medication should the nurse expect to be prescribed for this client on the day of surgery and in the immediate postoperative period?

Hydrocortisone succinate (Solu-Cortef)

A client with cirrhosis of the liver develops ascites, and the health care provider prescribes spironolactone (Aldactone).What should the nurse monitor the client for?

Hyperkalemia

During the first 48 hours after a thermal injury, the nurse should assess the client for:

Hyperkalemia and hyponatremia

Postoperatively a client who had a thyroidectomy complains of tingling and numbness of the fingers and toes, and the nurse observes muscle twitching. Which complication does the nurse suspect the client is experiencing?

Hypocalcemia

A client takes isosorbide dinitrate (Isordil) daily. The client states, "I would like to start taking sildenafil (Viagra) for erectile dysfunction. I was told I can't take sildenafil and isosorbide dinitrate at the same time." The nurse explains that taking both of these medications concurrently may result in severe:

Hypotension

A client complains of foot pain and is diagnosed with arterial insufficiency. The nurse provides teaching about what the client can do to increase arterial dilation and to decrease foot pain. The nurse concludes that further teaching is needed when the client states what?

I will elevate my foot.

A nurse is caring for a client who is admitted to the hospital with ascites and a diagnosis of cirrhosis of the liver. The nurse concludes that the probable cause of ascites is:

Impaired portal venous return

A client is hospitalized for intravenous antibiotic therapy and an incision and drainage of an abscess that developed at the site of a puncture wound. When should the nurse begin to teach the client about how to care for the wound?

In the preoperative period

A client has coronary artery bypass graft (CABG) surgery for the second time via a sternal incision. What should the nurse teach the client to expect when returning home?

Increased edema in the leg that provided the donor graft

What are expected changes that the nurse might identify when assessing the skin of an older adult? (Select all that apply.) 1 Scaly skin 2 Increased wrinkles 3 Signs of ecchymosis 4 Marked flaking of skin 5 Hyperpigmented patches

Increased wrinkles Hyperpigmented patches

A client with a long history of alcohol abuse is admitted to the hospital with ascites and jaundice. A diagnosis of hepatic cirrhosis is made. A nursing priority is to:

Institute fall prevention/safety measures

A client who recently was diagnosed as having myelocytic leukemia discusses the diagnosis by referring to statistics, facts, and figures. The nurse determines that the client is using the defense mechanism known as:

Intellectualization

A client is scheduled to begin chemotherapy two weeks after the client had surgery for colon cancer. The nurse explains to the client that the delay in instituting drug therapy is planned because the chemotherapy:

Interferes with cell growth and delays wound healing

The health care provider prescribes propylthiouracil (PTU) for a client with hyperthyroidism. The nurse explains that this drug:

Interferes with the synthesis of thyroid hormone

A client is admitted with a diagnosis of cancer of the colon. What information about malignant tumors of the colon should the nurse consider when caring for this client?

Intestinal obstructions usually are malignant

When discussing the therapeutic regimen of vitamin B12 for pernicious anemia with a client, the nurse explains that:

Intramuscular injections once a month will maintain control

A nurse is assessing two clients. One client has ulcerative colitis and the other client has Crohn's disease. Which is more likely to be identified in the client with ulcerative colitis than the client with Crohn's disease?

Involvement starting distally with rectal bleeding that spreads continuously up the colon

Valsartan (Diovan), an angiotensin II receptor antagonist, is prescribed for a client. For which possible side effects should the nurse monitor the client? (Select all that apply.)

Irregular pulse rate Orthostatic hypotension

A nurse provides dietary teaching for a client with an acute exacerbation of colitis, and afterward the client makes a list of foods that can be included on the diet. Which food choices indicate that the teaching was effective? (Select all that apply.)

Jelly sandwich Lean roast beef Scrambled eggs

The nurse is caring for a client who is experiencing signs and symptoms of a cardiac dysrhythmia who is scheduled to wear a Holter monitor for 24 hours. During the test, the client should be instructed to:

Keep a diary of activities

The nurse is caring for a client diagnosed with Cushing syndrome. The nurse expects that the client will exhibit:

Lability of mood

A client who had a laparoscopic cholecystectomy reports pain in the shoulder. In what position should the nurse place the client?

Left Sims

A client with a history of ulcerative colitis has a large portion of the large intestine removed and the creation of an ileostomy. For which potential life-threatening complication should the nurse assess the client after this surgery?

Limited water reabsorption caused by removal of intestine

A client with cancer develops pancytopenia during the course of chemotherapy. The client asks the nurse why this has occurred. The nurse explains that:

Noncancerous cells also are susceptible to the effects of chemotherapeutic drugs

An older adult is brought to the emergency department after being found in the street without a coat during a snowstorm. What actions should the nurse implement? (Select all that apply.)

Obtain a rectal temperature. Assess the fingers for areas of frostbite. Determine client's level of consciousness. Ask for client identification.

An older African-American client with hypertension is admitted to the hospital. Which data from the client's history and diagnostic workup represent risk factors for hypertension? (Select all that apply.)

Occasional cocaine use African-American heritage

A nurse obtains daily stool specimens for a client with chronic bowel inflammation. The nurse concludes that these stool examinations were prescribed to determine:

Occult blood

A nurse in the postanesthesia care unit identifies a progressive decrease in blood pressure in a client who had major abdominal surgery. What clinical finding supports the conclusion that the client is experiencing internal bleeding?

Oliguria

The nurse concludes that a client is experiencing hypovolemic shock. Which physical characteristic supports this conclusion?

Oliguria

As an acute episode of rheumatoid arthritis subsides, active and passive range-of-motion exercises are taught to the client's spouse. The nurse should teach that direct pressure should not be applied to the client's joints because this may precipitate:

Pain

A nurse is assessing a client with a diagnosis of hypoglycemia. What clinical manifestations support this diagnosis? (Select all that apply.)

Palpitations Diaphoresis Slurred speech

A client with impaired peripheral pulses and signs of chronic hypoxia in a lower extremity is scheduled for a femoral angiogram. What would be appropriate for the nurse to include in the postprocedure plan of care?

Perform a neurovascular assessment every two hours

Blood studies are being performed on a client with the potential diagnosis of hyperparathyroidism. What serum blood level should the nurse expect to be decreased when reviewing this client's hematological studies?

Phosphorus

A client is receiving total parenteral nutrition. The nurse assesses for which client response that indicates hyperglycemia?

Polyuria

A nurse is planning care for a client admitted to the hospital with abdominal spasms and pain associated with severe diarrhea. What primary serum blood level should the nurse monitor?

Potassium

After a subtotal gastrectomy a client is returned to the surgical unit. Which is the best nursing action to prevent pulmonary complications?

Promoting frequent turning and deep breathing to mobilize secretions

A nurse begins to develop a plan of care with a client who has left ventricular heart failure that resulted from a myocardial infarction (MI). What should be the primary focus of the plan during the acute phase of recovery?

Promoting physical and emotional rest

Which relationship does the nurse consider reflective of the relationship of naloxone (Narcan) to morphine sulfate?

Protamine sulfate to parenteral heparin

A client is brought to the emergency department with moderate substernal chest pain radiating to the inner aspect of the left arm, unrelieved by rest and nitroglycerin. The pain is associated with slight nausea and anxiety. What is the priority nursing intervention for this client?

Provide pain medication

During a client's routine physical examination, an abdominal aortic aneurysm is diagnosed. The client is admitted to the hospital immediately, and surgery is scheduled for the next morning. Which clinical finding should the nurse expect when completing the admission assessment?

Pulsating abdominal mass

A client is hospitalized for the treatment of thrombophlebitis. What should the nurse include in the client's teaching plan related to how to prevent thrombophlebitis?

Put on elastic stockings before arising

A client has a low hemoglobin level, which is attributed to nutritional deficiency, and the nurse provides dietary teaching. Which food choices by the client indicate that the nurse's instructions are effective? (Select all that apply.)

Raisins Spinach

A client's blood pressure increases dramatically 6 hours after a femoral-popliteal bypass graft. Which concern motivates the nurse to inform the health care provider?

Rapidly increasing blood pressure may rupture the graft.

A client who has type 1 diabetes is admitted to the hospital for major surgery. Before surgery the client's insulin requirements are elevated but well controlled. Postoperatively, the nurse anticipates that the client's insulin requirements will:

Remain elevated

An unresponsive older adult is admitted to the emergency department on a hot, humid day. The initial nursing assessment reveals hot, dry skin, a respiratory rate of 36 breaths/min, and a heart rate of 128 beats/min. What is the initial nursing action?

Remove the clothing.

A client with cirrhosis of the liver has a prolonged prothrombin time and a low platelet count. A regular diet is prescribed. What should the nurse instruct the client to do considering the client's condition?

Report signs of bleeding no matter how slight.

A nurse is caring for a client with a diagnosis of varicose veins. Which clinical findings can the nurse expect to identify when assessing this client? (Select all that apply

Reports of leg fatigue Tortuous veins in the legs Pain in lower extremities when standing

A nurse is caring for a client with a diagnosis of varicose veins. Which clinical findings can the nurse expect to identify when assessing this client? (Select all that apply.)

Reports of leg fatigue Tortuous veins in the legs Pain in lower extremities when standing

A client is admitted to the hospital after sustaining serious burns that involve a large surface of the skin. The nurse is caring for the client during the emergent phase after the injury. Which nursing objective is the priority during this phase? 1 Alleviate pain 2 Prevent infection 3 Replace blood loss 4 Restore fluid volume

Restore fluid volume

A client with type 1 diabetes asks what causes the several brown spots on the skin. The nurse's best response is, "The brown spots:

Result from small blood vessel damage; the blood contains iron, which leaves a brown spot."

An older adult is returned to the surgical unit after having a subtotal gastrectomy. Which dietary modification should the nurse anticipate that the health care provider will most likely prescribe?

Resume small, easily digested feedings gradually.

A nurse is caring for a client with an infection caused by group A beta-hemolytic streptococci. The nurse should assess this client for responses associated with which illness?

Rheumatic fever

A woman comes to the emergency department reporting signs and symptoms determined by the health care provider to be caused by a myocardial infarction. The nurse obtains a health history. Which reported symptoms does the nurse determine are specifically related to a myocardial infarction in women? (Select all that apply.)

Severe fatigue Sense of unease

A client is admitted for repair of bilateral inguinal hernias. Before surgery the nurse assesses the client for signs that strangulation of the intestine may have occurred. What is an early sign of strangulation?

Sharp abdominal pain

A health care provider tells a client that vitamin E and beta-carotene are important for healthier skin. Which foods should the nurse recommend that are excellent sources of both of these substances? 1 Spinach and mangoes 2 Fish and peanut butter 3 Oranges and grapefruits 4 Carrots and sweet potatoes

Spinach and mangoes

A nurse is teaching a postoperative client about the importance of vitamin C for wound healing. Which food selection demonstrates the client is applying the information correctly?

Strawberries

When planning discharge teaching for a client who had an ileostomy, the nurse places primary emphasis on:

Telling the client whom to contact if assistance is needed

A nurse is caring for a client who just had major abdominal surgery. What client responses indicate the possibility of developing a superficial venous thrombosis? (Select all that apply.)

Tender area in the posterior lower leg Warmth along the course of the involved vessel

A client has a diagnosis of superficial partial-thickness burns. The client asks what layers of skin are involved with this type of burn. What is an appropriate nursing response? 1 The epidermis is damaged 2 The dermis is damaged partially 3 Both the epidermis and the dermis are destroyed 4 The structures beneath the skin are destroyed

The epidermis is damaged

A client has a prescription for an antibiotic in an intravenous (IV) piggyback of 50 mL of D5W to run for 30 minutes. The microdrip tubing has a drop factor of 60 gtt/mL. At what rate should the nurse set the IV infusion? Record your answer using a whole number. __________ gtts/min

The total drops per minute should be divided by the total time in minutes: 50 mL x 60 gtt/mL 3000 ----------------------- = ------ = 100 gtt/minute 30 minutes 30

A client who has had a subtotal thyroidectomy does not understand how hypothyroidism can develop when the problem was initially hyperthyroidism. The nurse bases a response on the fact that:

There may not be enough thyroid tissue to supply adequate thyroid hormone

In response to a client's question, the nurse explains the difference between partial-thickness (second-degree) burns and full-thickness (third-degree) burns. What information about partial-thickness burns should the nurse include in the discussion? 1 They are painful, reddened, and have blisters 2 Grafting will be required before they can heal 3 There is destruction of both the epidermis and dermis 4 Months of extensive treatment are required before healing

They are painful, reddened, and have blisters

The nurse provides education related to manifestations of hyperglycemia to a client with type 1 diabetes. Which signs and symptoms identified by the client indicate that the teaching was effective? (Select all that apply.)

Thirst Fruity breath odor Excessive urination

A nurse providing care to a client who had major abdominal surgery monitors the client for postoperative complications. Which clinical findings are indicators of impending hypovolemic shock?

Thirst, cool skin, and orthostatic hypotension

A nurse is responding to the needs of victims at a collapsed building. What principle guides the nurse's priorities during this disaster?

Those requiring minimal care are treated first so they can help others.

A client has been taking levothyroxine (Synthroid) for hypothyroidism for three weeks. The nurse suspects that a decrease in dosage is needed when the client exhibits which clinical manifestations? (Select all that apply.)

Tremors Heat intolerance

After a head injury a client develops a deficiency of antidiuretic hormone (ADH). What should the nurse consider about the response to secretion of ADH before assessing this client?

Tubular reabsorption of water increases

A person sustains deep partial-thickness burns while working on a boat in a town marina and seeks advice from the nurse in the first aide station. The nurse encourages the client to seek medical attention but the client refuses. The nurse advises the person to go to a health care provider if: 1 Blisters appear 2 Urinary output decreases 3 Edema and redness occur 4 Low-grade fever develops

Urinary output decreases

Laboratory results of a client's blood after chemotherapy indicate bone marrow depression. What should the nurse encourage the client to do? (Select all that apply.)

Use a soft toothbrush. Read the ingredients in over-the-counter drugs before taking them.

When caring for a client, what clinical indicators should the nurse immediately report to the health care provider? (Select all that apply.) 1 Weakness 2 Diaphoresis 3 Tachycardia 4 Cold extremities 5 Flushed skin tone

Weakness Diaphoresis Tachycardia Cold extremities

Which is the best advice the nurse can give regarding foot care to a client diagnosed with diabetes?

Wear synthetic fiber socks when exercising

After surgical clipping of a cerebral aneurysm, the client develops the syndrome of inappropriate secretion of antidiuretic hormone (ADH). For which manifestations of excessive levels of ADH should the nurse assess the client? (Select all that apply.)

Weight gain Hyponatremia

The nurse stops at an accident scene to administer emergency care for a person who has sustained partial- and full-thickness burns to the chest, right arm, and upper legs as the result of a car fire. What should the nurse do first when caring for this person?

Wrap the person in a clean, dry sheet

A nurse is caring for a client with an underactive thyroid gland. Which responses should the nurse expect the client to exhibit as a result of decreased levels of triiodothyronine (T3 ) and thyroxine (T4 )? (Select all that apply.)

Weight gain Cold intolerance

A nurse identifies premature ventricular complexes (PVCs) on a client's cardiac monitor and concludes that these complexes are a sign of:

Cardiac irritability

When assessing for hemorrhage after a client has a total hip replacement, the most important nursing action is to:

Examine the bedding under the client

A nurse observes the following dysrhythmia on a client's cardiac monitor. What rhythm does the nurse identify?

Ventricular fibrillation

A client has a colon resection with an anastomosis. What assessments by the nurse support a suspicion of impending shock? Select all that apply.

Oliguria Irritability Hypotension

What is the primary focus of nursing care for a client admitted with tetanus caused by a puncture wound?

3 Decreasing external stimuli

When caring for an anxious patient, the nurse should monitor for which signs of hyperventilation?

4 Respiratory alkalosis

Metoprolol (Toprol-XL) is prescribed for a client with hypertension. For which side effect should the nurse monitor the client? 1. Hirsutism 2. Bradycardia 3. Restlessness 4. Hypertension

2. Bradycardia

A client with a dysrhythmia is admitted to telemetry for observation. In the morning, the client asks for a cup of coffee. What is the nurse's best response?

"Coffee has caffeine, which can affect your heart. It should be avoided."

A carpenter with full-thickness burns of the entire right arm confides, "I'll never be able to use my arm again and I'll be scarred forever" The nurse's best initial response is: 1 "The staff is taking steps to minimize scarring." 2"Think about how lucky you are. You are alive." 3 "Try not to worry for now. Concentrate on your range-of-motion exercises." 4"I know you're worried, but it is too early to tell how much scarring will occur."

"I know you're worried, but it is too early to tell how much scarring will occur."

A client has a new colostomy. The nurse has provided teaching related to when the client should irrigate the colostomy. Which client statement indicates correct understanding of the teaching?

"I plan to irrigate it in the late morning, the same time I had a bowel movement every day before I had my surgery."

An older client is brought to the hospital by a family member because of deep partial-thickness burns on the arms and hands. The client protests being hospitalized and asks, "Why can't I just go home and have my spouse care for me?" What is the best response by the nurse? 1 "You sound upset, but your health care provider knows best. You should do what is prescribed." 2 "Your spouse is very capable, but if your burns get infected, a family member can't give you the injections you will need." 3 "Your burns are more serious than you think, and we have specially trained people here just to take care of you." 4 "You may heal more slowly because of your age, and you may need the special care and equipment available in the hospital."

"You may heal more slowly because of your age, and you may need the special care and equipment available in the hospital."

A client experiences elevated triglycerides and cholesterol. The client appears discouraged and says, "Well, I guess I'd better cut out all the fat and cholesterol in my diet." Which is the nurse's most appropriate response?

"You need some fat to supply the necessary fatty acids, so it's mainly just a need to cut down on the amount of fat you consume."

A client has a diagnosis of myasthenia gravis. The nurse recalls that associated clinical manifestations include:

1 Blurred vision along with episodes of vertigo

A nurse is administering oxygen to a client with chest pain who is restless. What method of oxygen administration will most likely prevent a further increase in the client's anxiety level?

1 Cannula

Which statement regarding treatment with interferon indicates that the client understands the nurse's teaching?

1 "I will drink two to three quarts of fluid a day."

A client with the diagnosis of osteogenic sarcoma has metastasis to the lung. Which client statement about the concept of metastasis indicates a need for further instruction?

1 "I'm upset to know that the tumor may metastasize to my bones."

A client with chronic bronchitis smokes one or two cigarettes a day and has not been performing the prescribed pulmonary physiotherapy exercises because they are too tiring. What is the best response by the nurse?

1 "Tell me about your typical day before the exercises were prescribed."

A nurse is caring for a client who has had type 1 diabetes for 25 years. The client states, "I have been really bad for the last 15 years. I have not paid attention to my diet and have done little to control my diabetes." What signs of common complications of diabetes might the nurse expect to identify when assessing this client? (Select all that apply.) 1 . Leg ulcers 2 . Loss of visual acuity 3 . Thick, yellow toenails 4 . Increased growth of body hair 5 . Decreased sensation in the feet

1 . Leg ulcers 2 . Loss of visual acuity 3 . Thick, yellow toenails 5 . Decreased sensation in the feet

A client is receiving furosemide (Lasix) to relieve edema. The nurse should monitor the client for which responses? (Select all that apply.) 1 . Weight loss 2 . Negative nitrogen balance 3. Increased urine specific gravity 4. Excessive loss of potassium ions 5 . Pronounced retention of sodium ions

1 . Weight loss 4. Excessive loss of potassium ions

A client who had a brain attack (CVA) two weeks ago is having problems communicating. The nurse shows the client a picture of a baseball and asks the client to identify it and its characteristics. The client describes its color, size, and purpose but cannot identify it as a ball. The nurse documents this response as:

1 Anomia

What technique should a nurse use when cleaning a tracheostomy tube that has a nondisposable inner cannula?

1 Apply a precut dressing around the insertion site with the flaps pointing upward

What nursing action will limit hypoxia when suctioning a client's airway

1 Apply suction only after catheter is inserted

A nurse is caring for several postoperative clients. For what clinical manifestations of a pulmonary embolus should the nurse monitor these clients? (Select all that apply.)

1 Dyspnea 3 Hemoptysis 5 Feeling of impending doom

The nurse supports cognitive ability in clients with Alzheimer dementia, by: (Select all that apply.)

1 Using calendars, clocks, and pictures to support memory 2 Encouraging caregivers to support protected independence 3 Providing a limited number of choices to support decision making

A client arrives at the outpatient clinic with a painful leg ulcer, and the nurse performs a physical assessment. Which clinical findings in the lower extremity support a diagnosis of an arterial ulcer? (Select all that apply.) 1. Lack of hair 2. Thickened toenails 3. Pain at the ulcer site 4. Diminished pedal pulse 5 . Brown skin discoloration

1. Lack of hair 2. Thickened toenails 3. Pain at the ulcer site 4. Diminished pedal pulse

Which health problem should the nurse consider is most likely to precipitate acute hypoglycemia in a client? 1. Liver disease 2. Hypertension 3. Hyperthyroidism 4. Cushing's syndrome

1. Liver disease

A client is admitted to the hospital with a diagnosis of cirrhosis of the liver. For which classic signs of hepatic coma should the nurse assess this client? (Select all that apply.) 1. Mental confusion 2. Increased cholesterol 3. Brown-colored stools 4. Flapping hand tremors 5. Hyperactive deep tendon reflexes

1. Mental confusion 4. Flapping hand tremors

A client is admitted with dehydration. Which findings should the nurse expect the client to exhibit? (Select all that apply.) 1. Rapid, thready pulse 2. Increased skin turgor 3. Decreased hematocrit 4. Elevated specific gravity 5. Adventitious breath sounds

1. Rapid, thready pulse 4. Elevated specific gravity

A nurse is caring for a client with an indwelling urinary catheter. What is the most important action for the nurse to implement when irrigating the bladder? 1. Use sterile equipment. 2. Instill the fluid under high pressure. 3. Warm the solution to body temperature. 4. Aspirate immediately to ensure return flow.

1. Use sterile equipment.

A client is diagnosed with hyperthyroidism and is experiencing exophthalmia. Which measures should the nurse include when teaching this client how to manage the discomfort associated with exophthalmia? (Select all that apply.) 1. Use tinted glasses. 2. Use warm, moist compresses. 3. Elevate the head of the bed 45 degrees. 4. Tape eyelids shut at night if they do not close. 5. Apply a petroleum-based jelly along the lower eyelid.

1. Use tinted glasses. 3. Elevate the head of the bed 45 degrees. 4. Tape eyelids shut at night if they do not close.

A homosexual client is diagnosed with human immunodeficiency virus (HIV). The primary nurse informed the nursing team that the client wept when told of the diagnosis. A health care team member responded by saying, "I don't feel sorry for people like that. My philosophy is that you made your bed and now you can sleep in it." What is the basis of the team member's comment? 1. Values and beliefs about sexual lifestyles. 2. Anger and mistrust of homosexuals in general. 3. Discomfort with people who are unable to control their emotions. 4. Hostility over having to care for someone with a sexually related infection.

1. Values and beliefs about sexual lifestyles.

The nursing staff has a team conference on acquired immunodeficiency syndrome (AIDS) and discusses the routes of transmission of the human immunodeficiency virus (HIV). The discussion reveals that there is no risk of exposure to HIV when an individual:

2 Makes a donation of a pint of whole blood

A client develops subcutaneous emphysema after a chest injury with suspected pneumothorax. What assessment by the nurse is the best method for detecting this complication?

2 Palpating the neck or face

The nurse is caring for a 75-year-old client that had radical head and neck surgery. Thirty minutes after awakening from anesthesia, the client becomes agitated, disoriented, and confused. The nurse should:

2 Administer the prescribed oxygen

A nurse auscultates a client's lungs and hears a fine crackling sound in the left lower lung during respiration. The nurse charts, "crackles and rhonchi in the left lower lung." What does this documentation represent?

2 An inaccurate interpretation

What should the nurse take into consideration when planning nursing care for a client experiencing an acute episode of rheumatoid arthritis?

2 Bony ankylosis of a joint is irreversible and causes immobility.

A nurse is caring for a client who is admitted to the hospital with severe dyspnea and a diagnosis of cancer of the lung. The nurse concludes that the severe dyspnea probably is caused by:

2 Bronchial obstruction or pleural effusion

A client who had a tonic-clonic seizure of unknown etiology is to begin taking phenytoin (Dilantin). The nurse should instruct the client to:

2 Brush the teeth and gums three times daily

A nurse is caring for a client who had a total hip replacement. What nursing action should be incorporated into the plan of care to prevent thrombus formation?

2 Encouraging the client to perform ankle exercises

A client just had a thoracentesis. For which response is it most important for the nurse to observe the client?

2 Expectoration of blood

A primary health care provider prescribes airborne precautions for a client with tuberculosis. After being taught about the details of airborne precautions, the client is seen walking down the hall to get a glass of juice from the kitchen. The most effective nursing intervention is to:

2 Explore what the precautions mean to the client

In addition to Pneumocystis jiroveci, a client with acquired immunodeficiency syndrome (AIDS) also has an ulcer 4 cm in diameter on the leg. Considering the client's total health status, the most critical concern is:

2 Gas exchange

A client with type 1 diabetes consistently has high glucose levels on awakening in the morning. What should the nurse instruct the client to do to differentiate between the Somogyi effect and the dawn phenomenon? 1. Eat a snack before going to bed. 2. Measure the blood glucose level between 2 AM and 4 AM. 3. Administer the prescribed bedtime insulin immediately before going to bed. 4. Identify whether symptoms experienced in the morning are associated with either hyperglycemia or hypoglycemia

2. Measure the blood glucose level between 2 AM and 4 AM.

When receiving hemodialysis, the complication of the removal of too much sodium may occur. For which clinical findings associated with hyponatremia should the nurse assess the client? (Select all that apply.) 1. Chvostek sign 2. Muscle cramps 3. Extreme fatigue 4. Cardiac dysrhythmias 5. Increased temperature

2. Muscle cramps 3. Extreme fatigue

A client has a persistent productive cough that becomes blood tinged. A needle biopsy is scheduled. The client tells the nurse, "During the procedure, a needle will be inserted into my back to collapse my lung." Which is the most appropriate response by the nurse?

3 "Tell me more about the conversation you had with your health care provider."

A nurse is conducting cholesterol screening for a manufacturing corporation during a health fair. A 50-year-old man who is 6 feet tall and weighs 293 pounds puts out his cigarette and asks the nurse how to modify his risk factors for coronary artery disease. On which risk factors should the nurse help the client focus? (Select all that apply.) 1 . Age 2 . Height 3 . Weight 4 . Smoking 5 . Family history

3 . Weight 4 . Smoking

A nurse is caring for a client with end-stage renal disease. Which clinical indicators of end-stage renal disease should the nurse expect? (Select all that apply.) 1. Polyuria 2. Jaundice 3 .Azotemia 4. Hypertension 5. Polycythemia

3 .Azotemia 4. Hypertension

A client presents to the emergency department with a fever, headache, loss of appetite, and malaise. The nurse identifies raised red bumps on the client's arms and legs. A diagnosis of chickenpox is made. The client should be placed in a private room with what kind of precautions?

3 Airborne precautions

A client returns from the post-anesthesia care unit after a right rotator cuff repair. What should the nurse do when performing a neurovascular assessment?

3 Assess for capillary refill in the nail beds.

During the postoperative period after surgery for a kidney transplant, the client's creatinine level is 3.1 mg/dL. What should the nurse do first in response to this laboratory result?

3 Assess for decreased urine output.

A nurse is caring for a client after surgical creation of a conduit diversion. What is the major disadvantage of a conduit diversion that the nurse should consider when caring for this client? 1. Peristalsis is greatly decreased. 2. Stool continuously oozes from it. 3. Urine continuously drains from it. 4. Absorption of nutrients is diminished.

3. Urine continuously drains from it.

Which clinical indicator should the nurse identify as expected for a client with type 2 diabetes? 1. Ketones in the blood but not in the urine. 2. Glucose in the urine but not hyperglycemia. 3. Urine negative for ketones and hyperglycemia. 4. Blood and urine positive for both glucose and ketones

3. Urine negative for ketones and hyperglycemia.

The nurse is making rounds on a patient who has developed severe bone marrow depression after receiving chemotherapy for cancer. Which of these actions by the nurse is appropriate? (Select all that apply.) 1. Monitor for signs of alopecia. 2. Encourage an increase in fluids. 3. Wash hands before entering the client's room. 4. Advise use of a soft toothbrush for oral hygiene. 5. Report an elevation in temperature immediately. 6. Encourage the client to eat raw, fresh fruits and vegetables.

3. Wash hands before entering the client's room. 4. Advise use of a soft toothbrush for oral hygiene. 5. Report an elevation in temperature immediately.

When obtaining the history of a client recently diagnosed with type 1 diabetes, the nurse expects to identify the presence of: 1. Edema 2. Anorexia 3. Weight loss 4. Hypoglycemic episodes

3. Weight loss

When assessing a client with Graves disease, the nurse expects to identify: 1. Constipation, dry skin, and weight gain 2. Lethargy, weight gain, and forgetfulness 3. Weight loss, exophthalmos, and restlessness 4. Weight loss, protruding eyeballs, and lethargy

3. Weight loss, exophthalmos, and restlessness *Weight loss and restlessness occur because of an increased basal metabolic rate; exophthalmos occurs because of peribulbar edema

Ampicillin 250 mg by mouth every six hours is prescribed for a client who is to be discharged. Which statement indicates to the nurse that the client understands the teaching about ampicillin?

4 "The medicine should be taken one hour before or two hours after meals."

A client with liver dysfunction states, "My gums have been bleeding spontaneously." The nurse identifies small hemorrhagic lesions on the client's face. The nurse concludes that the client needs additional Vitamin: 1. D 2. E 3. A 4. K

4. K

A client is scheduled for an intravenous pyelogram (IVP). The nurse explains that on the day before the IVP the client must: 1. Avoid fats and proteins 2. Drink a large amount of fluids 3. Omit dinner and limit beverages 4. Take a laxative before going to bed

4. Take a laxative before going to bed **Laxatives remove feces and flatus, providing better visualization.

A 75-year-old client has a baseline blood pressure of 140/90 mm Hg. The nurse obtains a sitting blood pressure in the client's left arm, and the reading is 160/100 mm Hg. What action should the nurse take next? 1. Advise the client to restrict fluid and sodium intake, then begin to develop a teaching plan for the client. 2. Contact the primary health care provider immediately to report the blood pressure reading. 3. Record the findings, recognizing that the result is expected for an older adult. 4. Take the blood pressure in the right arm, and then take the blood pressure in both arms while the client is standing.

4. Take the blood pressure in the right arm, and then take the blood pressure in both arms while the client is standi

A client has a suspected peptic ulcer in the duodenum. What should the nurse expect the client to report when describing the pain associated with this disease?

A gnawing sensation relieved by food

A nurse is caring for an alert client who has diabetes and is receiving an 1800-calorie American Diabetic Association diet. The client's blood glucose level is 60 mg/dL. The health care provider's protocol calls for treatment of hypoglycemia with 15 g of a simple carbohydrate. The nurse should:

Ask the client to ingest one tube of glucose gel

A client is admitted to the emergency department with crushing chest pain. A diagnosis of acute coronary syndrome is suspected. The nurse expects that the client's initial treatment will include which medication?

Aspirin (ASA)

To be most effective when teaching colostomy care to a client, the nurse must first:

Assess barriers to learning colostomy care

A client is admitted with thrombocytopenia. What specific nursing actions are appropriate to include in the plan of care for this client? (Select all that apply.)

Avoid intramuscular injections Examine the skin for ecchymotic areas

A nurse is teaching a client who had a myocardial infarction about the prescribed 1500-calorie, 2-gram-sodium, weight-reducing diet. Which low-sodium, low-calorie nutrients should the nurse recommend that the client include in the diet? (Select all that apply.)

Baked chicken Mashed potatoes

A client with chronic heart failure is taking a diuretic twice a day. The health care provider prescribes a diet that includes the intake of dietary potassium. What foods that have a higher amount of potassium should the nurse instruct the client to consume? (Select all that apply.)

Bananas Baked potatoes with skins

A client who has bone pain of insidious onset is suspected of having multiple myeloma. The nurse expects that a diagnostic finding specific for multiple myeloma is:

Bence Jones protein in the urine

Which patients are at risk of developing health care-associated infections (HAIs)? Select all that apply.

C A patient with laryngeal cancer D A patient with diabetes mellitus E A patient with an indwelling urinary catheter

A client's laboratory values demonstrate an increased serum calcium level, and further diagnostic tests reveal hyperparathyroidism. For what clinical manifestations should the nurse assess this client? (Select all that apply.)

Cardiac dysrhythmias Hypoactive bowel sounds

A client with hypertension is to begin a 2-gram sodium diet. The nurse should teach the client to avoid which foods? (Select all that apply.)

Celery sticks Luncheon meat

A client is diagnosed with Hodgkin disease. Which lymph nodes does the nurse expect to be affected first? Correct1 Cervical

Cervical

A nurse is teaching a group of adults about the signs and symptoms of colorectal cancer. What is the most common clinical manifestation that the nurse should include in the teaching program?

Change in bowel habits

What should the nurse include in a teaching plan for a client taking calcium channel blockers such as Nifedipine (Procardia)? (Select all that apply.)

Change positions slowly. Report peripheral edema. Avoid drinking grapefruit juice

When monitoring a client for hyponatremia, what clinical findings should the nurse consider significant? (Select all that apply.)

Confusion Poor tissue turgor

A nurse is teaching a client about the differences between the terms saturated and unsaturated, when used in reference to fats. Which important factor in relation to these types of fats should the nurse include in the teaching?

Density

A nurse is caring for a client with right-sided heart failure. Which are key features of right-sided heart failure? (Select all that apply.)

Dependent edema Distended abdomen Polyuria at night

A nurse is assessing a client with a diagnosis of diabetes insipidus. For which signs indicative of diabetes insipidus should the nurse assess the client? (Select all that apply.)

Excessive thirst Dry mucous membranes Decreased urine specific gravity

Before a client's discharge after a thyroidectomy, the nurse teaches the client to observe for signs of surgically induced hypothyroidism. What clinical indicators identified by the client provide evidence that the nurse's instructions are understood? (Select all that apply.)

Fatigue Dry skin

A nurse is caring for a client who had an adrenalectomy. For what clinical response should the nurse monitor while steroid therapy is being regulated?

Hypotension

A nurse is providing postoperative care for a client one hour after the client had an adrenalectomy. Maintenance steroid therapy has not begun yet. The nurse should monitor the client for which complication?

Hypotension

A client with a long history of alcohol abuse develops cirrhosis of the liver. The client exhibits the presence of ascites. The nurse concludes that the most likely cause of this client's ascites is:

Impaired portal venous return

Which clinical indicator is the nurse most likely to identify when exploring the history of a client with open-angle glaucoma?

Impairment of peripheral vision

A client is admitted with a head injury. The nurse identifies that the client's urinary retention catheter is draining large amounts of clear, colorless urine. What does the nurse identify as the most likely cause?

Inadequate antidiuretic hormone (ADH) secretion

Thrombus formation is a danger for postoperative clients. Which independent interventions should the nurse perform to prevent this complication? (Select all that apply.)

Instruct the client to avoid crossing the legs Instruct the client to dorsiflex the feet routinely

A nurse is caring for several clients with type 1 diabetes, and they each have a prescription for a specific type of insulin. Which insulin does the nurse conclude has the fastest onset of action?

Insulin lispro (Humalog)

A client who had a subtotal thyroidectomy asks how hypothyroidism may develop when the problem was hyperthyroidism. What should the nurse consider when formulating a response?

Less thyroid tissue is available to supply thyroid hormone after surgery

A client reports experiencing nausea, dyspnea, and right upper quadrant pain unrelieved by antacids. The pain occurs most often after eating in fast-food restaurants. Which diet should the nurse instruct the client to follow?

Low fat

When developing a plan of care for a client who had a cardiac catheterization via a femoral insertion site, the nurse should include:

Maintaining the supine position for a minimum of four hours

A client comes to the emergency department reporting symptoms of the flu. When the health history reveals intravenous drug use and multiple sexual partners, acute retroviral syndrome is suspected, and a test for the human immunodeficiency virus (HIV) is performed. Which clinical responses are associated most commonly with this syndrome? (Select all that apply.)

Malaise Swollen lymph glands

A nurse reviews the laboratory test results of a client with emphysema who is recovering from a myocardial infarction. The nurse obtains the client's vital signs and performs a physical assessment. Which prescribed medication should the nurse consider the priority at this time?

Metoprolol (Lopresor)

The postoperative prescriptions for a client who had repair of an inguinal hernia include docusate sodium (Colace) daily. Before discharge, the nurse teaches the client that an intermittent side effect of this medication may be:

Mild abdominal cramping

A nurse is caring for a client with the clinical manifestation of hypotension associated with a diagnosis of Addison disease. Which hormone is impaired in its production as a result of this disease?

Mineralocorticoids

The serum ammonia level of a client with hepatic cirrhosis and ascites is elevated. The priority nursing intervention is to:

Observe the client for increasing confusion

A client with pulmonary tuberculosis is being treated at home. To help control the spread of the disease, the nurse instructs the client to:

Open the windows frequently to allow air to circulate throughout the house

A nurse teaches a client about warfarin (Coumadin). Which information is essential for the nurse to include in the education plan?

Periodic blood testing is necessary.

A nurse is teaching menu planning to a client who has a high triglyceride level. Which nutrient avoided by the client indicates that teaching about foods that are high in fat was understood?

Red meat

After a basal cell carcinoma is removed by fulguration, a client is given a topical steroid to apply to the surgical site. The nurse evaluates that the teaching regarding steroids and skin lesions is effective when the client states that the primary purpose of the medication is to:

Reduce inflammation at the surgical site

A client is diagnosed with diabetic ketoacidosis. Which insulin should the nurse expect the health care provider to prescribe?

Regular insulin (Novolin R)

When teaching about the dietary control of gout, the nurse evaluates that the dietary teaching is understood when the client states; "I will avoid eating:

Shellfish

A client is recovering from an acute episode of alcoholism that included esophageal involvement. What are the components of a therapeutic diet that are most appropriate for the nurse to include in the teaching plan for this client? (Select all that apply.)

Soft diet High protein diet High carbohydrate diet

When a client has a myocardial infarction, one of the major manifestations is a decrease in conductive energy provided to the heart. What is most important for the nurse to assess that has a direct relationship to the action potential of the heart?

Strength of contractions

The nurse provides a list of appropriate food choices to a client with newly diagnosed diabetes. The client reviews the list and says, "I do not like and refuse to eat asparagus, broccoli, and mushrooms." In response, the nurse teaches the client about the food exchange list. The nurse evaluates that the teaching is understood when the client states, "Instead of asparagus, broccoli, and mushrooms, I can eat:

String beans, beets, or carrots."

A person is brought to the emergency department after prolonged exposure to cold weather. What clinical manifestations of hypothermia does the nurse expect? (Select all that apply.)

Stupor Paresthesia in affected body parts

A nurse is caring for a client who had pelvic surgery. The nurse should monitor for which clinical manifestations of thrombophlebitis? (Select all that apply.)

Tender area on the leg Warm area over the calf

A nurse is caring for a client who was diagnosed with a myocardial infarction. While caring for the client two days after the event, the nurse identifies that the client's temperature is elevated. The nurse concludes that this increase in temperature is most likely the result of:

Tissue necrosis

A client is diagnosed with hyperthyroidism and is experiencing exophthalmia. Which measures should the nurse include when teaching this client how to manage the discomfort associated with exophthalmia? (Select all that apply.)

Use tinted glasses. Elevate the head of the bed 45 degrees. Tape eyelids shut at night if they do not close.

A nurse has difficulty palpating the pedal pulse of a client with venous insufficiency. What action should the nurse take next?

Verify the pulse by using a Doppler

A client reports pain as a result of a gastric ulcer. What clinical findings is the nurse most likely to identify during an assessment of the client's pain? (Select all that apply.)

Vomiting relieves pain. Pain described as gnawing. Pain occurs a half hour after meals.

The nurse is making rounds on a patient who has developed severe bone marrow depression after receiving chemotherapy for cancer. Which of these actions by the nurse is appropriate? (Select all that apply.)

Wash hands before entering the client's room. Advise use of a soft toothbrush for oral hygiene. Report an elevation in temperature immediately.

A nurse is preparing to administer a nasogastric tube feeding. List the steps of the procedure in the order in which they should be performed. 1. Instill the prescribed solution. 2. Wash the hands. 3. Aspirate the contents of the stomach. 4. Document the client's response to the procedure. 5. Verify the solution to be administered.

Wash the hands. Verify the solution to be administered. Aspirate the contents of the stomach. Instill the prescribed solution. Document the client's response to the procedure.

A nurse is preparing to administer insulin to a client with diabetes. In which order should the nurse perform the actions associated with insulin administration?

Washing the hands prevents cross contamination. Rotating the insulin vial distributes the drug evenly throughout the vial. Wiping the seal on the insulin vial prevents contamination of the needle and the fluid. Instilling air into the vial increases the pressure in the closed space so that the correct amount of fluid finally can be withdrawn.

When obtaining the history of a client recently diagnosed with type 1 diabetes, the nurse expects to identify the presence of:

Weight loss

What clinical indicators should a nurse expect when assessing a client with hyperthyroidism? (Select all that apply.)

Weight loss Tachycardia Restlessness Exophthalmos

When assessing a client with Graves disease, the nurse expects to identify:

Weight loss, exophthalmos, and restlessness

When assessing a wound that is healing by secondary intention, the nurse can classify it according to its condition and color. How should the nurse classify a wound that exhibits some soft necrotic tissue with a semiliquid slough and exudate?

Yellow


Ensembles d'études connexes

Intro to Dance Final Exam Review

View Set

Just studying for Anesthesia (from handouts)

View Set

VNSG1423 MODULE2: with ASSIGNMENT for Y's

View Set

B&G Chapter 6 Practice Questions

View Set

The Safe & Effective Care Environment: The Management of Care Practice Questions

View Set

Musculoskeletal modalities Prep U

View Set

International Business Chapter 6 (Trade Protectionism)

View Set

ExamFX Chapter 21: Workers Compensation - General Concepts

View Set